You are on page 1of 207

Version I :

1. Which one of the following was a critical figure in the evolution of modern psychology?
a) John B.Watson
b) WilhelmWundt
c) WilliamJames
d) SigmundFreud
2. What is functionalism?
a) The perspective on mind and behavior that focuses on the examination of their
functions in an organism’s interactions with the environment.
b) The view that all human mental experience can be understood as a combination of
simple elements or events.
c) The perspective according to which complex phenomena should be explained by the
simplest underlying principles.
d) Theory that all behavior is a result of past learning.
3. During puberty, researchers have documented particularly important changes in
which of the following?
a) Limbic system and Frontal lobes
b) Frontal lobes and Hippocampus
c) Hippocampus and Limbic system
d) None of above
4. A behavioral psychologist would probably say that:
a) Cognition is most important when analyzing behavior.
b) We act based on rewards and punishments received.
c) The unconscious stimulates our behavior.
d) Introspection is a useful way to study mental processes.
5. A sample of 40 university students are given either vitamin B, or a placebo. Before
starting the experiment, and then after taking the pills for two weeks, the students fill
out a survey about their energy levels. The dependent variable is
a) The student’s energy level.
b) The two weeks of the experiment.
c) The vitamin B
d) Placebo pill
6. Which one of the following is a multidisciplinary field that attempts to
understand the brain processes that underlie higher cognitive functions in humans?
a) Cognitive neuroscience
b) Behavioral neuroscience
c) Biological perspective
d) Evolutionary perspective
7. are rules for participation in conversations; social conventions
for communicating, sequencing sentences, and responding appropriately to
others.
a) Phonology
b) Phonetics
c) Pragmatics
d) None of above
8. Which of the following is a research design that uses each participant as his or her
own control? For example, the behavior of an experimental participant before receiving
treatment might be compared to his or her behavior after receiving treatment.
a) Within-subjects design
b) Between-subjects design
c) Placebo control
d) A random sapling
9. How can we determine if a test has good validity?
a) It measures what it is supposed to measure
b) It produces the same result no matter which version of the test is used
c) It produces the same result when it is given at different times to the same group of people
d) All of the questions on it can be answered accurately by the subject
10. What is dependent variable?
a) In an experimental setting, a variable that the researcher measures to assess the
impact of a variation in the other variable
b) In an experimental setting, a variable that the researcher manipulates with the
expectation of having an impact on values of the other variable
c) The factor manipulated by there searcher
d) A variable that represents a quantity that is being manipulated in an experiment
11. What is habituation ?
a) A frequently repeated stimulus.
b) An increase in behavioral response when a stimulus is presented repeatedly.
c) An increase in brain activity when stimulus is presented
d) A decrease in a behavioral response when a stimulus is presented repeatedly.
12. What is classical conditioning?
a) A method of learning that occurs through rewards and punishments for behavior.
b) A learning procedure in which a biologically potent stimulus (e.g. food) is paired with
a previously neutral stimulus (e.g. a bell).
c) The weakening of a conditioned association in the absence of a reinforcer or
unconditioned stimulus .
d) A form of learning in which animals acquire a response that will allow them to escape
from an aversive stimulus .
13. What is positive reinforcement ?
a) A part of an experiment where participant is presented with visual stimuli
b) A behavior is followed by the removal of a negative stimulus, making it more likely
that the behavior will occur again in the future.
c) A behavior is followed by the presentation of a reinforcing stimulus, increasing the
probability of that the behavior occurring again in the future.
d) Forcing a participant to engage a certain behavior
14. Which one of the following is an example of negative reinforcement?
a) A teacher allows students to not take final test if they have perfect attendance. I.e.
takes away negative experience to increase a certain behavior.
b) A teacher compliments students when they answer correctly.
c) At a gym, customers receive a discount if they keep up with their workout routine.
d) A student receives a praise after a musical performance
15. What is a primary reinforcer?
a) Any stimulus, that has no innate biological value.
b) A negative stimulus, that is taken away in order to reinforce a certain behavior
c) Non-biological reinforcer, such as appraisal
d) Biologically determined reinforcer, such as food and water.
16. According to Piaget, which of the two basic processes work in tandem to
achieve cognitive growth?
a) Assimilation and accommodation
b) Object permanence and Scheme
c) Scheme and accommodation
d) None of above
17. As a result of Pavlov’s classical conditioning, what happened when dog heard a ring
of a bell?
a) Dog became aggressive
b) Dog started salivating
c) Dog had no reaction
d) Dog ran away
18. What is behavioral data?
a) A method of self-observation in which participants report their thoughts andfeelings
b) A set of assumptions
c) Scientific predictions
d) Reports of observations about the behavior of organisms and the condition under
which the behavior occurs
19. Which one of the following would be considered the best definition of psychology?
a) The study of individual’s mental processes, such as thinking, planning, and reasoning
b) The scientific study of human behavior
c) The scientific study of the behavior of individuals and their mental processes
d) The study of people’s behavior patterns
20. What is internalization according to Vygotsky?
a) The process through which children absorb knowledge from the social context.
b) An unconscious defense mechanism by which an individual "projects" his or her own
internal characteristics onto the outside world
c) The ability to explain and predict other people’s behavior
d) Being upset easily
21. Which principle suggests that that children must keep track of the relationship
between the order in which words appear and the meanings they express?
a) Operating principle
b) Operant theory
c) Child-directed stimuli
d) Speech principle
22. What prepares the body for emotional responses through the action of
both its sympathetic and parasympathetic divisions?
a) Somatic nervous system
b) Autonomic nervous system(ANS)
c) Medulla
d) Spinal cord
23. Which theory states that an emotional stimulus produces two co-occurring
reactions— arousal and experience of emotion—that do not cause each other.
a) James–Lange theory of emotion
b) Attachment theory
c) Appraisal theory
d) Cannon–Bard theory of emotion
24. What has neuroanatomy research particularly been focused on, as a part of the
limbic system that acts as a gateway for emotion and as a filter for memory?
a) Amygdala
b) Limbic system
c) Hypothalamus
d) Pons
25. Which one of the following is a sequence of internal activities triggered when an
organism is faced with a threat; prepares the body for combat and struggle or for
running away to safety; recent evidence suggests that the response is characteristic
only of males.
a) Withdrawal response
b) Approach behavior
c) Fight-or-flight response
d) None of above
26. Which one of the following best describes psychosomatic disorder?
a) Mental disorder mainly characterized by unclear or shifting self-image
b) Mental disorder attributed to acute stress
c) Physical disorder aggravated by or primarily attributable to prolonged emotional
stress or other psychological causes.
d) None of above
27. What are the big five personality traits?
a) Extraversion, agreeableness, humility, proclivity to anger, conscientiousness
b) Extraversion, agreeableness, conscientiousness, neuroticism, openness to experience
c) Introversion, agreeableness, humility, openness to experience, honesty
d) Introversion, proclivity to anger, humility, agree ableness, honesty

28. Individuals who score high on neuroticism are most likely to be:
a) Anxious, unstable, and temperamenta
b) Careless, frivolous, and irresponsible
c) Quiet , reserved, and shy
d) Talkative, energetic, and assertive
29. What is consistency paradox?
a) The observation that certain psychological theories are not consistent across different
cultures
b) The observation that human emotions have explainable pattern
c) The observation that humans consistently behave in a similar pattern in familiar
situations.
d) The observation that personality ratings across time and among different observers are
consistent while behavior ratings across situations are not consistent
30. Which theory of personality shares the assumption that personality is shaped
by and behavior is motivated by inner forces?
a) Psychodynamic personality theory
b) Social-cognitive personality theory
c) Trait personality theory
d) None of above
31. Which of the following best describes cognitive development?
a) The development of processes of knowing, including imagining, perceiving, reasoning,
and problem solving.
b) The stage where children enter puberty
c) The ability of a young child at the preoperational stage to take the perspective of another
person.
d) None of above
32. Which of the following is the field of psychology that specializes in mental testing in
any of its facets, including personality assessment, intelligence evaluation, and aptitude
measurement.
a) Cognitive psychology
b) Psychometrics
c) Organizational psychology
d) None of above
33. What is the factor of general intelligence underlying all intelligent
performance, According to Spearman?
a) r
b) s
c) N
d) g
34. which form of intelligence is the ability to understand other people and
social interactions?
a) Interpersonal
b) Intrapersonal
c) Spatial
d) Logical-mathematical
35. Which type of psychologist is least likely to focus on genetic aspects of
human psychology?
a) Industrial–organizational psychologists
b) Developmental psychologists
c) Personality psychologists
d) Biological psychologists
36. Georgian University is conducting a study where one group of students are instructed
to get only 8 hours of sleep and another group of students are advised to get 5 hours of
sleep. What type of experimental design is the university using to test how sleeping
affects students’ well-being?
a) Within-subjects design
b) Between-subjects design
c) Repeated measures design
d) None of above
37. Emily is a psychologist who is interested in the effects of noise level on concentration.
She believes that the noisier a room is, the less people will be able to concentrate. To test
her hypothesis, Emily gathers a bunch of volunteers and gives them a book to read in a
noisy room. Afterwards, she tests their memory of the reading the book. Then, she puts
all of the volunteers into a room that's quiet and has them read another book. Finally,
she tests them on the book they read in the quiet room. What type of experimental
design is Emily using?
a) Between-subjects design
b) Within-subjects design
c) Observational study
d) None of above
38. Research supports all of the following conclusions about the perception of faces
EXCEPT which conclusion?
a) infants (6 month and younger) prefer top-heavy visual displays
b) infants (6 months and younger) have better memory for human than monkeyfaces
c) children have better memory for faces from racial groups they see growing up than
faces from other racial groups
d) faces are perceived and remembered configurally rather than part-by-part
39. reliability refers to the:
a) Validity of data
b) Values of data
c) Levels of data
d) Consistency of data
40. Infants prefer visual displays, and at 6monthsrecognize
monkeyfaces human faces.
a) bottom-heavy; worse than
b) top-heavy; as well as
c) top-heavy; worse than
d) bottom-heavy; as well as
41. Which one of the following is the latest version of DSM?
a) V
b) IV
c) VI
d) III
42. What are side effects of serotonin reuptake inhibitors(SSRIs)?
a) Nausea
b) Dizziness
c) Lethargy
d) all of above
43. What class of medication is mostly prescribed for people who suffer from severe
anxiety?
a) SNRIs
b) SSRIs
c) Benzodiazepines
d) None of above
44. What physical symptoms are stress related to?
a) Increased heartrate
b) Tightened muscles
c) Increased blood pressure
d) All of above
45. Which hormone is known as “stress hormone”?
a) Cortisol
b) Serotonin
c) Dopamine
d) None of above
46. How would Freud describeid?
a) The storehouse of the fundamental drives, governmed by pleasure principle
b) The storehouse of individual values
c) “Ideal self” of personality
d) None of above
47. What is the function of ego?
a) To mediate between the unrealistic id and the external real world
b) To incorporates the values and morals of society which are learned from one's parents
and others
c) To engage in primary process thinking, which is primitive, illogical, irrational ,and
fantasy oriented.
d) None of above
48. What is the healthiest defense mechanism?
a) displacement
b) Isolation
c) projection
d) sublimation
49. People who use isolation as their defense mechanism usually:
a) Cut off emotional charge from hurtful situations or separate incompatible attitudes
into logic-tight compartments (holding conflicting attitudes that are never thought of
simultaneously or in relation to each other)
b) Placie blame for one’s difficulties on others or attribute one’s own “forbidden” desires
to others
c) Prevente dangerous desires from being expressed by endorsing opposing attitudes and
types of behavior and using them as “barriers”
d) None of above
50. People who protect self rom unpleasant reality by refusing to perceive it have
which of the following defense mechanisms?
a) Denial of reality
b) Fantasy
c) Sublimation
d) Reaction formation
………………………………………………………………………………………………….
Version II :
1. When a psychologist is using the method of naturalistic observation to find out what
type of public places attract humans the most, psychologist would:

a) carefully design controlled situations in which to observe behavior.


b) rely on observations of subjects' responses to questionnaires.
c) observe behavior as it happens outside the laboratory or clinic.
d) make records of the behavior of clients treated in therapy.

2. The study we use to determine the degree of relationship between (two) to variables is
called:

a) Naturalistic observation.
b) the correlational method.
c) a controlled1 experiment.
d) the survey method.

3. Explanations for aggressive behavior from this perspective include genetic


predisposition, high testosterone level and frontal lobe damage.

a) Cognitive
b) Social
c) Biological
d) Cross-cultural
4. Which perspective attempts to study the structure of conscious experience
through the method of introspection.

a) Functionalism
b) Behaviorism
c) Gestalt
d) Structuralism
5. psychologists note that only the fittest organisms reach maturity and reproduce
,and transmit genes to future generations

a) Social cognitive
b) Behavioral
c) Evolutionary
d) Psychodynamic
6. “The Strange Situation” test, where child is left alone in the room, is used to assess
what aspect of psychological development?
a) Attachment
b) Meta-cognitive awareness
c) Moral reasoning
d) Psychosocial stages
7. Referring to previous study described in question 10, what method was used to
measure effects of vitamin B on student’s energy levels?
a) Between-subjects
b) Within subjects
c) Observational study
d) All of above
8. Who is known as the father of classical conditioning?

a) Ivan Pavlov
b) B.F.Skinner
c) Sigmund Freud
d) Jean-Paul Sartre
9. According to Freud, which inner force contains the libido?
a) Superego
b) Ego
c) Id
d) None of above
10. Which of these might be prescribed to a patient who has been diagnosed with
major depressive disorder?
a) Norepinephrine
b) DSM
c) Acetylcholine
d) SSRIs
11. What is the name of the book that holds the diagnosis criteria and overview
of all documented psychological disorders?
a) Diagnostic and Statistical Manual of Mental Disorders
b) APA Manual of Psychological Disorders
c) Overview of Criteria of Mental Disorders
d) None of Above
12. What experimental technique in which biased expectations of experimenters
are eliminated by keeping both participants and experimental assistants unaware
of which participants have received which treatment.
a) Double-blind control
b) Random technique
c) Placebo control
d) None of above
13. Which one of the following is a definition of representative sample?
a) A subset of a population that closely matches the overall characteristics of the
population with respect to the distribution of males and females, racial and ethnic
groups, and soon
b) The entire set of individuals to which generalizations will be made based on an
experimental sample.
c) A subset of a population selected as participants in an experiment.
d) All of above
14. According to Piaget, what is the name of the process whereby new cognitive elements
are fitted in with old elements or modified to fit more easily; this process works in
tandem with accommodation
a) Scheme
b) Assimilation
c) Perception
d) None of above
15. What ages does formal operation stage cover?
a) From 1 to3
b) From 4 to7
c) From age 11on.
d) None of above
16. What is theory of mind?
a) The ability to explain and predict other people’s behavior based on an understanding
of their mental states.
b) The ability to clearly perceive other’s speech
c) The ability to express oneself
d) None of above
17. Who was one of the first psychologists to emphasize the pervasive role of
immediate, unlearned affective (emotional)reactions?
a) Paul Ekman
b) David Neal
c) William James
d) Silvan Tomkins
18. Which one of the following is a peripheral-feedback theory of emotion stating
that an eliciting stimulus triggers a behavioral response that sends different sensory
and motor feedback to the brain and creates the feeling of a specific emotion.
a) James–Lange theory of emotion
b) Theory of emotional response
c) Cannon–Bard theory of emotion
d) None of above
19. What is subjective well-being?
a) The processes through which people change the intensity and duration of the emotions
they experience
b) Individuals’ overall evaluation of life satisfaction and happiness.
c) Both A and B
d) None of above
20. What is stressor?
a) An internal or external event or stimulus that induces stress.
b) An internal or external event that decreases stress
c) An emotional state
d) Nona of above
21. Which one of the following is a response to stressors that is hypothesized to be typical
for females; stressors prompt females to protect their offspring and join social groups to
reduce vulnerability?
a) Fight-or-Flight
b) Tend-and-befriend response
c) Crying
d) None of above
22. How many stages does general adaptation syndrome (GAS)involve?
a) 1
b) 2
c) 3
d) 4
23. Before you participate in an experiment, the researcher should provide you with
information about procedures, potential risks, and expected benefits. This process is
called
a) A risk/gain assessment.
b) Informed debriefing.
c) Informed consent.
d) Operational definitions
24. Freud believed that every young boy has an innate impulse to view his father as
asexual rival for his mother’s attentions. What was the name of this complex?
a) Zeus
b) Oedipus
c) Inadequacy complex
d) None of above

25. When does Freudian slip occur?


a) When an unconscious desire is betrayed by your speech or behavior
b) When your unconscious desires are repressed by superego
c) When you’re unable to progress normally to the next stage of development
d) None of above
26. The perspective draws on the ways in which human mental abilities
serve adaptive purposes.
a) cognitive
b) humanistic
c) evolutionary
d) sociocultural
27. Which one of the following defense mechanisms involve retreating to earlier
developmental levels involving more childish responses and usually a lower level
of aspiration
a) Regression
b) Reaction formation
c) Sublimation
d) None of above
28. From the following answers, what would be the best definition ego defense
mechanism?
a) Mental strategy (conscious or unconscious) used by the ego to defend itself against
conflicts experienced in the normal course of life.
b) Physical strategy used by the ego to defend itself against life’s stressors
c) Mental strategy used by ego to withdraw from situations when feeling threatened
d) None of above
29. From psychodynamic perspective, what is an intense emotional response triggered
when a repressed conflict is about to emerge into consciousness?
a) Joy
b) Anxiety
c) Regression
d) None of above
30. What is retrospection?
a) The process of thinking about past events
b) The process of thinking about future events
c) The process of uncovering unconscious thoughts
d) None of above
31. What is androcentric?
a) Female-centered
b) Male-centered
c) Gender neutral
d) None of above
………………………………………………………………………………………………………
version III :
1. Which one of the following psychologists focused on the way in which the mind
understands many experiences as gestalts—organized wholes—rather than as the sums
of simple parts?

a) Max Wertheimer
b) Freud
c) B.F. Skinner
d) Edward Titchener
2. A school of psychology that maintains that psychological phenomena can be understood
only when viewed as organized, structured wholes, not when broken down into
primitive perceptual elements.

a) CBT
b) Gestalt psychology
c) Structuralism
d) None of above

3. Which one of the following is psychodynamic perspective?

a) A psychological model that emphasizes an individual’s phenomenal world and inherent


capacity for making rational choices and developing to maximum potential.
b) A psychological model in which behavior is explained in terms of past experiences and
motivational forces; actions are viewed as stemming from inherited instincts, biological
drives, and attempts to resolve conflicts between personal needs and social requirements.
c) A multidisciplinary field that attempts to understand the brain processes that underlie
behavior.
d) None of above

4. When explaining nature of aggression and violence psychologists who follow


humanistic perspective would look for:
a) Personal values and social conditions that foster self-limiting, aggressive perspectives instead
of growth-enhancing, shared experiences.
b) Consider what conditions would have made aggression an adaptive behavior for early
humans.
c) Study the impact of violence in films and videos, including pornographic violence, on
attitudes toward gun control, rape, and war.
d) None of above

5. Who pioneered the use of electrical stimulation to probe structures deep in the brain.
a) Paul Broca
b) Walter Hess
c) Giacomo Rizzolatti
d) None of above

6. Which brain structure serves as a relay station between the brain and the endocrine
system?
a) the hippocampus
b) the hypothalamus
c) the pons
d) the amygdala
7. Genetic disorder, known as phenylketonuria is associated to have:

a) A potential negative impact on IQ


b) A potential positive impact on visual perception
c) Positive impact on development of bipolar disorder
d) None of above

8. What is crystalized intelligence?

a) the factor of general intelligence underlying all intelligent performance.


b) The facet of intelligence involving the knowledge a person has already acquired and the
ability to access that knowledge; measures by vocabulary, arithmetic, and general
information tests.
c) The aspect of intelligence that involves the ability to see complex relationships and solve
problems.
d) None of above

9. Which one of the following is associated with being at risk for confirming a negative
stereotype of one’s group.

a) Stereotype threat
b) Negative stereotype theory
c) General anxiety disorder
d) None of above
10. Which one of the following is the research effort designed to describe what is
characteristic of a specific age or developmental stage.

a) Normative investigation
b) Experimental observation
c) Correlational design
d) None of above

11. What are the first two weeks after formation of the zygote are known as?

a) Embryonic stage
b) The fetal stage
c) Germinal stage
d) None of above

12. Newborns may find it relatively easy to learn to recognize their mothers’ faces because

a) they have deeper emotional connection with their mother


b) they are associated with voices with which they are already familiar
c) they are associated with a scent with which they are already familiar
d) None of above

13. ______ is recognition that objects exist independently of an individual’s action or


awareness; an important cognitive acquisition of infancy;

a) Object permanence
b) Assimilation
c) Scheme
d) None of above

14. From what age does the concrete operations stage begin?

a) 4
b) 5
c) 10
d) 7

15. What is rich factual knowledge?

a) Knowledge about the contexts of life and their temporal (developmental) relationships
b) General and specific knowledge about the conditions of life and its variations
c) Knowledge about the relative indeterminacy and unpredictability of life and ways to manage
it
d) None of above

16. What’s is concept of a grammatical error, usually appearing during early language
development, in which rules of the language are applied too widely, resulting in
incorrect linguistic forms.
a) Overregularization

b) Pragmatics theory

c) Language-making capacity theory

d) None of above

17. According to Erickson, what “crisis” children from birth to 1 year experience?

a) Autonomy vs. self-doubt


b) Trust vs. mistrust
c) Secure & insecure attachment
d) None of above
18. According to Erickson, what “crisis” do people in their 30s and 40s usually experience?
a) Intimacy vs. Isolation
b) Ego integrity vs. Despair
c) Generativity vs. Stagnation
d) None of above

19. What did John Bowlby (1973), an influential theorist suggested?


a) Infants and adults are biologically predisposed to form attachments.
b) Children usually form anxious attachment
c) Children’s attachment style is easily changed
d) None of above

…………………………………………………………………………………………………………
Version I :

1. Hypothesis is testable explanation of the relationship between two or more variables.


2. Debriefing is the process of reviewing articles that are submitted to scientific journals.
3. Fixation is a state in which a person remains attached to objects or activities more
appropriate for an earlier stage of psychosexual development.
4. Conscious is the domain of the psyche that stores repressed urges and primitive impulses.
5. Libido The psychic energy that drives individuals toward sensual pleasures of all types,
especially sexual ones.
6. Superego is the aspect of personality involved in self-preservation activities and in
directing instinctual drives and urges into appropriate channels.
7. Idis the primitive, unconscious part of the personality that represents the internalization
of society’s values, standards, and morals
8. According to Freud, anxiety is an intense emotional response caused by the preconscious
recognition that a repressed conflict is about to emerge into consciousness.
9. Collective unconscious is the part of an individual’s unconscious that is inherited,
evolutionarily developed, and common to all members of the species.
10. Self-concept is a concept in personality psychology referring to a person’s constant
striving to realize his or her potential and to develop inherent talents and capabilities
11. Repression is the basic defense mechanism by which painful or guilt producing thoughts,
feelings, or memories are excluded from conscious awareness.
12. Scheme is Piaget’s term for a cognitive structure that develops as infants and young
children learn to interpret the world and adapt to their environment.
13. Those who suffer from depression experience persistent feelings of sadness and
hopelessness and lose interest in activities they once enjoyed, though they physical symptoms
are never presented.
14. Individual psychotherapy is the single most frequently used activity of clinicians and still
occupies the largest percentage of a clinician’s time.
15. Anxiety and depression are proven to be negatively correlated with each other.
16. In order for someone to be diagnosed with schizophrenia, they must experience visual
hallucinations.
17. Schizophrenia has only two stages of development.

18. Appearance of cognitive psychology was a challenge to the limits of behaviorism.


19. Object permanence refers to children’s understanding that objects exist and behave
independently of their actions or awareness.
20. a longitudinal design, the same individuals are repeatedly observed and tested over
time, often for many years.
……………………………………………………………………………………………
Version II :

1. With mild, unpleasant stimulation, the sympathetic division is more active; with mild,
pleasant stimulation, the parasympathetic division is more active.
2. There are two distinct systems in the brain that handle approach-related and withdrawal-
related emotional responses.
3. Placing blame for one’s difficulties on others or attributing one’s own “forbidden” desires
to others is reaction formation
4. Experimental group is a group in an experiment that is not exposed to a treatment or does
not experience a manipulation of the independent variable.
5. Within-subjects design is a procedure that ensures that every member of a population has
an equal likelihood of participating in an experiment.
6. Selective optimization with compensation is a strategy for successful aging in which one
makes the most gains while minimizing the impact of losses that accompany normal
aging. According to Freud, Eros, as a broadly defined sexual drive, does not suddenly appear at
puberty but operates from birth. One of the major obstacles of psychosexual development, at
least for boys, occurs in the phallic stage. An archetype is a primitive symbolic representation of
a particular experience or object.
7. Unconditional positive regard is striving to realize one’s inherent potential.
8. Cardinal traits are traits around which a person organizes his or her life.

9. Psychologists define personality as the complex set of psychological qualities that


influence an individual’s characteristic patterns of behavior across different situations and
overtime.
10. B. F. Skinner suggested that children acquire language through ordinary processes of
learning.
11. Developmental psychologists Study the changes that occur in the physical, cognitive, and
social functioning of individuals across the life span; study the influence of genetics and
environments on those changes.
12. A hypothesis is a tentative and testable statement about the relationship between
causes and consequences.
13. A variable is any factor that varies in amount or kind.
………………………………………………………………………………………………………
Version III :

42. Vygotsky’s concept of internalization helps explain the effect culture has on cognitive
development.

52. fluid intelligence shows that it is positively correlated with age


53. Naturalistic observation is an intensive observation of a particular individual or small
group of individuals.
55. Predictions in psychology are statements about the likelihood that a certain behavior
will occur or that a given relationship will be found.
56. The cortex is involved in emotional experiences through its internal neural networks and its
connections with other parts of the body.

57. Self-concept is a concept in personality psychology referring to a person’s constant striving


to realize his or her potential and to develop inherent talents and capabilities.
61. In a longitudinal design, the same individuals are repeatedly observed and tested over time,
often for many years.

62. According to behaviorist theory, behavior is driven, or motivated, by powerful inner forces

63. Maslow, and their colleagues defined a perspective that strives to deal with the whole person,
practicing a holistic approach to human psychology.

64. Positive psychology is a movement within psychology that applies research to provide
people with the knowledge and skills that allow them to experience fulfilling lives.
65. Stressor is the pattern of specific and nonspecific responses an organism makes to stimulus
events that disturb its equilibrium and tax or exceed its ability to cope.

66. According to Rotter, expectancy is the extent to which people believe that their behaviors
in particular situations will bring about rewards.

69. Encodings is the way you categorize information about yourself, other people, events, and
situations.
70. reciprocal determinism a concept of Albert Bandura’s social-learning theory that refers to the
notion that a complex reciprocal interaction exists among the individual, his or her behavior, and
environmental stimuli and that each of these components affects the others.
…………………………………………………………………………………………………….
Version I :
51. 4 goals of psychology are to describe, explain, predictand control behavior
52. Serotonin reuptake inhibitors (SSRIs) are mostly prescribed to patients who are
depressed
53. Benzodiazepines enhance the effect of the neurotransmitter GABA
54. The way you categorize information about yourself, other people, events, and situations is
encoding
55. A belief that one can perform adequately in a particular situation is self-efficacy

56. Generalized evaluation of the self is a person’s self-esteem


57. Terror management theory proposes that self-esteem helps people cope with the
inevitability of death

58. case study is intensive observation of a particular individual or small group of individuals
59. Informed consent is the process through which individuals are informed about
experimental procedures, risks, and benefits before they provide formal consent to
become research participants.
……………………………………………………………………………………………………
Version II :
32. In Maslow’s hierarchy of needs biological physiological needs are considered the most
primary

33. Correlation does not imply causation


34. significant difference A difference between experimental groups or conditions that
would have occurred by chance less than an accepted criterion; in psychology, the
criterion most often used is a probability of less than 5 times out of 100, or p <.05.

35. DSM stands for diagnostic and statistical manual


36. Behavior is the actions by which an organism adjusts to its environment.

37. Psychometrics is the field of psychology that specializes in mental testing in any of its
facets, including personality assessment, intelligence evaluation, and aptitude measurement.
38. Feelings of helplessness and hopelessness, loss of interest in daily activities, appetite or
weight changes could be the symptoms of depression
39. Depression disorder has the highest suicide rate.
………………………………………………………………………………………………………
Version III :
71. To discover the basic elements, Titchener relied on the technique of introspection
72. According to the psychodynamic perspective, behavior is driven, or motivated, by powerful
inner force.

73. Behavioral measures are ways to study overt actions and observable and recordable reactions.

74 self report measures are verbal answers, either written or spoken, to questions the researcher
poses.
75. hallucinogen is a drug that alters cognitions and perceptions and causes
hallucinations.

76. morality is a system of beliefs and values that ensures that individuals will keep their
obligations to others in society and will behave in ways that do not interfere with the rights
and interests of others.

77. gender is a psychological phenomenon that refers to learned sex-related behaviors and
attitudes of males and females.
78.gender stereotype is a belief about attributes and behaviors regarded as appropriate for males
and females in a particular culture.
79. Erikson described intimacy as the capacity to make a full commitment to another person.
80.autonomic nervous system (ANS) prepares the body for emotional responses through the
action of both its sympathetic and parasympathetic divisions
…………………………………………………………………….
1._ _ _ _ _ _ _ _ _ _ _ _ _ _ _fundamental attribution error (FAE) _ _ _ _ _ _ _ _ _ _ _ _ _ _ _ _ _ _ _
represents the dual tendency for people to overestimate dispositional factors (blame or credit
people) and to underestimate situational factors (blame or credit the environment) when searching
for the cause of some behavior or outcome.
2. When people doubt their ability to perform a task, they may engage in _ _ _ _ _ _ self-
handicapping_ _ _ _ _ _ _ _ _ _ _ _ _ _ _ _ _ _ _ _ _ _ _ _ _ _ _ _ — They deliberately sabotage their
performance. The purpose of this strategy is to have a ready-made excuse for failure that does not
imply lack of ability.
3. George L. Engel’s _ _ _ _ _ _ _ _ _ biopsychosocial model of health and illness _ _ _ states that the
causes, manifestations, and outcomes of wellness and diseases are determined by the interactions
between biological, psychological, and social factors.
4. The same situations might cause stress in some people but not in others. According to Lazarus, to
experience some situations as stressful, first step is to evaluate the situations as stressful. _ _ _ _ _ _
_ _ _ _ _ _cognitive appraisal _ _ _ _ _ _ _ _ _ _ _ _ _ _ _ _ _ _ _ _ is the cognitive interpretation and
evaluation of a stressor.
5. According to Schachter, the experience of emotion is the joint effect of the two factors: _ _ _ _ _ _
_ _ _ _ _ _ _ _ _ _physiological arousal _ _ _ _ _ _ _ _ _ _ _ _ _ _ _ _ _ and cognitive appraisal.
6. _ _ _ _ _ _ _ _gender serotypes _ _ _ _ _ _ _ _ _ _ _ _ _ _ _ _ _ _ _ _ _ _ _ _ are beliefs about
attributes and behaviors regarded as appropriate for males and females in a particular culture.
7._ _ _ _ _ _crystallized intelligence _ _ _ _ _ _ _ _ _ _ _ _ _ _ _ _ _ _ _ _ _ _ _ _ _ _ _ _ involves the
knowledge a person has already acquired and the ability to access that knowledge; it is measured
by tests of vocabulary, arithmetic, and general information.
8. Four main goals of psychologists are describing, explaining, _ _ _ _ _ predicting_ _ _ _ _ _ _ _ _ _ _
_ _ _ and controlling behavior
9.When different people observe the same events, because of their personal motives or expectations,
they don’t always “see” the same thing. What can be a remedy / solution for this challenge to
objectivity? _ _ _ _ _ _ standardization and operional definitions_
Parents use various reinforcement schedules to teach children new behavior, strengthen the desired
behavior or try to stop undesired behavior. A continuous schedule is often the best in teaching a
new behavior. Once the response has been learned, _ _ _ _ _ _ _ _positive reinforcement _ _ _ _ _ _ _
_ _ _ _ _ _ _ _ _ _ _ _ can be used to strengthen the new behavior.
After listening to a list of words such as hot, snow, warm, winter, ice, wet, frigid, chilly, heat, weather,
freeze, air, shiver, Arctic, and frost, people often remember having heard the related word cold even
though it was not presented (Gallo, 2006; Roediger & McDermott, 1995).
This happens due to _ _ _ _ _ _ _ _ _ _ _ _ _encoded to long term memory _ _ _ _ _ _ _ _ _ _ _ _ _ _ _
__________________

…………………………………………………………………………..
1. According to Piaget, assimilation restructures or modifies the child’s existing schemes so that
new information is accounted for more completely.
2. Gardner’s Multiple Intelligences theory suggests that whether people people have mathematical
and linguistic intelligence or not is determined by the heritability estimate
3. Retrieval cues are the stimuli available as you search for a particular memory.
4. When a behavior is followed by the removal of an aversive stimulus, the event is called negative
reinforcement.
5. Reflex is a learned response elicited by specific stimuli that have biological relevance for an
organism.
6. Groups make decisions that are riskier and less conservative than would be made by the
members acting alone.
7. A correlation coefficient of -0.7 indicates a weaker relationship between two variables than a
correlation coefficient of 0.4.
8.Each perspective of psychology (the humanistic, cognitive, sociocultural, evolutionary,
humanistic, behaviorist perspectives) stress the biological, psychological, or social-cultural factors
more than another, each of the perspectives are interesting and valuable by themselves but
unfortunately for the science of psychology, the different perspectives contradict one another,
they’re impossible to use in combination and you have to choose one of them that you agree with
more.
9.In a Carr and VanDeusen (2004) study, male college students at a large midwestern university
completed an anonymous survey on which they indicated whether they had ever engaged in
sexually coercive behavior as well as the frequency with which they viewed various forms of
pornography. The researchers found a statistically significant correlation, such that the more
pornography the students reported using, the greater the likelihood that they had committed sexual
violence (Hald, Malamuth, & Yuen, 2010). From this study we can draw the conclusion that using
pornography made the male students more likely to commit sexual violence.
10. Every time Ana posts half-naked photographs of herself, she gets lots of attention, likes and
comments on social media. Now uploading half-naked photos and receiving attention became
strongly associated for her, so the number of such posts on her page is steadily increasing. This is
an example of classical conditioning.
11. The difference between flashbulb and other types of memories is that flashbulb memories are
accurate because processes of reconstruction do not apply to them.
…………………………………………………………………………………………
People tend to take credit for their successes while denying or explaining away responsibility for
their failures. In many situations, people tend to make dispositional attributions for success and
situational attributions for failure. “I got the prize because of my ability”; “I lost the competition
because it was rigged.” These are examples of
A. self-fulfilling prophecy.
B. confirmation bias.
C. self-serving bias.
D. covariation bias.

(1 point) A teacher believes boys are inherently better at math, so he doesn’t spend equally enough
time and energy to helping female and male students. As a result, his male students get higher
scores on final math exam than girls. This difference between the final performance levels between
girls and boys in his class occurred because of the teacher’s
A. social role.
B. informational influence.
C. self-fulfilling prophecy.
D. self-serving bias.

(1 point) When a group makes a decision there’s a tendency to filter out undesirable input so that a
consensus may be reached, especially if it is in line with the group leader’s viewpoint. This tendency
is referred to as
A. group conformity.
B. group polarization.
C. groupthink.
D. group influence.

(1 point) When examining a patient’s problem every doctor in a team came to the same conclusion
except one of them but even though he had a different opinion he decided not to voice his different
view so that he wouldn’t feel fool and deviant from others. This is an example of
A. conformity.
B. social cognition.
C. obedience.
D. social role.

(1 point) According to Rogers, self-actualization is


A. the use of psychological theory to explain an individual’s course through life.
B. a person’s mental model of his or her typical behaviors and unique qualities.
C. a constant striving to realize one’s inherent potential.
D. complete love and acceptance of an individual by another person.

(1 point) The values for the Independent Construal of the Self in individualistic cultures (such as the
USA) are:
A. connectedness with others, group harmony and social relations.
B. personal freedom, self-fulfillment, independence, self-reliance.
C. loyalty to one’s group, duty to one’s family, adherence to social traditions.
D. enhancement of one’s groups’ status and well-being.

(1 point) Some people believe that events in their life derive primarily from their own actions. For
example, when receiving exam results, they tend to praise or blame themselves and their abilities,
not the teacher or the exam test. These people are said to have
A. internal locus of control.
B. acquired introversion.
C. external locus of control.
D. acquired extroversion.

(1 point) Self-concept refers to


A. the idea that people observe themselves to figure out the reasons they act as they do.
B. a person’s mental model of his or her typical behaviors and unique qualities.
C. the set of beliefs that one can perform adequately in a particular situation.
D. a generalized evaluative attitude toward the self that influences both moods and behavior.

(1 point) Which of the following is not a source of self-efficacy judgments?


A. Vicarious experience (your observations of the performance of others).
B. Persuasion (others may convince you person that you can do something, or you may convince
yourself).
C. Monitoring of your emotional arousal as you think about or approach a task (e.g. anxiety suggests
low expectations of efficacy; excitement suggests expectations of success).
D. Self-handicapping (you anticipate failure and you deliberately sabotage your performance so that
you can have a ready-made excuse for failure).

(1 point) Which of the following is not correct?


A. Wellness is a general condition of soundness and vigor of body and mind; not simply the absence
of illness or injury.
B. Health promotion is the development and implementation of general strategies and specific tactics
to eliminate or reduce the risk that people will become ill.
C. Health is a general condition of soundness and vigor of body and mind; not simply the absence of
illness or injury.
D. Wellness is optimal health, incorporating the ability to function fully and actively over the
physical, intellectual, emotional, spiritual, social, and environmental domains of health..

(1 point) According to the Transtheoretical Model (Stages of Change), when changing their
unhealthy behaviors to healthy ones, individuals move through six stages of change:

A. precontemplation, contemplation, preparation, action, maintenance, and termination.


B. action, termination, precontemplation, contemplation, preparation and maintenance.
C. preparation, precontemplation, contemplation, action, maintenance, and termination.
D. contemplation, precontemplation, action, maintenance, preparation, and termination.

(1 point) Failing to adhere to treatment regimens that doctors give to their patients is one of the
most serious problems in health care. Which of the following statements about patient adherence is
correct?
A. Generally, patients who perceive greater threat from a disease also show greater likelihood to
adhere to treatment regimen given by a doctor.
B. Patients who face serious diseases that leave them in poor physical health always show higher
levels of adherence than patients who are less debilitated by the same diseases.
C. Physicians should stop trying to understand their patients’ attitudes and modify their behaviors to
match those attitudes.
D. When patients can rely on social support that allows them to accomplish their regimens correctly
they are less likely to adhere to treatments.

(1 point) Out of the seven strategies listed below, which ones does Problem-Directed Stress Coping
involve?
1. Fight (destroy, remove, or weaken the threat)
2. Somatically focused activities (use of antianxiety medication, relaxation, biofeedback)
3. Flight (distance oneself from the threat)
4. Cognitively focused activities (planned distractions, fantasies, thoughts about oneself)
5. Seek options to fight-or-flight (negotiating, bargaining, compromising)
6. Therapy to adjust conscious or unconscious processes that lead to additional anxiety
7. Prevent future stress (act to increase one’s resistance or decrease strength of anticipated
stress)

A. 1, 2, 3, 6
B. 2, 4, 3, 7
C. 1, 3, 5, 7
D. 2, 3, 5, 6

(1 point) Traditionally psychologists assumed that subjective mental experience of an emotion


preceded the bodily expression or action. However, according to the James-Lange theory of
emotion, the arousal of the physical response precedes the appearance of the emotion (perceiving a
stimulus causes autonomic arousal and other bodily actions that lead to the experience of a specific
emotion). Which of the following would not serve as an appropriate example of James’ and Lange’s
understanding of how we experience emotions?

A. I see a wild animal, I feel fear, and I run away.


B. I see a wild animal, I run, and then I experience the emotion of fear.
C. I cry because I feel sorry.
D. I’m afraid because I tremble.

(1 point) In his experiments Robert Zajonc demonstrated mere exposure effect which describes how
sometimes people have preferences without knowing why. The mere exposure effect means that
A. repeated exposure to a stimulus (such as a song, a person, a painting) increases liking of that
stimulus.
B. repeated exposure to a stimulus (such as a song, a person, a painting) makes a perceiver feel
bored of that stimulus.
C. repeated exposure to a stimulus (such a song, a person, a painting) increases perceivability of the
details that went unnoticed before.
D. repeated exposure to a stimulus (such a song, a person, a painting) decreases willing to pay
attention to more details.
(1 point) According to the Kohlberg’s theory of moral development, on the last stage of moral
reasoning a person’s intention to behave morally is
A. To avoid pain or not to get caught
B. To gain acceptance and avoid disapproval
C. To be true to universal principles and feel oneself part of a cosmic direction that transcends social
norms
D. To promote the society’s welfare

(1 point) Children at the preoperational stage (roughly from 2 to 7 years of age) are unable to take
the perspective of another person. For example, a child talking on the phone might pinpoint to a toy
that she/he is talking about with the other person on the phone, she can’t take into account that the
person on the phone can’t see what she/he is pinpointing with his/her fingers to.
In Piaget’s theory, this inability is referred to as
A. centration.
B. conservation.
C. egocentrism.
D. assimilation.

(1 point) As children’s cognitive abilities develop, they come to understand that other people have
cognitive experiences of the world—and that those cognitive experiences may not be exactly the
same. Children become able to explain and predict other people’s behavior based on an
understanding of their mental states. In other words, children develop
A. expectancy theory.
B. theory of mind.
C. equity theory.
D. theory of individual psychology.

(1 point) According to Gardner’s Multiple Intelligences theory, some people have great ability to
plan and understand sequences of movements and they’re more likely to become great dancer or
athletes. This is to say that these people have high
A. bodily-kinesthetic intelligence.
B. existential intelligence.
C. spatial intelligence.
D. naturalist intelligence.

(1 point) Which of the following abilities does emotional intelligence involve?


1. The ability to perceive, appraise, and express emotions accurately and appropriately
2. The ability to use emotions to facilitate thinking, to understand and analyze emotions
3. The ability to use emotional knowledge effectively
4. The ability to regulate one’s emotions to promote both emotional and intellectual growth

A. 1, 2, 3
B. 1, 3, 4
C. 1, 2, 3, 4
D. 2, 3, 4

(1 point) A procedure conducted at the end of an experiment in which the researcher provides the
participant with as much information about the study as possible and makes sure that no
participant leaves feeling confused, upset, or embarrassed is called
A. informed consent.
B. cover story.
C. debriefing.
D. intentional deception.

(1 point) Which one of the following statements about memory is correct?


A. Semantic memories preserve the specific events that you have personally experienced.
B. Recognition is a method of retrieval in which an individual is required to reproduce the
information to which he/she was previously exposed.
C. When individuals are motivated enough to remember information accurately their memories can
be absolutely accurate.
D. No matter how emotionally important a memory or information can be for an individual,
memories are can always be subject to misinformation effect.

(1 point) “Boys are more physically aggressive than girls because a male hormone testosterone is
linked with aggression.” This statement is likely made by a psychologist working in _ _ _
perspective of psychology.
A. the biological perspective
B. the evolutionary perspective
C. the psychodynamic perspective
D. the behaviorist perspective

(1 point) “In this study, Japanese students refrained themselves from expressing as anger and
disgust because in Japanese culture it’s not that desirable to show negative emotions.” .” This
statement is likely made by a psychologist working in _ _ _ perspective of psychology.
A. the humanistic perspective
B. the cognitive perspective
C. the sociocultural perspective
D. the evolutionary perspective

(1 point) “Our beliefs about ourselves matter. Do we see ourselves as in control of things? Do we
view ourselves as relatively superior or inferior? Our answers influence our emotions and actions.”
This statement is likely made by a psychologist working in _ _ _ perspective of psychology.
A. the humanistic perspective
B. the cognitive perspective
C. the sociocultural perspective
D. the behaviorist perspective

(1 point) To test the effect of a new drug on depression, a researcher randomly assigns people to
control and experimental groups. Those in the experimental group take a pink pill containing the
new medication; those in the control group take a pink pill that contains no medication. Which
statement is true?
A. The medication is the dependent variable.
B. Depression is the independent variable.
C. Participants in the control group take a placebo.
D. Participants in the experimental group take a placebo.

(1 point) A researcher wants to test the hypothesis that people are less likely to help a person having
a seizure in the street if there are some other people witnessing this situation than if they’re alone.
In the experiment he plans, the researcher will vary the number of people witnessing the situation
where an actor (an assistant of the experimenter) acts as if he’s having a seizure. The number of
people will be his
A. placebo control.
B. double-blind control.
C. independent variable.
D. dependent variable.

(1 point) A double-blind procedure is often used to prevent researchers’ biases from influencing the
outcome of an experiment. In this procedure,
A. only the participants know whether they are in the control group or the experimental group.
B. experimental and control group members will be carefully matched for age, sex, income, and
education level.
C. neither the participants nor the researchers know who is in the experimental group or control
group.
D. someone separate from the researcher will ask people to volunteer for the experimental group or
the control group.

(1 point) Ana started working for a pet food store. In the beginning she was annoyed by the smell in
the store but after some period of time she got used to it. What is the technical term for “getting
used to bad smell” to the extent that it doesn’t bother Ana, and that she doesn’t react to it
anymore?
A. Sensitization.
B. Stimulus discrimination.
C. Habituation.
D. Response acquisition.

(1 point) When playing with a slot machine players have no way of knowing how many times they
have to play before they win. All they know is that eventually, a play will win. This is why slot
machines are so effective, and players are often reluctant to quit. There is always the possibility
that the next coin they put in will be the winning one.
What reinforcement contingency do they psychologists use to design slot machines?
A. Variable-Ratio Schedule
B. Fixed-Ratio Schedule
C. Fixed-Interval Schedule
D. Intermittent Reinforcement Schedule

(1 point) When a toddler throws a tantrum in the store, parents usually refuse to give in. But once in
a while, when they’re tired or in a hurry, they decide to buy the candy, thinking they will do it just
that one time. This is an example that in daily life, parents sometimes unknowingly _ _ undesired
behavior.
A. reinforce
B. extinct
C. punish
D. discriminate

(1 point) Students usually remember what was they read in the beginning and the end of a book
chapter and they don’t remember the middle sections quite well. This is due to
A. reconstructive nature of memory.
B. the serial position effect.
C. semantic encoding principle.
D. encoding specificity effect.

(1 point)Tom waits remembers he was born on 7th December, 1949. This is his
A. semantic memory
B. procedural memory
C. episodic memory
D. flashbulb memory

(1 point) Suppose you’re on your way home from watching a scary movie about a hitchhiker who
was a murderer when you see someone talking loudly with a friend. Because you saw the movie, you
assume that you are witnessing an argument that will probably end in a fight. This is an example of
A. priming.
B. meta-memory.
C. context effects.
D. spontaneous recovery.
Introduction to Psychology Final Exam Test
This test involves questions based on all chapters included in the syllabus (7 fill in the gaps questions, 7
true/false questions, 21 multiple-choice questions).
Total 35 points

Fill in the gaps questions


Fill in the gaps.

(1 point)
_ fundamental attribution error_ _ _ _ _ _ _ _ _ _ _ _ _ _ _ _ _ _ _ _ _ _ _ _ _ _ _ _ _ _ _ _ _ represents
the dual tendency for people to overestimate dispositional factors (blame or credit people) and to
underestimate situational factors (blame or credit the environment) when searching for the cause of
some behavior or outcome.

(1 point)
When people doubt their ability to perform a task, they may engage in _self-handicapping _ _ _ _ _ _ _
_ _ _ _ _ _ _ _ _ _ _ _ _ _ _ _ _ _ _ _ _ _ _ _ _ _ — They deliberately sabotage their performance. The
purpose of this strategy is to have a ready-made excuse for failure that does not imply lack of ability.

(1 point)
George L. Engel’s _ _ _ _ biopsychosocial model of health and illness._ _ _ _ _ _ _ _ _ _ _ _ _ _ _ _ _ _ _ _
_ _ _ _ _ _ _ _ _ _ states that the causes, manifestations, and outcomes of wellness and diseases are
determined by the interactions between biological, psychological, and social factors.

(1 point)
The same situations might cause stress in some people but not in others. According to Lazarus, to
experience some situations as stressful, first step is to evaluate the situations as stressful. _ _ _ _ _ _ _
_ _ _ _ _ _ _ _ _ _ _ _ cognitive appraisal_ _ _ _ _ _ _ _ _ _ _ _ _ is the cognitive interpretation and
evaluation of a stressor.

(1 point)
According to Schachter, the experience of emotion is the joint effect of the two factors: _ _ _ _ _ _ _ _
_ _ _ _ psychological arousal_ _ _ _ _ _ _ _ _ _ _ _ _ _ _ _ _ _ _ _ _ and cognitive appraisal.

(1 point)
_ _ _ _ _ _ _ _ _ _ gender streotypes /gender role_ _ _ _ _ _ _ _ _ _ _ _ _ _ _ _ _ _ _ _ _ _ are beliefs
about attributes and behaviors regarded as appropriate for males and females in a particular culture.

(1 point)
_ _ _ _ _ _ _ _ _ _ _ _ _ _ _ _ crystallized intelligence_ _ _ _ _ _ _ _ _ _ _ _ _ _ _ _ _ _ involves the
knowledge a person has already acquired and the ability to access that knowledge; it is measured by
tests of vocabulary, arithmetic, and general information.

True or False questions


Read each statement and indicate whether they are correct by underlining either True or False.

(1 point)
According to Piaget, assimilation restructures or modifies the child’s existing schemes so that new
information is accounted for more completely.
True or False?

(1 point)
Gardner’s Multiple Intelligences theory suggests that whether people people have mathematical and
linguistic intelligence or not is determined by the heritability estimate.
True or False?

(1 point)
Retrieval cues are the stimuli available as you search for a particular memory.
True or False?

(1 point)
When a behavior is followed by the removal of an aversive stimulus, the event is called negative
reinforcement.
True or False?

(1 point)
Reflex is a learned response elicited by specific stimuli that have biological relevance for an organism.
True or False?

(1 point)
Groups make decisions that are riskier and less conservative than would be made by the members
acting alone.
True or False?
(1 point)
A correlation coefficient of -0.7 indicates a weaker relationship between two variables than a
correlation coefficient of 0.4.
True or False?

Multiple-choice questions
Circle the correct answer.

(1 point)
People tend to take credit for their successes while denying or explaining away responsibility for their
failures. In many situations, people tend to make dispositional attributions for success and situational
attributions for failure. “I got the prize because of my ability”; “I lost the competition because it was
rigged.” These are examples of
A. self-fulfilling prophecy.
B. confirmation bias.
C. self-serving bias.
D. covariation bias.

(1 point)
A teacher believes boys are inherently better at math, so he doesn’t spend equally enough time and
energy to helping female and male students. As a result, his male students get higher scores on final
math exam than girls. This difference between the final performance levels between girls and boys in
his class occurred because of the teacher’s
A. social role.
B. informational influence.
C. self-fulfilling prophecy.
D. self-serving bias.

(1 point)
When a group makes a decision there’s a tendency to filter out undesirable input so that a consensus
may be reached, especially if it is in line with the group leader’s viewpoint. This tendency is referred
to as
A. group conformity.
B. group polarization.
C. groupthink.
D. group influence.

(1 point)
When examining a patient’s problem every doctor in a team came to the same conclusion except one
of them but even though he had a different opinion he decided not to voice his different view so that
he wouldn’t feel fool and deviant from others. This is an example of
A. conformity.
B. social cognition.
C. obedience.
D. social role.

(1 point)
According to Rogers, self-actualization is
A. the use of psychological theory to explain an individual’s course through life.
B. a person’s mental model of his or her typical behaviors and unique qualities.
C. a constant striving to realize one’s inherent potential.
D. complete love and acceptance of an individual by another person.

(1 point)
The values for the Independent Construal of the Self in individualistic cultures (such as the USA) are:
A. connectedness with others, group harmony and social relations.
B. personal freedom, self-fulfillment, independence, self-reliance.
C. loyalty to one’s group, duty to one’s family, adherence to social traditions.
D. enhancement of one’s groups’ status and well-being.

(1 point)
Some people believe that events in their life derive primarily from their own actions. For example,
when receiving exam results, they tend to praise or blame themselves and their abilities, not the
teacher or the exam test. These people are said to have
A. internal locus of control.
B. acquired introversion.
C. external locus of control.
D. acquired extroversion.

(1 point)
Self-concept refers to
A. the idea that people observe themselves to figure out the reasons they act as they do.
B. a person’s mental model of his or her typical behaviors and unique qualities.
C. the set of beliefs that one can perform adequately in a particular situation.
D. a generalized evaluative attitude toward the self that influences both moods and behavior.

(1 point)
Which of the following is not a source of self-efficacy judgments?
A. Vicarious experience (your observations of the performance of others).
B. Persuasion (others may convince you person that you can do something, or you may convince
yourself).
C. Monitoring of your emotional arousal as you think about or approach a task (e.g. anxiety
suggests low expectations of efficacy; excitement suggests expectations of success).
D. Self-handicapping (you anticipate failure and you deliberately sabotage your performance so
that you can have a ready-made excuse for failure).

(1 point)
Which of the following is not correct?
A. Wellness is a general condition of soundness and vigor of body and mind; not simply the
absence of illness or injury.
B. Health promotion is the development and implementation of general strategies and specific
tactics to eliminate or reduce the risk that people will become ill.
C. Health is a general condition of soundness and vigor of body and mind; not simply the absence
of illness or injury.
D. Wellness is optimal health, incorporating the ability to function fully and actively over the
physical, intellectual, emotional, spiritual, social, and environmental domains of health..

(1 point)
According to the Transtheoretical Model (Stages of Change), when changing their unhealthy
behaviors to healthy ones, individuals move through six stages of change:

A. precontemplation, contemplation, preparation, action, maintenance, and termination.


B. action, termination, precontemplation, contemplation, preparation and maintenance.
C. preparation, precontemplation, contemplation, action, maintenance, and termination.
D. contemplation, precontemplation, action, maintenance, preparation, and termination.

(1 point)
Failing to adhere to treatment regimens that doctors give to their patients is one of the most serious
problems in health care. Which of the following statements about patient adherence is correct?
A. Generally, patients who perceive greater threat from a disease also show greater likelihood to
adhere to treatment regimen given by a doctor.
B. Patients who face serious diseases that leave them in poor physical health always show higher
levels of adherence than patients who are less debilitated by the same diseases.
C. Physicians should stop trying to understand their patients’ attitudes and modify their behaviors
to match those attitudes.
D. When patients can rely on social support that allows them to accomplish their regimens
correctly they are less likely to adhere to treatments.
(1 point)
Out of the seven strategies listed below, which ones does Problem-Directed Stress Coping involve?
1. Fight (destroy, remove, or weaken the threat)
2. Somatically focused activities (use of antianxiety medication, relaxation, biofeedback)
3. Flight (distance oneself from the threat)
4. Cognitively focused activities (planned distractions, fantasies, thoughts about oneself)
5. Seek options to fight-or-flight (negotiating, bargaining, compromising)
6. Therapy to adjust conscious or unconscious processes that lead to additional anxiety
7. Prevent future stress (act to increase one’s resistance or decrease strength of anticipated
stress)

A. 1, 2, 3, 6
B. 2, 4, 3, 7
C. 1, 3, 5, 7
D. 2, 3, 5, 6

(1 point)
Traditionally psychologists assumed that subjective mental experience of an emotion preceded the
bodily expression or action. However, according to the James-Lange theory of emotion, the arousal of
the physical response precedes the appearance of the emotion (perceiving a stimulus causes
autonomic arousal and other bodily actions that lead to the experience of a specific emotion). Which
of the following would not serve as an appropriate example of James’ and Lange’s understanding of
how we experience emotions?

A. I see a wild animal, I feel fear, and I run away.


B. I see a wild animal, I run, and then I experience the emotion of fear.
C. I cry because I feel sorry.
D. I’m afraid because I tremble.

(1 point)
In his experiments Robert Zajonc demonstrated mere exposure effect which describes how
sometimes people have preferences without knowing why. The mere exposure effect means that
A. repeated exposure to a stimulus (such as a song, a person, a painting) increases liking of that
stimulus.
B. repeated exposure to a stimulus (such as a song, a person, a painting) makes a perceiver feel
bored of that stimulus.
C. repeated exposure to a stimulus (such a song, a person, a painting) increases perceivability of
the details that went unnoticed before.
D. repeated exposure to a stimulus (such a song, a person, a painting) decreases willing to pay
attention to more details.

(1 point)
According to the Kohlberg’s theory of moral development, on the last stage of moral reasoning a
person’s intention to behave morally is
A. To avoid pain or not to get caught
B. To gain acceptance and avoid disapproval
C. To be true to universal principles and feel oneself part of a cosmic direction that transcends
social norms
D. To promote the society’s welfare

(1 point)
Children at the preoperational stage (roughly from 2 to 7 years of age) are unable to take the
perspective of another person. For example, a child talking on the phone might pinpoint to a toy that
she/he is talking about with the other person on the phone, she can’t take into account that the
person on the phone can’t see what she/he is pinpointing with his/her fingers to.
In Piaget’s theory, this inability is referred to as
A. centration.
B. conservation.
C. egocentrism.
D. assimilation.

(1 point)
As children’s cognitive abilities develop, they come to understand that other people have cognitive
experiences of the world—and that those cognitive experiences may not be exactly the same.
Children become able to explain and predict other people’s behavior based on an understanding of
their mental states. In other words, children develop
A. expectancy theory.
B. theory of mind.
C. equity theory.
D. theory of individual psychology.

(1 point)
According to Gardner’s Multiple Intelligences theory, some people have great ability to plan and
understand sequences of movements and they’re more likely to become great dancer or athletes. This
is to say that these people have high
A. bodily-kinesthetic intelligence.
B. existential intelligence.
C. spatial intelligence.
D. naturalist intelligence.
(1 point) Which of the following abilities does emotional intelligence involve?
1. The ability to perceive, appraise, and express emotions accurately and appropriately
2. The ability to use emotions to facilitate thinking, to understand and analyze emotions
3. The ability to use emotional knowledge effectively
4. The ability to regulate one’s emotions to promote both emotional and intellectual growth

A. 1, 2, 3
B. 1, 3, 4
C. 1, 2, 3, 4
D. 2, 3, 4

(1 point)
A procedure conducted at the end of an experiment in which the researcher provides the participant
with as much information about the study as possible and makes sure that no participant leaves
feeling confused, upset, or embarrassed is called
A. informed consent.
B. cover story.
C. debriefing.
D. intentional deception.

(1 point)
Which one of the following statements about memory is correct?
A. Semantic memories preserve the specific events that you have personally experienced.
B. Recognition is a method of retrieval in which an individual is required to reproduce the
information to which he/she was previously exposed.
C. When individuals are motivated enough to remember information accurately their memories
can be absolutely accurate.
D. No matter how emotionally important a memory or information can be for an individual,
memories are can always be subject to misinformation effect.
Introduction to Psychology Midterm Exam Test
This test involves questions based on syllabus sections from 1st to 6th (5 fill in the gaps questions, 5
true/false questions, 15 multiple-choice questions).
Total 25 points

Fill in the gaps questions


Fill in the gaps.

(1 point) Four main goals of psychologists are describing, explaining, _ _ _ _ _ _ predict_ _ _ _ _ _ _ _ _


_ _ _ _ and controlling behavior.

(1 point) When different people observe the same events, because of their personal motives or
expectations, they don’t always “see” the same thing. What can be a remedy / solution for this
challenge to objectivity? _ _ _ _ _ _ _ _ _ _ _ _ _ _ _ _ _ _ _ _ _ _ kants sulution_ _ _ _ _ _ _ _ _ _ _ _ _ _
________

(1 point) Parents use various reinforcement schedules to teach children new behavior, strengthen the
desired behavior or try to stop undesired behavior. A continuous schedule is often the best in teaching
a new behavior. Once the response has been learned, _ _ _ _ _ _ _ _ _ _ _ _ _positive reinforcement _
_ _ _ _ _ _ _ _ _ _ _ _ _ can be used to strengthen the new behavior.

(1 point) After listening to a list of words such as hot, snow, warm, winter, ice, wet, frigid, chilly, heat,
weather, freeze, air, shiver, Arctic, and frost, people often remember having heard the related word
cold even though it was not presented (Gallo, 2006; Roediger & McDermott, 1995).
This happens due to _ _ _ _ _ _ _ _ _ _ _ _ _ _ _encoding realated world in long term memory _ _ _ _ _
__________________________

True or False questions


Read each statement and indicate whether they are correct by underlining either True or False

(1 point) Each perspective of psychology (the humanistic, cognitive, sociocultural, evolutionary,


humanistic, behaviorist perspectives) stress the biological, psychological, or social-cultural factors
more than another, each of the perspectives are interesting and valuable by themselves but
unfortunately for the science of psychology, the different perspectives contradict one another, they’re
impossible to use in combination and you have to choose one of them that you agree with more.

True or false? false

(1 point) In a Carr and VanDeusen (2004) study, male college students at a large midwestern university
completed an anonymous survey on which they indicated whether they had ever engaged in sexually
coercive behavior as well as the frequency with which they viewed various forms of pornography. The
researchers found a statistically significant correlation, such that the more pornography the students
reported using, the greater the likelihood that they had committed sexual violence (Hald, Malamuth,
& Yuen, 2010).
From this study we can draw the conclusion that using pornography made the male students more
likely to commit sexual violence.
True or false? true

(1 point) Every time Ana posts half-naked photographs of herself, she gets lots of attention, likes and
comments on social media. Now uploading half-naked photos and receiving attention became
strongly associated for her, so the number of such posts on her page is steadily increasing. This is an
example of classical conditioning.
True or false? false

(1 point) The difference between flashbulb and other types of memories is that flashbulb memories
are accurate because processes of reconstruction do not apply to them.
True or false? False

Multiple-choice questions
Circle the correct answer.

(1 point) “Boys are more physically aggressive than girls because a male hormone testosterone is
linked with aggression.” This statement is likely made by a psychologist working in _ _ _ perspective of
psychology.

A. the biological perspective


B. the evolutionary perspective
C. the psychodynamic perspective
D. the behaviorist perspective

(1 point) “In this study, Japanese students refrained themselves from expressing as anger and disgust
because in Japanese culture it’s not that desirable to show negative emotions.” .” This statement is
likely made by a psychologist working in _ _ _ perspective of psychology.
A. the humanistic perspective
B. the cognitive perspective
C. the sociocultural perspective
D. the evolutionary perspective

(1 point) “Our beliefs about ourselves matter. Do we see ourselves as in control of things? Do we view
ourselves as relatively superior or inferior? Our answers influence our emotions and actions.” This
statement is likely made by a psychologist working in _ _ _ perspective of psychology.

A. the humanistic perspective


B. the cognitive perspective
C. the sociocultural perspective
D. the behaviorist perspective

(1 point) To test the effect of a new drug on depression, a researcher randomly assigns people to
control and experimental groups. Those in the experimental group take a pink pill containing the new
medication; those in the control group take a pink pill that contains no medication. Which statement
is true?
A. The medication is the dependent variable.
B. Depression is the independent variable.
C. Participants in the control group take a placebo.
D. Participants in the experimental group take a placebo.

(1 point) A researcher wants to test the hypothesis that people are less likely to help a person having a
seizure in the street if there are some other people witnessing this situation than if they’re alone. In
the experiment he plans, the researcher will vary the number of people witnessing the situation
where an actor (an assistant of the experimenter) acts as if he’s having a seizure. The number of
people will be his

A. placebo control.
B. double-blind control.
C. independent variable.
D. dependent variable.

(1 point) A double-blind procedure is often used to prevent researchers’ biases from influencing the
outcome of an experiment. In this procedure,
A. only the participants know whether they are in the control group or the experimental group.
B. experimental and control group members will be carefully matched for age, sex, income, and
education level.
C. neither the participants nor the researchers know who is in the experimental group or control
group.
D. someone separate from the researcher will ask people to volunteer for the experimental group
or the control group.

(1 point) Ana started working for a pet food store. In the beginning she was annoyed by the smell in
the store but after some period of time she got used to it. What is the technical term for “getting used
to bad smell” to the extent that it doesn’t bother Ana, and that she doesn’t react to it anymore?

A. Sensitization.
B. Stimulus discrimination.
C. Habituation.
D. Response acquisition.

(1 point) When playing with a slot machine players have no way of knowing how many times they
have to play before they win. All they know is that eventually, a play will win. This is why slot
machines are so effective, and players are often reluctant to quit. There is always the possibility that
the next coin they put in will be the winning one.
What reinforcement contingency do they psychologists use to design slot machines?
A. Variable-Ratio Schedule
B. Fixed-Ratio Schedule
C. Fixed-Interval Schedule
D. Intermittent Reinforcement Schedule

(1 point) When a toddler throws a tantrum in the store, parents usually refuse to give in. But once in a
while, when they’re tired or in a hurry, they decide to buy the candy, thinking they will do it just that
one time. This is an example that in daily life, parents sometimes unknowingly _ _ undesired
behavior.
A. reinforce
B. extinct
C. punish
D. discriminate

(1 point) Students usually remember what was they read in the beginning and the end of a book
chapter and they don’t remember the middle sections quite well. This is due to

A. reconstructive nature of memory.


B. the serial position effect.
C. semantic encoding principle.
D. encoding specificity effect.

(1 point)Tom waits remembers he was born on 7th December, 1949. This is his
A. semantic memory
B. procedural memory
C. episodic memory
D. flashbulb memory

(1 point) Suppose you’re on your way home from watching a scary movie about a hitchhiker who was
a murderer when you see someone talking loudly with a friend. Because you saw the movie, you
assume that you are witnessing an argument that will probably end in a fight. This is an example of
A. priming.
B. meta-memory.
C. context effects.
D. spontaneous recovery.
Question N1:

In order to achieve a fuller understanding of the psychological phenomenon in question


it is important to combine multiple psychological perspectives.

Answer: True
Point: 0.50
Question N2:

_ _ _ is an approach to psychology that emphasizes the inherited, adaptive aspects of


behavior and mental processes.

Answer:

B. Evolutionary perspective

Point: 0.50
Question N3:

In _ _ _ early childhood experiences, inner conflicts, biological drives and instincts are
used to explain human behavior.

Answer:

C. psychodynamic perspective

Point: 0.50
Question N4:

“Differences in gender roles may exist due to the way culture shapes our way of
thinking”. This statement is likely made by a psychologist working in _ _ _ perspective of
psychology.

Answer:

the sociocultural perspective

Point: 0.50
Question N5:

In correlational studies researchers manipulate an independent variable and


measurement of its effect on a dependent variable.

Answer: True
Point: 0.00
Question N6:
The disadvantage of experimental research is that researchers cannot establish a
cause-and-effect relationship between variables measured.

Answer: True
Point: 0.00
Question N7:

The advantage of conducting case studies is that researchers can safely generalize the
findings to general population.

Answer: False
Point: 0.50
Question N8:

Which of the following is true about the ethical conduct of psychological research?

Answer:

If research participants are misled about a study they must be fully debriefed at the end
of the study.

Point: 0.50
Question N9:

The most successful managers were video-recorded at their workplace to analyze their
habits. This represents an example of

Answer:

naturalistic observation

Point: 0.50
Question N10:

Which one of the following can’t be a rule for a consumer of psychological information?

Answer:

Be suspicious of complex answers to simple psychological questions.

Point: 0.50
Question N11:

Any institution intending to do a psychological research is required to have an


institutional review board that reviews the ethical aspects of the research before it is
conducted.
Answer: True
Point: 0.50
Question N12:

A research article “Functional Neuroimaging Study Revealed the Impact of Early


Parent-Infant Interactions on Children’s Brain” describes a study. In this study of 125
full-term and premature newborns at Nationwide Children's Hospital in Columbus, Ohio,
found early, gentle displays of affection from parents have lasting effects on how baby
brains react to gentle touch. The authors found that the more the parents hugged their
toddlers, the more developed brains they had. The children that were hugged more also
showed more positive brain responses than children whose parents hugged them less.

The study suggests that

Answer:

All of the above (A, B and C)

Point: 0.00
Question N13:

The _ _ _ method is most likely to use a double-blind design.

Answer:

C. experimental

Point: 0.50
Question N14:

Studying language by listening to people in public places is an example of _ _ _


research.

Answer:

observational

Point: 0.50
Question N15:

Children’s reading skill is measured after taking either a special reading class or a
standard reading class. The dependent variable is

Answer:
Reading skills
Point: 0.50
Question N16:

Under a variable interval schedule, reinforcement will be delivered after a specified


average number of responses have been made.

Answer: False
Point: 0.50
Question N17:

Bedwetting alarms have been used against nocturnal enuresis in adults since the
beginning of the 20th century. The enuresis alarm is triggered when a sensor in the
sheets or night clothes becomes wet with urine. This sets off a signal which wakes the
person to void in the toilet.

Enuresis alarms “teach” the users to wake up when they need to urinate. What kind of
learning is used?

Answer:

A. Classical conditioning

Point: 0.00
Question N18:

Bedwetting alarms have been used against nocturnal enuresis in adults since the
beginning of the 20th century. The enuresis alarm is triggered when a sensor in the
sheets or night clothes becomes wet with urine. This sets off a signal which wakes the
person to void in the toilet.

Enuresis alarms “teach” the users to wake up when they need to urinate. In terms of
classical conditioning, urinating in sleep is

Answer:

Positively punished

Point: 0.50
Question N19:

Children tend to imitate what a model does and says, whether the modeled behavior is
prosocial (positive, constructive, and helpful) or antisocial.

Answer: True
Point: 0.50
Question N20:
In classical conditioning, the consequences of behavior, such as rewards and
punishments, influence the probability that the behavior will occur again

Answer: False
Point: 0.50
Question N21:

People check more frequently for the mail as the delivery time approaches, just like
experimental pigeons peck a key more frequently as the anticipated time for reward
draws near. In this case, both people and pigeons are reinforced with _ _ _ schedules.

Answer:

fixed interval

Point: 0.50
Question N22:

When there is no knowing when the waiting will be over, people’s persistent behavior of
re-checking their e-mails is rewarded with _ _ _ schedules.

Answer:

variable interval

Point: 0.50
Question N23:

In horror movies, sexually arousing images of women are sometimes paired with
violence against women. In terms of classical conditioning principles, after the pairing,
sexual arousal stemming from viewing viewing an attractive semi-nude woman is

Answer:

an unconditioned response

Point: 0.00
Question N24:

As we develop, through _ _ _ we learn cues that lead us to expect and prepare for
good and bad events. Through _ _ _ we learn to repeat behaviors that bring rewards.
And through _ _ _ we watch others and learn. What do psychologists call these three
types of learning?

Answer:
classical conditioning, operant conditioning, observational learning

Point: 0.50
Question N25:

Jason’s parents and older friends all smoke, but they advise him not to. Juan’s parents
and friends don’t smoke, but they say nothing to deter him from doing so. Will Jason or
Juan be more likely to start smoking?

Answer:

Jason

Point: 0.50
Question N26:

The value of elaborative rehearsal over maintenance rehearsal has been cited as
evidence for the Levels of Processing model of memory.

Answer: True
Point: 0.50
Question N27:

If it is easier to remember something in the place where you learned it, you have
context-dependent memory.

Answer: True
Point: 0.50
Question N28:

Specific odors, visual images, emotions, or other associations that help us access a
memory can be examples of

Answer:

retrieval cues.

Point: 0.50
Question N29:

Students usually remember what was they read in the beginning and the end of a book
chapter and they don’t remember the middle sections quite well. This is due to

Answer:

the serial position effect.


Point: 0.50
Question N30:

Judgmental heuristics represent sophisticated rules for making complex decisions or


drawing inferences in an accurate manner requiring our conscious effort.

Answer: False
Point: 0.50
Question N31:

Suppose you wanted to make your friend Ana to feel like a more assertive person.
According to research on _ _ _ , you should ask her to think of _ _ _ times in the past
when she acted in an unassertive manner.

Answer:
Availability heuristic; 12
Point: 0.50
Question N32:

According to research in social psychology, why do many people believe that their
horoscopes are accurate descriptions of who they are and what is likely to happen to
them?

Answer:

Horoscopes are written in a vague way so that most people view them as representative
of their personalities and past behaviors.

Point: 0.50
Question N33:

Even when people knew that the author’s choice of an essay topic was externally
caused, they assumed that what he wrote reflected how he really felt about Castro. That
is, they made a mistake based on self-serving attributional bias

Answer: False
Point: 0.50
Question N34:

“Whether you think you can or you think you can’t, you’re right.” The knowledge of self-
fulfilling prophecy is relevant for understanding this quote from Henry Ford.

Answer: True
Point: 0.50
Question N35:
“If men define situations as real, they are real in their consequences.” The knowledge of
self-fulfilling prophecy is relevant for understanding this quote from W.I. Thomas.

Answer: True
Point: 0.50
Question N36:

Which statement most clearly reflects the fundamental attribution error?

Answer:

People who are unemployed are too lazy to work.

Point: 0.50
Question N37:

“Victory finds a hundred fathers but defeat is an orphan.” (Count Galeazzo Ciano, The
Ciano Diaries, 1938). Match the phrase of a famous author with the corresponding
social psychological notion.

Answer:

Self-serving bias

Point: 0.50
Question N38:

Couples in happy marriages tend to make _ _ _ attributions for each other’s positive
actions (“That’s just the way he is”) and _ _ _ attributions for negative actions (“He’s
just so busy providing for all of us that he doesn’t have the time”).

Answer:
internal | external
Point: 0.50
Question N39:

Normative social influence is present when we conform because we believe that others’
interpretation of an ambiguous situation is correct and can help us choose an
appropriate course of action.

Answer: False
Point: 0.50
Question N40:
In Milgram’s original study, about what proportion of the “teacher-subjects” gave the
maximum shock?

Answer:

about two-thirds

Point: 0.50
Question N41:

The Stanford prison experiment illustrates the power of to influence people’s behavior.

Answer:

the situation

Point: 0.50
Question N42:

What was the independent variable in the Stanford Prison Experiment?

Answer:

prisoner or guard roles

Point: 0.50
Question N43:

If you wanted to stop bullying in your school, what would most likely be an effective
strategy to follow?

Answer:

Change the entire school system to have zero tolerance for bullying

Point: 0.50
Question N44:

In Asch’s line studies, participants who were alone when asked to report the length of
the lines gave the correct answer 98% of the time. However, when they were with the
confederates who sometimes gave an obviously wrong answer, 76% of participants
gave the wrong answer at least once. This suggests that Asch’s studies are an
illustration of

Answer:
public compliance without private acceptance.
Point: 0.50
Question N45:

Altruism is any helping behavior that a person carries out to increase other people’s
warfare.

Answer: False
Point: 0.50
Question N46:

Which of the following statements is correct?

Answer:

Prejudice is an unjustifiable negative attitude toward a group and its members, while
unjustifiable negative behavior towards them is discrimination.

Point: 0.50
Question N47:

prejudice is a(n) _ _ _ , while discrimination is a(n) _ _ _.

Answer:

attitude/behavior

Point: 0.50
Question N48:

If you were a victim in an emergency, what lessons from social psychology would you
apply to get the help you need?

Answer:

Do all of the above.

Point: 0.50
Question N49:

Most wars and terrorist acts are _ _ _ aggression.

Answer:
Hostile
Point: 0.00
Question N50:
What is an implicit prejudice?

Answer:
A person holds a prejudice without being aware of it.
Point: 0.50
Question N51:

Our stress response system is well suited to acute stress but less effective in the face of
chronic stress.

Answer: True
Point: 0.50
Question N52:

Using of antianxiety medication is a problem-directed stress-coping strategy.

Answer: False
Point: 0.50
Question N53:

Selye’s general adaptation syndrome consists of an alarm reaction followed by

Answer:

resistance then exhaustion.

Point: 0.50
Question N54:

In which stage of the general adaptation syndrome are the pituitary and adrenals
stimulated?

Answer:

alarm

Point: 0.50
Question N55:

Which stage of the general adaptation syndrome is associated with the outcome of
disease?

Answer:
alarm
Point: 0.00
Question N56:
Health is the state of absence of illness or injuries.

Answer: False
Point: 0.50
Question N57:

Health constitutes the freedom from disease, pain, or defect.

Answer: False
Point: 0.50
Question N58:

Out of the seven strategies listed below, which ones does Problem-Directed Stress
Coping involve?

1. Fight (destroy, remove, or weaken the threat)

2. Somatically focused activities (use of antianxiety medication, relaxation,


biofeedback)

3. Flight (distance oneself from the threat)

4. Cognitively focused activities (planned distractions, fantasies, thoughts about


oneself)

5. Seek options to fight-or-flight (negotiating, bargaining, compromising)

6. Therapy to adjust conscious or unconscious processes that lead to additional


anxiety

7. Prevent future stress (act to increase one’s resistance or decrease strength of


anticipated stress)

Answer:
1, 3, 5, 7
Point: 0.50
Question N59:

A person is aware their behavior is problematic, but are not committed to do anything
about it. According to the Prochaska’s model, he/she is on the stage of

Answer:
Contemplation
Point: 0.50
Question N60:
A person has to work hard to prevent relapse. According to the Prochaska’s model,
he/she is on the stage of

Answer:
Maintenance
Point: 0.50
Open Questions

Behavior

Point: 0.0

Operationalize variables

Point: 0.0

A questionnaire to know the extent to which different people empathize with different
situations because each person has a different feeling and a different perspective on
everything in life

Point: 0.0

Classical Conditioning

Point: 0.0

observational learning

Point: 0.0

procedural

Point: 0.0

available heuristic

Point: 0.0

Fundamental attribution error

Point: 0.0

normative
Point: 0.0

Behem bgo javab midam

Point: 0.0
Introduction to Psychology Final Exam Test
This test involves questions based on all chapters included in the syllabus (7 fill in the gaps questions, 7
true/false questions, 21 multiple-choice questions).
Total 35 points

Fill in the gaps questions


Fill in the gaps.

(1 point)
_ fundamental attribution error_ _ _ _ _ _ _ _ _ _ _ _ _ _ _ _ _ _ _ _ _ _ _ _ _ _ _ _ _ _ _ _ _ represents the
dual tendency for people to overestimate dispositional factors (blame or credit people) and to
underestimate situational factors (blame or credit the environment) when searching for the cause of
some behavior or outcome.

(1 point)
When people doubt their ability to perform a task, they may engage in _self-handicapping _ _ _ _ _ _ _ _ _
_ _ _ _ _ _ _ _ _ _ _ _ _ _ _ _ _ _ _ _ _ _ _ _ — They deliberately sabotage their performance. The purpose
of this strategy is to have a ready-made excuse for failure that does not imply lack of ability.

(1 point)
George L. Engel’s _ _ _ _ biopsychosocial model of health and illness._ _ _ _ _ _ _ _ _ _ _ _ _ _ _ _ _ _ _ _ _
_ _ _ _ _ _ _ _ _ states that the causes, manifestations, and outcomes of wellness and diseases are
determined by the interactions between biological, psychological, and social factors.

(1 point)
The same situations might cause stress in some people but not in others. According to Lazarus, to
experience some situations as stressful, first step is to evaluate the situations as stressful. _ _ _ _ _ _ _ _ _
_ _ _ _ _ _ _ _ _ _ cognitive appraisal_ _ _ _ _ _ _ _ _ _ _ _ _ is the cognitive interpretation and evaluation
of a stressor.

(1 point)
According to Schachter, the experience of emotion is the joint effect of the two factors: _ _ _ _ _ _ _ _ _ _
_ _ psychological arousal_ _ _ _ _ _ _ _ _ _ _ _ _ _ _ _ _ _ _ _ _ and cognitive appraisal.

(1 point)
_ _ _ _ _ _ _ _ _ _ gender streotypes /gender role_ _ _ _ _ _ _ _ _ _ _ _ _ _ _ _ _ _ _ _ _ _ are beliefs about
attributes and behaviors regarded as appropriate for males and females in a particular culture.

(1 point)
_ _ _ _ _ _ _ _ _ _ _ _ _ _ _ _ crystallized intelligence_ _ _ _ _ _ _ _ _ _ _ _ _ _ _ _ _ _ involves the
knowledge a person has already acquired and the ability to access that knowledge; it is measured by
tests of vocabulary, arithmetic, and general information.

True or False questions


Read each statement and indicate whether they are correct by underlining either True or False.

(1 point)
According to Piaget, assimilation restructures or modifies the child’s existing schemes so that new
information is accounted for more completely.
True or False?

(1 point)
Gardner’s Multiple Intelligences theory suggests that whether people people have mathematical and
linguistic intelligence or not is determined by the heritability estimate.
True or False?

(1 point)
Retrieval cues are the stimuli available as you search for a particular memory.
True or False?

(1 point)
When a behavior is followed by the removal of an aversive stimulus, the event is called negative
reinforcement.
True or False?

(1 point)
Reflex is a learned response elicited by specific stimuli that have biological relevance for an organism.
True or False?

(1 point)
Groups make decisions that are riskier and less conservative than would be made by the members acting
alone.
True or False?

(1 point)
A correlation coefficient of -0.7 indicates a weaker relationship between two variables than a correlation
coefficient of 0.4.
True or False?

Multiple-choice questions
Circle the correct answer.

(1 point)
People tend to take credit for their successes while denying or explaining away responsibility for their
failures. In many situations, people tend to make dispositional attributions for success and situational
attributions for failure. “I got the prize because of my ability”; “I lost the competition because it was
rigged.” These are examples of
A. self-fulfilling prophecy.
B. confirmation bias.
C. self-serving bias.
D. covariation bias.

(1 point)
A teacher believes boys are inherently better at math, so he doesn’t spend equally enough time and
energy to helping female and male students. As a result, his male students get higher scores on final
math exam than girls. This difference between the final performance levels between girls and boys in his
class occurred because of the teacher’s
A. social role.
B. informational influence.
C. self-fulfilling prophecy.
D. self-serving bias.

(1 point)
When a group makes a decision there’s a tendency to filter out undesirable input so that a consensus
may be reached, especially if it is in line with the group leader’s viewpoint. This tendency is referred to as
A. group conformity.
B. group polarization.
C. groupthink.
D. group influence.

(1 point)
When examining a patient’s problem every doctor in a team came to the same conclusion except one of
them but even though he had a different opinion he decided not to voice his different view so that he
wouldn’t feel fool and deviant from others. This is an example of
A. conformity.
B. social cognition.
C. obedience.
D. social role.

(1 point)
According to Rogers, self-actualization is
A. the use of psychological theory to explain an individual’s course through life.
B. a person’s mental model of his or her typical behaviors and unique qualities.
C. a constant striving to realize one’s inherent potential.
D. complete love and acceptance of an individual by another person.

(1 point)
The values for the Independent Construal of the Self in individualistic cultures (such as the USA) are:
A. connectedness with others, group harmony and social relations.
B. personal freedom, self-fulfillment, independence, self-reliance.
C. loyalty to one’s group, duty to one’s family, adherence to social traditions.
D. enhancement of one’s groups’ status and well-being.

(1 point)
Some people believe that events in their life derive primarily from their own actions. For example, when
receiving exam results, they tend to praise or blame themselves and their abilities, not the teacher or the
exam test. These people are said to have
A. internal locus of control.
B. acquired introversion.
C. external locus of control.
D. acquired extroversion.

(1 point)
Self-concept refers to
A. the idea that people observe themselves to figure out the reasons they act as they do.
B. a person’s mental model of his or her typical behaviors and unique qualities.
C. the set of beliefs that one can perform adequately in a particular situation.
D. a generalized evaluative attitude toward the self that influences both moods and behavior.

(1 point)
Which of the following is not a source of self-efficacy judgments?
A. Vicarious experience (your observations of the performance of others).
B. Persuasion (others may convince you person that you can do something, or you may convince
yourself).
C. Monitoring of your emotional arousal as you think about or approach a task (e.g. anxiety suggests
low expectations of efficacy; excitement suggests expectations of success).
D. Self-handicapping (you anticipate failure and you deliberately sabotage your performance so that
you can have a ready-made excuse for failure).

(1 point)
Which of the following is not correct?
A. Wellness is a general condition of soundness and vigor of body and mind; not simply the absence of
illness or injury.
B. Health promotion is the development and implementation of general strategies and specific tactics
to eliminate or reduce the risk that people will become ill.
C. Health is a general condition of soundness and vigor of body and mind; not simply the absence of
illness or injury.
D. Wellness is optimal health, incorporating the ability to function fully and actively over the physical,
intellectual, emotional, spiritual, social, and environmental domains of health..

(1 point)
According to the Transtheoretical Model (Stages of Change), when changing their unhealthy behaviors to
healthy ones, individuals move through six stages of change:
A. precontemplation, contemplation, preparation, action, maintenance, and termination.
B. action, termination, precontemplation, contemplation, preparation and maintenance.
C. preparation, precontemplation, contemplation, action, maintenance, and termination.
D. contemplation, precontemplation, action, maintenance, preparation, and termination.

(1 point)
Failing to adhere to treatment regimens that doctors give to their patients is one of the most serious
problems in health care. Which of the following statements about patient adherence is correct?
A. Generally, patients who perceive greater threat from a disease also show greater likelihood to
adhere to treatment regimen given by a doctor.
B. Patients who face serious diseases that leave them in poor physical health always show higher
levels of adherence than patients who are less debilitated by the same diseases.
C. Physicians should stop trying to understand their patients’ attitudes and modify their behaviors to
match those attitudes.
D. When patients can rely on social support that allows them to accomplish their regimens correctly
they are less likely to adhere to treatments.

(1 point)
Out of the seven strategies listed below, which ones does Problem-Directed Stress Coping involve?
1. Fight (destroy, remove, or weaken the threat)
2. Somatically focused activities (use of antianxiety medication, relaxation, biofeedback)
3. Flight (distance oneself from the threat)
4. Cognitively focused activities (planned distractions, fantasies, thoughts about oneself)
5. Seek options to fight-or-flight (negotiating, bargaining, compromising)
6. Therapy to adjust conscious or unconscious processes that lead to additional anxiety
7. Prevent future stress (act to increase one’s resistance or decrease strength of anticipated stress)

A. 1, 2, 3, 6
B. 2, 4, 3, 7
C. 1, 3, 5, 7
D. 2, 3, 5, 6

(1 point)
Traditionally psychologists assumed that subjective mental experience of an emotion preceded the
bodily expression or action. However, according to the James-Lange theory of emotion, the arousal of
the physical response precedes the appearance of the emotion (perceiving a stimulus causes autonomic
arousal and other bodily actions that lead to the experience of a specific emotion). Which of the
following would not serve as an appropriate example of James’ and Lange’s understanding of how we
experience emotions?

A. I see a wild animal, I feel fear, and I run away.


B. I see a wild animal, I run, and then I experience the emotion of fear.
C. I cry because I feel sorry.
D. I’m afraid because I tremble.

(1 point)
In his experiments Robert Zajonc demonstrated mere exposure effect which describes how sometimes
people have preferences without knowing why. The mere exposure effect means that
A. repeated exposure to a stimulus (such as a song, a person, a painting) increases liking of that
stimulus.
B. repeated exposure to a stimulus (such as a song, a person, a painting) makes a perceiver feel bored
of that stimulus.
C. repeated exposure to a stimulus (such a song, a person, a painting) increases perceivability of the
details that went unnoticed before.
D. repeated exposure to a stimulus (such a song, a person, a painting) decreases willing to pay
attention to more details.

(1 point)
According to the Kohlberg’s theory of moral development, on the last stage of moral reasoning a
person’s intention to behave morally is
A. To avoid pain or not to get caught
B. To gain acceptance and avoid disapproval
C. To be true to universal principles and feel oneself part of a cosmic direction that transcends social
norms
D. To promote the society’s welfare

(1 point)
Children at the preoperational stage (roughly from 2 to 7 years of age) are unable to take the
perspective of another person. For example, a child talking on the phone might pinpoint to a toy that
she/he is talking about with the other person on the phone, she can’t take into account that the person
on the phone can’t see what she/he is pinpointing with his/her fingers to.
In Piaget’s theory, this inability is referred to as
A. centration.
B. conservation.
C. egocentrism.
D. assimilation.

(1 point)
As children’s cognitive abilities develop, they come to understand that other people have cognitive
experiences of the world—and that those cognitive experiences may not be exactly the same. Children
become able to explain and predict other people’s behavior based on an understanding of their mental
states. In other words, children develop
A. expectancy theory.
B. theory of mind.
C. equity theory.
D. theory of individual psychology.

(1 point)
According to Gardner’s Multiple Intelligences theory, some people have great ability to plan and
understand sequences of movements and they’re more likely to become great dancer or athletes. This is
to say that these people have high
A. bodily-kinesthetic intelligence.
B. existential intelligence.
C. spatial intelligence.
D. naturalist intelligence.

(1 point) Which of the following abilities does emotional intelligence involve?


1. The ability to perceive, appraise, and express emotions accurately and appropriately
2. The ability to use emotions to facilitate thinking, to understand and analyze emotions
3. The ability to use emotional knowledge effectively
4. The ability to regulate one’s emotions to promote both emotional and intellectual growth

A. 1, 2, 3
B. 1, 3, 4
C. 1, 2, 3, 4
D. 2, 3, 4

(1 point)
A procedure conducted at the end of an experiment in which the researcher provides the participant
with as much information about the study as possible and makes sure that no participant leaves feeling
confused, upset, or embarrassed is called
A. informed consent.
B. cover story.
C. debriefing.
D. intentional deception.

(1 point)
Which one of the following statements about memory is correct?
A. Semantic memories preserve the specific events that you have personally experienced.
B. Recognition is a method of retrieval in which an individual is required to reproduce the information
to which he/she was previously exposed.
C. When individuals are motivated enough to remember information accurately their memories can
be absolutely accurate.
D. No matter how emotionally important a memory or information can be for an individual, memories
are can always be subject to misinformation effect.
1. Which heuristic can best explain why so many people believe in the pseudoscience of astrology?
Representativeness Heuristic

2.In emergency situations,The larger the number of people present, the higher the probability of helping

True

False

3. in yoga classes, students learn through watching their teacher and classmates. it is an demonstrates that
..observational learning.... ….plays an important role in many sports and activities.

4. Advertiser often exploit classical conditioning principles They try to create associations in people's mind
between their product (for example, sports cars) and passion in this case the element in their
advertisement (e.g. sexually provocative individuals or situations) serve as .....and feelings of sexual arousal
is ..... the product itself (in this case sport cars ) is ..... so that the feelings of arousal will become associated
with it

A. unconditioned stimulus , unconditioned response, conditioned stimulus


B. unconditioned response, conditioned response ,conditioned stimulus,
C. unconditioned stimulus ,neutral stimulus, conditioned response
D. conditioned stimulus , unconditioned response , unconditioned stimulus

5. a stereotype is

a) the cognitive form of a prejudice

b) always inaccurate

c) a negative impression of a group of people

d) a cognitive summary that can be positive or negative

6. The difference between flashbulb and other types of memories is that flashbulb memories are accurate
because processes of reconstruction do not apply to them

True false

7. "Victory finds a hundred fathers but defeat is an orphan." (Count Galeazzo Ciano, The Ciano Diaries,
1938). Match the phrase of a famous author with the corresponding social psychological notion.

A. Self-serving bias
B. The need of causal attribution
C. Social comparison theory
D. Self-
8. ...Standardization / Operationalization... method involves predetermined steps aimed at minimizing
biases and errors to ask questions . Gather and analyze data and answer the question accordingly

9. Seeing someone disobey a questionable order makes people__likely to obey the order themselves.

A. willing
B. prone
C. less likely
D. more likely
10. person behavior is aware their behavior is problematic but not committed to do anything about it
According to the Prochaska's model he/she is on stage of.

A. Precontemplation
B. Contemplation
C. Preparation
D. action
E. Maintenance
11. Which of the following statements about daily hassles is true?

A. Some of the most common hassles involve threats to survival.


B. As daily hassles diminish, people's sense of well-being increases
C. More frequent and intense hassles a are associated with better health
D. The effects of hassles do not accumulate: Many hassles are no worse than a few.

12. sex sells is a common saying in advertising with the knowledge about this form of learning, we can fairly
well explain how sexual images in advertisements can condition your response to a product What is this
form of learning. Classical Conditioning

13. Which one of the following can't be a rule for a consumer of psychological information?

A. Be suspicious of complex answers to simple psychological


B. Avoid the inference that correlation is causation / answer is either C, or D.. we don’t have the other
choices
14. accidental deaths are more likely to be reported by the media, so people find it easier to bring to mind
examples of such deaths than deaths from strokes, so more people end up fearing dying by an accident
than by stroke. This exemplifies

A. The representativeness heuristic


B. Base rate information
C. Base rate information (choice continuation was not visible) I say it’s D probably hehe
15. In contrast to most people, depressed individuals tend to attribute negative outcomes to lasting,
internal causes such as their own traits or lack of ability but attribute positive outcomes to temporary ,
external causes such as good luck or special favors from others. This suggests that ….. Self-serving bias....
may play a role in developing protecting from depression

16. In order to achieve a fuller understanding of the psychological phenomenon in question it is important
to combine multiple psychological perspectives.

True false

17. playing the piano uses ……procedural… memory

18. All of the following are good examples of the self-fulfilling prophecy except one. Which one?

A. A teacher believes that boys are better at math than girls, and boys in his class do better than girls in
math.
B. Ana is worried that her son is not gifted in music, but he does better at his piano lessons than she
expected
C. Ana thinks her dog isn't very good at learning tricks. Her dog knows fewer tricks than most dogs.
D. Ana thinks that members of the Alpha-Beta team members are unfriendly and snobby. Whenever he
meet members of this team, they are unfriendly toward him.
19. Which of the following had the least influence on participants' willingness to keep giving shocks in the
Milgram studies?
A. Loss of personal responsibility
B. Self-justification
C. Informational social influence
D. Participants' aggression
20. The Stanford prison experiment illustrates the power of to influence people's behavior.

A. personality
B. childhood experiences
C. heredity
D. the situation
21. The----method is most likely to use a double-blind design

A. self-report
B. Behavioral
C. experimental
D. A and B
E. B and C
F. A, B and C
22. Students usually remember what was they read in the beginning and the end of a book chapter and
they don't remember the middle sections quite well. This is due to the

A. reconstructive nature of memory.


B. the serial position effect.
C. the semantic encoding principle.
D. the encoding specificity effect
23.Suppose a researcher found a strong negative correlation between college students' GPA (grade point
average) and the amount of TV shows they watch. Which of the following is the best conclusion from this
study?

A. If you know how much TV a student watches, you can predict his or her GPA fairly well.
B. And The higher a student's GPA, the more he or she watches TV.
C. The higher a student's GPA, the less he or she watches TV.
D. Students with a high GPA study more and thus have less time to watch TV.
E. Watching TV a lot interferes with studying.
F. People who are intelligent get higher grades and watch TV less.
24. "Are there alternative ways of interpreting the evidence? This is a question that a critical information
consumer should ask after reading a conclusion."

true false

25. Feeling sad upon hearing a song associated with a long - lost relationship illustrates. ____

A. acquisition
B. sensitization
C. spontaneous recovery
D. experimental extinction
E. respondent conditioning
26. A gymnastics coach encourage her students to improve their routines by watching recordings of
Olympic gold medal gymnastics performances? This decision might be informed by the research on

A. Observational learning
B. Operant learning
C. Pavlovian learning
D. Experiential learning
27. George knows and likes most of his Armenian classmates but privately believes that his Georgian
culture is superior to all others. His belief is evidence of his

A. anti - Armenian prejudice.


B. stereotyping a minority.
C. ethnocentrism
D. out - group homogeneity
28. A person is aware of their behavior is problematic, but are not committed to do anything about it.
According to the Prochaska's model, he / she is on the stage of

A. Precontemplation
B. Contemplation
C. Preparation
D. Action
E. Maintenance
29. In - depth interview is an example of behavioral measure.

true false

30. If a man's conditioned fear of spiders is triggered by the sight of other creatures that look like spiders,
he is demonstrating…stimulus generalization……….

31. According to the Transtheoretical Model (Stages of Change), when changing their unhealthy behaviors
to healthy ones, individuals move through six stages of change:

A. precontemplation, contemplation, preparation, action, maintenance, and termination.


B. action, termination, precontemplation, contemplation, preparation and maintenance.
C. preparation, precontemplation, contemplation action, maintenance, and termination.
D. contemplation, precontemplation action, maintenance preparation, and termmation.

32. Solomon Asch's experiments, about what percent of participants went along with the group's obviously
mistaken judgment at least once?

A. 90 percent
B. 70 percent
C. 50 percent
D. 30 percent

33. Which response shows the stages of the general adaptation syndrome in the correct order?

A. alarm reaction, exhaustion, resistance


B. resistance, alarm reaction, exhaustion
C. exhaustion, resistance, alarm reaction
D. alarm reaction resistance exhaustion
34. The value of elaborative rehearsal over maintenance rehearsal has been cited as evidence for the Levels
of Processing model of memory

true False

35. When playing with a slot machine players have no way of knowing how many times they have to play
before they win. All they know is that eventually, a play will win. This is why slot machines are so effective,
and players are often reluctant to quit. There is always the possibility that the next coin they put in will be
the winning one. What reinforcement contingency do they psychologists use to design slot machines?

A. Variable - Ratio Schedule


B. Fixed - Ratio Schedule
C. Fixed-Interval Schedule
D. Variable interval Schedule
36. to improve your studying and memory, it is advised to use elaborative rehearsal and creating a variety
of memory cues for each concept.

true false

37. Using of antianxiety medication is a problem - directed stress - coping strategy.

true false

38. The predictions implied by a theory are called

A. operational definitions.
B. correlations.
C. hypothesis
D. replications
39. Knowing that two events are negatively correlated provides

A. proof that as one increases, the other decreases,


B. a basis for prediction
C. an explanation of why the events are related.
D. proof that as one increases, the other also increases
E. an indication that an underlying third factor is at work
40. When consumers respond more positively to ground beef described as 75 percent lean than to the
same product labeled "25 percent fat," they have been influenced by

A. belief perseverance
B. belief fixation
C. confirmation bias
D. Framing effect
41. …behavior….. it is an action of human beings and nonhuman animals that serve as the means to adapt
to their environments.

42. What was the independent variable in the Stanford Prison Experiment?

A. prisoner or guard roles


B. IQ level differences of those in the two roles
C. cultural backgrounds of the volunteers
D. all of the above
43. Physicians' diagnoses are influenced by how easily they can bring different diseases to mind. This
means that they seem to use the ……. representativeness heuristic …… when making diagnosis.

44. Which of the following statements about daily hassles is true?

A. -Some of the most common hassles involve threats to survival.


B. -As daily hassles diminish, people's sense of well-being increases
C. -More frequent and intense hassles are associated with better health
D. -The effects of hassles do not accumulate: Many hassles are no worse than a few.
45. When examining a patient’s problem every doctor in a team came to the same conclusion except one of
them even though he had a different opinion he decided not to voice his different view so that he wouldn’t
feel fool and deviant from others. This an example of:

A. -Conformity
B. -Social cognition
C. -Obedience
D. -Social role

46. Joining the end of a ticket line is an example of _ __, whereas forming two lines when a theater
employee requests it is an example of compliance.

_ANSWER----- Conformity

47. In this study, Japanese students refrained themselves from expressing as anger and disgust because in
Japanese culture it’s not that desirable to show negative emotions. This statement is likely made by a
psychologist working in _ _ _ perspective of psychology.

A. the humanistic perspective


B. the cognitive perspective
C. the sociocultural perspective
D. the evolutionary perspective

48. Researchers (Schemer et al; 2008) found that people were more interested in products that had been
embedded in music videos of artists that they liked and less likely to be interested when the products were in
videos featuring artists that they did not like. Which form of learning does finding demonstrate?

ANSWER---- Classical Conditioning

49. Driving to school one wintry day, George narrowly misses a car that slides through a red light. "Slow down! What
a terrible driver." he thinks to himself, Moments later, George himself slips through an intersection and yelps,
Wow These roads are awful. The city snow plows need to get out here. What social psychology principle has
George just demonstrated?

ANSWER -------- Fundamental Attribution Error

50. The actual reason you enjoy eating, exercise, or sex is that during these activities brain releases endorphins
which are hormones that reduce pain and boost pleasure. This statement is likely made by a psychologist
working in ___________ perspective of psychology.

A. the biological perspective


B. the evolutionary perspective
C. the psychodynamic
D. the behaviorist perspective
51. Children tend to imitate what a model does and says, whether the modeled behavior is prosocial (positive,
constructive, and helpful) or antisocial.

-True – Fales

52. When consumers respond more positively to ground beef


A. -belief perseverance.
B. -belief fixation.
C. -confirmation bias
D. -framing effects

53."If men define situations as real, they are real in their consequences.” The knowledge of self-fulfilling
prophecy is relevant for understanding this quote from W.I. Thomas.

-True – False

54. When Gordon Allport said that "defeated intellectually, prejudice lingers emotionally," what did he
mean?
A.
-You can't argue intellectually with a prejudiced person
B. -A prejudiced person cannot intellectually defend his or her attitude
C. -A person's implicit prejudices may decline while explicit prejudices remain
D. -A person's explicit prejudices may decline while implicit prejudices remain

55. People's perceptions, including those of researchers, are subjective and can be influenced by their
motives or expectations. Looking at the same thing, different people might “see” different thing. What
procedure is the solution to minimize this observer bias?

Operationalize variables

56. if a man begins dating a man whom he feels strongly connected to, he may feel that this person is the
one. Since he expects the relationship to last, he treats his partner with love and respect; as a consequence
the relationship ends up fulfilling. This exemplifies self-fulfilling prophecy.

-True – False

57. Which of the following statements is true?

A. -The reasons behind the relationships they reveal cannot be established by correlational studies
alone.
B. -Experiments can reveal cause-and-effect relationships between variables unless the researchers
use randomization.
C. -Psychologists try to obtain representative sample in order to achieve random sampling.
D. -Naturalistic observation entails watching behavior occurring in its natural settings and altering the
behavior being observed.
58. In Watson's work with Little Albert, why was Albert afraid of the Santa Claus mask?

A. He had been classically conditioned with the mask


B. The mask was an unconditional stimulus creating fear
C. He generalized there his learned fear of the rat
D. Instrumental conditioning
59. Which of the following is true about the ethical Conduct of psychological research?

A. -It is good scientific procedure to tell participants about the research hypothesis before they
participate.
B. -If research participants are misled about a study they must be fully debriefed at the end of the study.
C. -Research participants are should not approve any public sharing of the records of their behavior
D. -It is never permissible to use deception.
59. If a study shows positive correlation between holding authoritarian beliefs and infectious diseases, it
means that

A. -people with more authoritarian beliefs tend to have infectious diseases more
B. -infectious diseases are more common in people that don't hold authoritarian beliefs
C. -people that oppose authoritarian beliefs tend to have more infectious diseases
D. -infectious diseases are less common in people that hold authoritarian beliefs
60. Living near a busy train line can be unpleasant. However, over time, most people who live near major
train lines become ……… to the sound of trains going past.

A. -Reinforced
B. -Punished
C. -Sensitized
D. -Habituated
E. -Acquainted
61. Serotonin is one of the four major neurotransmitters that regulate mood." This statement is likely made
by a psychologist working in _ _ _ perspective of psychology

A. -the biological perspective


B. -evolutionary perspective
C. -the psychodynamic perspective
D. -the behaviorist perspective

62. Prejudice is a(n) ….. .. . .. . . , while discrimination is a(n) _ _ _.

A. -behavior/attitude
B. -attitude/behavior
C. -instinct/choice
D. -stimulus/response

63. A person has no intention of changing their behavior in the near future. According to the Prochaska's
model, he/she is on the stage of

A. Precontemplation
B. Contemplation
C. Preparation
D. Action
E. Maintenance
64. A procedure conducted at the end of an experiment in which the researcher provides the participant
with as much information about the study as possible and makes sure that no participant leaves feeling
confused, upset, or embarrassed is called

a. informed consent
b. cover story.
c. debriefing
d. intentional deception.

65. What pattern of remembering emerged in Hermann Ebbinghaus’s Research?

A) Loss occurred at a steady rate.


B) A small initial loss was followed by no further loss.
C) There was no initial loss, but then there was a gradual decline.
D) A sharp initial loss was followed by a gradual decline.

66. The traditional biomedical model of health links among the nervous system, the immune system,
behavioral styles, cognitive processing, and environmental domains of health.

True / False

67. Each psychological research method could be used to explore a specific research question is a powerful
tool in some ways and a weak tool in others.

True / False

68. George and ana meet on a blind date. They get along well until they get into his black cabriolet to go to
a movie. Ana is quiet and reserved for the rest of the evening. It turns out that her brother had recently
been in a serious accident in that same type of car and seeing it brought up those unwanted emotions.
George assumes that Ana has a cold and reserved personality, thereby demonstrating:

a. _Actor-observer effect
b. _Fundamental Attribution error
c. _Perceptual Salience
d. _Downward social comparison
69. The best psychological perspective so far is the sociocultural perspective since its view on human nature
is not limited to ethnocentric research conducted on Western societies only and take cultural diversity into
account

True false

70. The smoker is not yet thinking about quitting. He / she is on the stage of

A. Precontemplation
B. Contemplation
C. Preparation
D. Action
E. Maintenance

71. Which of the following statements is correct?

A. Stereotype is an unjustifiable negative attitude toward a group and its members, while justifiable
negative feeling towards them prejudice
B. Discrimination is an unjustifiable negative attitude toward a group and its members, while
unjustifiable negative behavior towards them is prejudice
C. Prejudice is an unjustifiable negative attitude toward a group and its members, while unjustifiable
negative behavior towards them is discrimination
D. Both stereotype and prejudice stand for unjustifiable negative attitude toward a group and its
members while unjustifiable negative bar towards them is discrimination
72. If you wanted to stop bullying in your school, what would most likely be an effective strategy to follow?

A. Punish the bully publicly.


B. Teach the victim to fight back
C. Reward the bully for not bullying any victims
D. Change the entire school system to have zero tolerance for bullying

73. In August 2014 the bucket challenge exploded on social media, capitalizing on normative conformity to
raise unprecedented amounts of money in the battle against ALS. This exemplifies the implication of
normative social influence

true false

74. When there is no knowing when the waiting will be over people's persistent behavior of re - checking
their e - mails is rewarded with ………. schedules

A. variable ratio
B. fixed ratio
C. Variable interval
D. fixed interval

75. Bedwetting alarms have been used against nocturnal enuresis in adults since the beginning of the 20th
century. The enuresis alarm sensor in the sheets or night clothes becomes wet with urine. This sets off a
signal which wakes the person to void in the toilet. Enuresis alarm Teach "the users to wake up when they
need to urinate. In terms of classical conditioning, alarm sounds is

A. Unconditioned stimulus
B. Unconditioned response
C. Conditioned stimulus
D. Conditioned response

76.Correlation coefficients of .50 and -.50 indicate relationships that are of different strengths.

true false

77. A research article Functional Neuroimaging Study Revealed the impact of Early Parent-Infant
interactions on Children's Brain describes a study. In this study of 125 full -term and premature newborns
at Nationwide Children's Hospital in Columbus, Ohio, found early gentle displays of affection from parents
have lasting effects on how baby brains react to gentle touch the Authors found that the more the parents
hugged their toddlers , the more developed trains they had the children that were hugged more also
showed more positive brain responses than children whose parents hugged them less study suggest that

A. early exposure to hugs stimulates positive brain responses and train development

B. it’s highly recommended for parents to hug their children as Frequently as possible

C. Hugging can have lasting effect on their children

A and B

B and C
All of the above (A,B and C) shows as wrong but the B has space before it

78. hostile attitude toward members of a group, based solely on their membership in that group is referred
to as__Prejudice___

79. Our stress response system is well suited to acute stress but less effective in the face of chronic stress.

true false

80. The smoker is thinking about quitting but has not yet undertaken any behavioral changes. He / she is
on the stage of

A. Precontemplation
B. Contemplation
C. Preparation
D. Action
E. Maintenance

81. The traditional biomedical model of health links among the nervous system, the immune system,
behavioral styles, cognitive processing, and environmental domains of health.

true false

82. The number of short-term illnesses and stress-related psychological disorders was higher than usual in
the months following an earthquake. Such findings suggest that

A. daily hassles have adverse health consequences .

B. expenencing a very stressful event increases a person's vulnerability to illness

C. the amount of stress a person feels is directly related to the number of stressors expenenced

D. small bad events dont stress ones can be toxic

83. in classical conditioning, the consequences of behavior, such as rewards and punishments, influence the
probability that the behavior will occur again

true

false

84. if you believe that people are poor and obese only because they're lazy , you are committing
the…Fundamental Attribution…… error

85. Fantasizing about winning money is a problem-focused way of coping with financial stress.

True

False

86. What is a suppressed prejudice?

a. a person holds a prejudice without being aware of it


b. a person has a tendency to become prejudiced under the right circumstances

c. a person knows he or she is prejudiced but chooses not to express it in public

d. a person reveals a prejudice subtly, by implying a bias rather than saying so outright

87. 9 in 10 college students rate condom as effective if it has supposed "95% success rate" in stopping HIV
virus that causes AIDS -only 4 in 10 think it successful given "5% failure rate" this is ……framing …… effect

88. Questionnaire, questionnaire aims at measuring people's levels of empathy and in fact it measure
peoples capacity to place oneself in another’s position we can say that this questionnaire..... is a
Questionnaire for Cognitive and Affective Empathy (QCAE).............

89. A researcher wants to test the hypothesis that people are less likely to help a person having a seizure in
the street if there are some other people witnessing this situation than if they re alone. In the experiment
he plans, the researcher will vary the number witnessing the situation where an actor (an assistant of the
experimenter) acts as if he's having a seizure .The number of people will be his.

a. placebo control

b. double-blind control

c. independent variable

d. dependent vanable

90. The most successful managers were video-recorded at their workplace to analyze their habits, This
represents an example of

a. Naturalistic observation

b. Correlational research

c. exponential study

d. case-study

91. Researchers are advised to follow the three Rs. Which one doesn't belong to the list?

a. Reduce the number of animals used in research

b. regulate the amount of time animals are exposed to pain and distress

92. Our beliefs about ourselves matter. Do we see ourselves as in control of things? Do we view ourselves
as relatively superior or inferior? Our answers influence our emotions and actions this statement is likely
made by a psychologist working in ………. perspective of psychology

a. the humanistic perspective

b. the cognitive perspective

c. the sociocultural perspective

d. the behaviorst perspective

93. Out of the seven strategies listed below, which ones does Problem-Directed Stress Coping involve?

1. Fight (destroy remove, or weaken the threat)

2. Somatically focused activities ( use of antianxiety medication relaxation biofeedback )


3. Flight (distance oneself from the threat)

4. Cognitively focused activities (planned distractions, fantasies, thoughts about oneself)

5. Seek options to fight -or-flight (negotiating bargaining compromising )

6. Therapy to adjust conscious unconscious processes that lead to additional anxiety

7. Prevent future stress ad to increase one's resistance or decrease strength of anticipated stress)

a. 1,2,3,6

b. 2,4,3,7

c. 1,3,5,7

d. 2,3,5,6

94. when we try to categorize something by judging how similar it is to our conception of the typical
member of the category we use representativeness heuristic

true

false

95. _ _ _ _ _ _ _ _ _ _ _ _ _ _ _ fundamental attribution error _ _ _ _ _ _ _ _ _ _ _ _ _ _ _ _ _ _ _ represents


the dual tendency for people to overestimate dispositional factors (blame or credit people) and to
underestimate situational factors (blame or credit the environment) when searching for the cause of
some behavior or outcome.

96. When people doubt their ability to perform a task, they may engage in _ _ _ _ _ _ _ _ handicapping_ _
_ _ _ _ _ _ _ _ _ _ _ _ _ _ _ _ _ _ _ _ _ _ _ _ _ — They deliberately sabotage their performance. The
purpose of this strategy is to have a ready-made excuse for failure that does not imply lack of ability.

97. George L. Engel’s _ _ _ _ _ _ _ _ _ _ _ biopsychosocial model of health and illness _ _ _ _ _ _ _ _ _ _ _ _ _


_ _ _ _ _ _ _ _ _ _ states that the causes, manifestations, and outcomes of wellness and diseases are
determined by the interactions between biological, psychological, and social factors.

98. The same situations might cause stress in some people but not in others. According to Lazarus, to
experience some situations as stressful, first step is to evaluate the situations as stressful. _ _ _ _ _ _ _ _ _
_ _ _ _ _ cognitive appraisal_ _ _ _ _ _ _ _ _ _ _ _ _ is the cognitive interpretation and evaluation of a
stressor.

99. According to Schachter, the experience of emotion is the joint effect of the two factors: _ _ _ _ _ _ _ _
_ _ _ _ _ _ _ _ _ psychological arousal _ _ _ _ _ _ _ _ _ _ _ _ _ _ _ _ and cognitive appraisal.

100. _ _ _ _ _ _ _ _ _ _ _ _ _ _ _ _ _ gender stereotypes _ _ _ _ _ _ _ _ _ _ _ _ _ _ _ are beliefs about


attributes and behaviors regarded as appropriate for males and females in a particular culture.

101. _ _ _ _ _ _ _ _ _ _ _ _ _ _ _ _ _ _ crystalized intelligence _ _ _ _ _ _ _ _ _ _ _ _ _ _ _ _ involves the


knowledge a person has already acquired and the ability to access that knowledge; it is measured by
tests of vocabulary, arithmetic, and general information.
102. According to Piaget, assimilation restructures or modifies the child’s existing schemes so that new
information is accounted for more completely.
True or False?

103. Gardner’s Multiple Intelligences theory suggests that whether people people have mathematical
and linguistic intelligence or not is determined by the heritability estimate.
True or False?

104. Retrieval cues are the stimuli available as you search for a particular memory.
True or False?

105. When a behavior is followed by the removal of an aversive stimulus, the event is called negative
reinforcement.
True or False?

106. Reflex is a learned response elicited by specific stimuli that have biological relevance for an
organism.
True or False?

107. Groups make decisions that are riskier and less conservative than would be made by the members
acting alone.
True or False?

108. A correlation coefficient of -0.7 indicates a weaker relationship between two variables than a
correlation coefficient of 0.4.
True or False

109. People tend to take credit for their successes while denying or explaining away responsibility for
their failures. In many situations, people tend to make dispositional attributions for success and
situational attributions for failure. “I got the prize because of my ability”; “I lost the competition because
it was rigged.” These are examples of
E. self-fulfilling prophecy.
F. confirmation bias.
G. self-serving bias.
H. covariation bias.

110. A teacher believes boys are inherently better at math, so he doesn’t spend equally enough time and
energy to helping female and male students. As a result, his male students get higher scores on final
math exam than girls. This difference between the final performance levels between girls and boys in his
class occurred because of the teacher’s
E. social role.
F. informational influence.
G. self-fulfilling prophecy.
H. self-serving bias.
111. When a group makes a decision there’s a tendency to filter out undesirable input so that a
consensus may be reached, especially if it is in line with the group leader’s viewpoint. This tendency is
referred to as
E. group conformity.
F. group polarization.
G. groupthink.
H. group influence.

112. According to Rogers, self-actualization is


E. the use of psychological theory to explain an individual’s course through life.
F. a person’s mental model of his or her typical behaviors and unique qualities.
G. a constant striving to realize one’s inherent potential.
H. complete love and acceptance of an individual by another person.

113. The values for the Independent Construal of the Self in individualistic cultures (such as the USA) are:
E. connectedness with others, group harmony and social relations.
F. personal freedom, self-fulfillment, independence, self-reliance.
G. loyalty to one’s group, duty to one’s family, adherence to social traditions.
H. enhancement of one’s groups’ status and well-being.

114. Some people believe that events in their life derive primarily from their own actions. For example,
when receiving exam results, they tend to praise or blame themselves and their abilities, not the teacher
or the exam test. These people are said to have
E. internal locus of control.
F. acquired introversion.
G. external locus of control.
H. acquired extroversion.

115. Self-concept refers to


E. the idea that people observe themselves to figure out the reasons they act as they do.
F. a person’s mental model of his or her typical behaviors and unique qualities.
G. the set of beliefs that one can perform adequately in a particular situation.
H. a generalized evaluative attitude toward the self that influences both moods and behavior.

116. Which of the following is not a source of self-efficacy judgments?


E. Vicarious experience (your observations of the performance of others).
F. Persuasion (others may convince you person that you can do something, or you may convince
yourself).
G. Monitoring of your emotional arousal as you think about or approach a task (e.g. anxiety suggests
low expectations of efficacy; excitement suggests expectations of success).
H. Self-handicapping (you anticipate failure and you deliberately sabotage your performance so that
you can have a ready-made excuse for failure).

117. Which of the following is not correct?


E. Wellness is a general condition of soundness and vigor of body and mind; not simply the absence of
illness or injury.
F. Health promotion is the development and implementation of general strategies and specific tactics
to eliminate or reduce the risk that people will become ill.
G. Health is a general condition of soundness and vigor of body and mind; not simply the absence of
illness or injury.
H. Wellness is optimal health, incorporating the ability to function fully and actively over the physical,
intellectual, emotional, spiritual, social, and environmental domains of health..

118. Failing to adhere to treatment regimens that doctors give to their patients is one of the most serious
problems in health care. Which of the following statements about patient adherence is correct?
E. Generally, patients who perceive greater threat from a disease also show greater likelihood to
adhere to treatment regimen given by a doctor.
F. Patients who face serious diseases that leave them in poor physical health always show higher
levels of adherence than patients who are less debilitated by the same diseases.
G. Physicians should stop trying to understand their patients’ attitudes and modify their behaviors to
match those attitudes.
H. When patients can rely on social support that allows them to accomplish their regimens correctly
they are less likely to adhere to treatments.

119. Traditionally psychologists assumed that subjective mental experience of an emotion preceded the
bodily expression or action. However, according to the James-Lange theory of emotion, the arousal of
the physical response precedes the appearance of the emotion (perceiving a stimulus causes autonomic
arousal and other bodily actions that lead to the experience of a specific emotion). Which of the
following would not serve as an appropriate example of James’ and Lange’s understanding of how we
experience emotions?

E. I see a wild animal, I feel fear, and I run away.


F. I see a wild animal, I run, and then I experience the emotion of fear.
G. I cry because I feel sorry.
H. I’m afraid because I tremble.

120. In his experiments Robert Zajonc demonstrated mere exposure effect which describes how
sometimes people have preferences without knowing why. The mere exposure effect means that
E. repeated exposure to a stimulus (such as a song, a person, a painting) increases liking of that
stimulus.
F. repeated exposure to a stimulus (such as a song, a person, a painting) makes a perceiver feel bored
of that stimulus.
G. repeated exposure to a stimulus (such a song, a person, a painting) increases perceivability of the
details that went unnoticed before.
H. repeated exposure to a stimulus (such a song, a person, a painting) decreases willing to pay
attention to more details.

121. According to the Kohlberg’s theory of moral development, on the last stage of moral reasoning a
person’s intention to behave morally is
E. To avoid pain or not to get caught
F. To gain acceptance and avoid disapproval
G. To be true to universal principles and feel oneself part of a cosmic direction that transcends social
norms
H. To promote the society’s welfare

122. Children at the preoperational stage (roughly from 2 to 7 years of age) are unable to take the
perspective of another person. For example, a child talking on the phone might pinpoint to a toy that
she/he is talking about with the other person on the phone, she can’t take into account that the person
on the phone can’t see what she/he is pinpointing with his/her fingers to.
In Piaget’s theory, this inability is referred to as
E. centration.
F. conservation.
G. egocentrism.
H. assimilation.

123. As children’s cognitive abilities develop, they come to understand that other people have cognitive
experiences of the world—and that those cognitive experiences may not be exactly the same. Children
become able to explain and predict other people’s behavior based on an understanding of their mental
states. In other words, children develop
E. expectancy theory.
F. theory of mind.
G. equity theory.
H. theory of individual psychology

124. According to Gardner’s Multiple Intelligences theory, some people have great ability to plan and
understand sequences of movements and they’re more likely to become great dancer or athletes. This is
to say that these people have high
E. bodily-kinesthetic intelligence.
F. existential intelligence.
G. spatial intelligence.
H. naturalist intelligence.

125. Which of the following abilities does emotional intelligence involve?


5. The ability to perceive, appraise, and express emotions accurately and appropriately
6. The ability to use emotions to facilitate thinking, to understand and analyze emotions
7. The ability to use emotional knowledge effectively
8. The ability to regulate one’s emotions to promote both emotional and intellectual growth

E. 1, 2, 3
F. 1, 3, 4
G. 1, 2, 3, 4
H. 2, 3, 4

126. Which one of the following statements about memory is correct?


E. Semantic memories preserve the specific events that you have personally experienced.
F. Recognition is a method of retrieval in which an individual is required to reproduce the information
to which he/she was previously exposed.
G. When individuals are motivated enough to remember information accurately their memories can
be absolutely accurate.
H. No matter how emotionally important a memory or information can be for an individual, memories
are can always be subject to misinformation effect.

127. Four main goals of psychologists are describing, explaining, _ _ _ _ _ _ _ _ predicting _ _ _ _ _ _ _ _ _


_ _ and controlling behavior.
128. When different people observe the same events, because of their personal motives or expectations,
they don’t always “see” the same thing. What can be a remedy / solution for this challenge to
objectivity? _ _ _ _ _ _ _ _ _ _ _ _ _ _ _ __ standardization and operional definitions _ _ _ _ _ _ _ _ _ _ _ _ _
_______________

129. Parents use various reinforcement schedules to teach children new behavior, strengthen the desired
behavior or try to stop undesired behavior. A continuous schedule is often the best in teaching a new
behavior. Once the response has been learned, _ _ _ _ _ _ _ _ positive reinforcement _ _ _ _ _ _ _ _ _ _ _ _
_ _ _ _ _ _ _ can be used to strengthen the new behavior.

130. After listening to a list of words such as hot, snow, warm, winter, ice, wet, frigid, chilly, heat,
weather, freeze, air, shiver, Arctic, and frost, people often remember having heard the related word cold
even though it was not presented (Gallo, 2006; Roediger & McDermott, 1995).
This happens due to _ _ _ _ _ _ _ _ _ _ _ _ _ _ _ _ encoded to long term memory _ _ _ _ _ _ _ _ _ _ _ _ _ _ _
________

131. Each perspective of psychology (the humanistic, cognitive, sociocultural, evolutionary, humanistic,
behaviorist perspectives) stress the biological, psychological, or social-cultural factors more than
another, each of the perspectives are interesting and valuable by themselves but unfortunately for the
science of psychology, the different perspectives contradict one another, they’re impossible to use in
combination and you have to choose one of them that you agree with more.

True or false?

132. In a Carr and VanDeusen (2004) study, male college students at a large midwestern university
completed an anonymous survey on which they indicated whether they had ever engaged in sexually
coercive behavior as well as the frequency with which they viewed various forms of pornography. The
researchers found a statistically significant correlation, such that the more pornography the students
reported using, the greater the likelihood that they had committed sexual violence (Hald, Malamuth, &
Yuen, 2010).
From this study we can draw the conclusion that using pornography made the male students more likely
to commit sexual violence.
True or false?

133. Every time Ana posts half-naked photographs of herself, she gets lots of attention, likes and
comments on social media. Now uploading half-naked photos and receiving attention became strongly
associated for her, so the number of such posts on her page is steadily increasing. This is an example of
classical conditioning.
True or false? I think it is operant conditioning (positive reinforcement)

134. “Boys are more physically aggressive than girls because a male hormone testosterone is linked with
aggression.” This statement is likely made by a psychologist working in _ _ _ perspective of psychology.

A. the biological perspective


B. the evolutionary perspective
C. the psychodynamic perspective
D. the behaviorist perspective
135. To test the effect of a new drug on depression, a researcher randomly assigns people to control and
experimental groups. Those in the experimental group take a pink pill containing the new medication;
those in the control group take a pink pill that contains no medication. Which statement is true?
A. The medication is the dependent variable.
B. Depression is the independent variable.
C. Participants in the control group take a placebo.
D. Participants in the experimental group take a placebo.

136. A double-blind procedure is often used to prevent researchers’ biases from influencing the outcome
of an experiment. In this procedure,
A. only the participants know whether they are in the control group or the experimental group.
B. experimental and control group members will be carefully matched for age, sex, income, and
education level.
C. neither the participants nor the researchers know who is in the experimental group or control
group.
D. someone separate from the researcher will ask people to volunteer for the experimental group or
the control group.

137. Ana started working for a pet food store. In the beginning she was annoyed by the smell in the store
but after some period of time she got used to it. What is the technical term for “getting used to bad
smell” to the extent that it doesn’t bother Ana, and that she doesn’t react to it anymore?

A. Sensitization.
B. Stimulus discrimination.
C. Habituation.
D. Response acquisition.
138. When a toddler throws a tantrum in the store, parents usually refuse to give in. But once in a while,
when they’re tired or in a hurry, they decide to buy the candy, thinking they will do it just that one time.
This is an example that in daily life, parents sometimes unknowingly _ _ undesired behavior.
A. reinforce
B. extinct
C. punish
D. discriminate
139. Tom waits remembers he was born on 7th December, 1949. This is his
A. semantic memory (facts)
B. procedural memory
C. episodic memory (personal events/experiences)
D. flashbulb memory

140. Suppose you’re on your way home from watching a scary movie about a hitchhiker who was a
murderer when you see someone talking loudly with a friend. Because you saw the movie, you assume
that you are witnessing an argument that will probably end in a fight. This is an example of
A. priming.
B. meta-memory.
C. context effects.
D. spontaneous recovery.
141. a hostile attitude toward members of a group, based solely on their membership in that group is
referred to as --Prejudice-------

142. Couples in happy marriages tend to make --------attributions for each others positive actions (that's
just the way he is )and----------attributions for negative actions (hes just so busy providing for all of us that
he does not have the time)

A. self- serving external


B. self – serving internal
C. internal- external
D. external- internal
143. confirmation bias can lead people to be believe things that are not true because evidence can
generally be found in support of even the most questionable propositions

true

false

144. selye’s general adaptation syndrome consists of a reaction followed by

A. fight or flight
B. resistance then exhaustion
C. challenge then recovery
D. stressful the events
145. to cope with stress,we tend to use -------------- strategies when we feel in control of our world,and to
use ------------strategies when we believe we cannot change a situation

A. emotion -focused ,problem -focused


B. problem- focused ,emotion-focused
C. positive-emotion , negative-emotion
D. positive- emotion ,negative emotion
E. negative emotion ,positive emotion
146. according to the observational learning principle, children do what they see, if a model’s actions and
words are inconsistent, children may imitate the hypocrisy they observe

true

false

147. -(i am good, you are lucky)-the phrase shortly summarizes the idea behind self -serving attributional
bias

true

false

148. if you were a victim in an emergency ,what lessons from social psychology would you apply to get the
help you need

A. ask for it
B. make your request specific
C. engage particular individual observers
D. do all of the above
149. what is stereotype threat?

A. Some members of a majority group feel threatened by minorities


B. some members of a stereotyped group feel anxious when they are made aware of a stereotype
about them
C. some members of a majority group threaten a minority by stereotype them
D. some members of a minority group threaten against the stereotype they find unfair
150. people tend to make self -serving attributions which means they have the tendency to believe that
people behavior corresponds to (matches)their dispositions

true

false

151. in which stage of the general adaption syndrome are the pituitary and adrenals stimulated?

A. Exhaustion
B. reaction
C. alarm
D. resistance
152. in horror movies, sexually arousing images of women are sometimes paired with violence against
women.in terms of classical conditioning principles, after the pairing ,sexual arousal stemming from viewing
an attractive semi-nude woman is

A. an unconditioned stimulus
B. an unconditioned response
C. a conditioned stimulus
D. a conditioned response
153. when experimenting with the influence of music on consumer choice ,75 %of subjects given a pairing
of the colored pen with the attractivemusic chose the pen over a non-exposedpen when given a choice
.and only 30 %of the participants chose the pen paired with unattractive music if they had received a
pairing of this pen with the aversive music

What form of learning manifested itself in this experiment?

A. Experiential learning
B. experimental learning
C. classical conditioning
D. observational learning
154. If potential benefits gained from the study outweigh possible risks associated with the
experimental procedures, researchers are no allowed to conduct the experiment
true / false

155. An article in the Consumers Reports on health newsletter says “Need more motivation to
exercise? Exercise leads better sex. in one study men who exercised were five times as likely to
achieve normal sexual function as a less-active group”. This study shows exercise is one cause that
determines of sexual functioning quality.
True / False
156. If a woman starts dating someone under the assumption that he is really not the one. She
will likely refrain from investing much time or effort into it. This may cause her partner response
accordingly. This instance exemplifies self-fulfilling prophecy.
True / false
157. George constantly misbehaves at preschool even though his teacher scolds him several times
each day. Why does his misbehavior continue?
- Verbal Instructions are not as effective as modeling
- Negative reinforcement is not as effective as positive reinforcement

- Positive reinforcement is not as effective as positive punishment


- positive reinforcement is not as effective as negative reinforcement
- reinforcement is not as effective as punishment

158. In the longest study nearly 80 years from Harvard researchers who have analyzed the rich
data coming from thousands of participants including cast medical records and hundreds of in-
person interviews and questionnaires, found a strong correlation between people’s flourishing
lives and their relationships with family, friends and community. Several studies found that
people’s level of satisfaction with their relationships at age 50 was a better predictor of physical
health than their cholesterol levels were. The data showed that those who kept warm relationships
got to live longer and happier and the loners often died earlier.
The conclusion from this

a) Our relationships and how happy we are in our relationships has a


powerful influence on our health
b) Embracing community makes us happier and helps us live longer
c) Loneliness is just as powerful as smoking or alcoholism and it kills people

d) The key to healthy aging is empathy, attachment and in general


relationships
e) Close relationships more than money or fame are what keep people happy
throughout their lives

157 .The advantage of conducting case studies is that researchers can safely generalize the findings
general population

True false
158. Under a variable interval schedule reinforcement will be delivered after a specified average
number of responses have been made
True false (it should be fixed ration schedule)
159. Descriptive (observational) and correlational studies describe behavior detect relationships
and predict behavior But to explain behaviors psychologists use

A. naturalistic observation
B. experiments

C. survey
D. case study
160. If we encounter a person who appears to be high on drugs, and we make the fundamental
attribution error, we will probably attribute the person's behavior to

A. moral weakness or an addictive personality


B. peer pressure
C. the easy availability of the drugs on the streets

D. society's acceptance of drug use

161. In trying to understand why George was late for an appointment Ana blamed his lack of
conscientiousness and ignored the facts of rush hour traffic and a maior storm that hit town Ana is
guilty
A. benevolent prejudice

B. modern sexism
C. self-serving bias
D. belief perseverance
E. Fundamental Attribution Error

162. Fruit machines in a modern casino pay out on ...... that are determined by the random
generator that controls the play of the machines A casino might advertise that it pays out on every
100 pulls on average which could be true however one player might hit a jackpot after 3 pulls on a
fruit machine whereas another player might not until after 80 pulls the ratio of responses to
reinforcements is variable which helps casinos stay in business

A. Variable ratio
B. Fixed ratio
C. Variable interval

D. Fixed interval

163. In Watson's work with Little Albert, why was Albert afraid of the Santa Claus mask?
A. He had been classically conditioned with the mask

B. The mask was an unconditional stimulus creating fear


C. He generalized there his learned fear of the rat
D. Instrumental conditioning a

164. If it is easier to remember something in the place where you learned it, you have context-
dependent memory.

True / false

165. In order to produce extinction of a classically conditioned behavior,


-an experimenter would reward the behavior.
-pair the behavior with negative reinforcement.

-present the conditioned stimulus in the absence of the unconditioned stimulus.


-model the behavior for the organism.

166. The frustration-aggression hypothesis states that aggression leads to experiencing higher
levels of frustration.

True / false
167. Children's reading skill is measured after taking either a special reading class or a standard
reading class. The dependent variable is

-Reading class
-Reading skills

-Class type
-Student type
-Adults

-Children

168. Health constitutes the freedom from disease, pain, or defect


true

false

169. –Informed Consent---------is an agreement to participate in an experiment, granted in full


awareness of the nature of a research study, which has been explained in advance.

170. Ana took a personality test and results indicated she is an extroverted person. After 2 weeks
she took the test again and now she turned out to be an introvert. The test probably has

- high reliability
-low reliability

-high validity
-low validity

-high discriminative value


-low discriminative value

171. In Asch's line studies, participants who were alone when asked to report the length of the
lines gave the correct answer 98% of the time. However, when they were with the confederates
who sometimes gave an obviously wrong answer, 76% of participants gave the wrong answer at
least once. This suggests that Asch's studies are an illustration of

-public compliance with private acceptance.


-public compliance without private acceptance.

-informational influence.
-private compliance.

172. Health is the state of absence of illness or injuries


True / False

173. Taking an aspirin can relieve headache pain, so people learn to do so through the process of _
-negative reinforcement
-positive punishment
-negative punishment

-positive reinforcement

174. Lesson for parents: --- also works with children. Occasionally giving in to children’s tantrums
for the sake of peace and quiet reinforces the tantrums. This is the very best procedure for making
a behavior persist.
- negative reinforcement
- positive reinforcement

- partial reinforcement
- continuous reinforcement

175. Which one of the following is not an example of normative social influence?

A. Leaving the classroom when a lecturer tells you to because she didn't like what you were doing
B. Getting on your knees when everyone else's doing this, while
attending a religious ritual in a church.

C. Smiling at a lecturer's joke when everyone else does this, even though you don't find the joke
funny.

D. Raising your hand when an informal leader on a demonstration against human rights violation
asks you to indicate if you've been discriminated
against.

A and D

A, B and D

176.In Milgram's original study, about what proportion of the "teacher-subjects" gave the
maximum shock?

about two-thirds
about 50 percent
about 25 percent
nearly all

177. When using availability heuristic, people classify something according to how similar it is to a
typical case.

True / false
178. Health constitutes the freedom from disease, pain, or defect.
True false

166. Normative influence processes refer to wanting to be liked, accepted , and approved of by
others.
True false
167. In Pavlov's early work, bell is to food as
A. unconditioned response is to conditioned response
B. conditioned stimulus is to unconditioned stimulus
C. unconditioned response is to conditioned stimulus
D. conditioned stimulus is to conditioned response

168. According to research in social psychology, why do many people believe that their horoscopes
are accurate descriptions of who they are and what is likely to happen to them?
A. Horoscopes are written in a vague way so that most people view them as representative of
their personalities and past behaviors.
B. Horoscopes trigger automatic decision making.
C. People find it difficult to bring to mind examples that are similar to the horoscope
D. Horoscopes automatically prime people's life goals

169. Jason's parents and older friends all smoke, but they advise him not to. Juan's parents and
friends don't smoke, but they say nothing to deter him from doing so. Will Jason or Juan be more
likely to start smoking?
A. Jason
B. Juan
C. None of them
D. Both equally likely
170. Whether you think you can or you think you can't, you're right.The knowledge of self-
fulfilling prophecy is relevant for understanding his quote from Henry Ford
True false

171. Remembering names is usually harder than remembering faces because names
require ……………. , while faces require ………….
A. context/visual processing
B. visual processing/acoustic processing
C. Recall /recognition
D. recognition /recall

172. Which of the following stressors would be the most likely to cause the immune system to
malfunction and even cause harm?
A. accidentally slipping and falling on an icy surface
B. caring for a dying family member for a prolonged period
C. being rejected by someone you are romantically interested in
D. receiving a bad grade on an important test

173. Americans rated men as more independent than women . But Koreans showed the opposite
pattern: they rated men as more interdependent than women This statement is likely made by a
psychologist working in perspective of psychology
A. the humanistic perspective
B. the cognitive perspective
C. the sociocultural perspective
D. the evolutionary perspective

174. double-blind procedure is often used to prevent researchers' biases from influencing the
outcome of an experiment
True false

175. is an approach to psychology that emphasizes the inherited , adaptive aspects of behavior and
mental processes
A. Behavioral neuroscience
B. Evolutionary perspective
C. Biological approach
D. Humanistic perspective
E. A ,B and C
F. B and C

176. Bedwetting alarms have been used against nocturnal enuresis in adults since the beginning of the 20th
century. The enuresis alarm sensor in the sheets or night clothes becomes wet with urine. This sets off a
signal which wakes the person to void in the toilet. Enuresis alarm Teach "the users to wake up when they
need to urinate. In terms of classical conditioning, urinating in sleep is

A. Positively punished
B. Positively reinforced
C. Negatively reinforced
D. Negatively punished

177. Studying language by listening to people in public places is an example of which research
A. Correlational
B. behaviorist
C. observational
D. experimental

178. Ana, the president of the Psychology Club needs assistance preparing for an upcoming fund-
raising event. She made a plea for help in an e-mail she sent in a mass mailing to all 60 club
members. So far, however, nobody has responded - either to help or not help What might be the
problem ? (What might be causing people's failure to help her?)
179. The validity of survey data depends partly on whether the wording is clear and how questions
are phrased.
True false
180. The validity of survey data depends partly on whether the wording is clear and how questions
are phrased.
A. They must be taken from storage to be used.
B. They must be put in a form the brain can register.
C. They must be transferred from one network to another
D. They must be put in a passive storehouse
181. The fundamental Attribution error is best defined as the tendency to
A. Explain our own and other people's behavior entirely in terms of personality traits , thereby
underestimating the power of social influence
B. explain people's behavior in terms of the social situation, thereby underestimating the power
of personality factors.
C. believe that people's group memberships influence their behavior more than their
personalities.
D. believe that people's personalities influence their behavior more than their group
memberships
182. Usually human behavior is complex; therefore, combined explanations are advised to be
avoided.
True false
Nearly 197 questions total

Introduction to Psychology Midterm Exam Test


This test involves questions based on syllabus sections from 1st to 6th (5 fill in the gaps questions, 5
true/false questions, 15 multiple-choice questions).
Total 25 points
Fill in the gaps questions
Fill in the gaps.

(1 point) Four main goals of psychologists are describing, explaining, _ _ _ _ _ _ predict_ _ _ _ _ _ _ _ _ _ _


_ _ and controlling behavior.

(1 point) When different people observe the same events, because of their personal motives or
expectations, they don’t always “see” the same thing. What can be a remedy / solution for this challenge
to objectivity? _ _ _ _ _ _ _ _ _ _ _ _ _ _ _ _ _ _ _ _ _ _ kants sulution_ _ _ _ _ _ _ _ _ _ _ _ _ _ _ _ _ _ _ _ _ _

(1 point) Parents use various reinforcement schedules to teach children new behavior, strengthen the
desired behavior or try to stop undesired behavior. A continuous schedule is often the best in teaching a
new behavior. Once the response has been learned, _ _ _ _ _ _ _ _ _ _ _ _ _positive reinforcement _ _ _ _
_ _ _ _ _ _ _ _ _ _ can be used to strengthen the new behavior.

(1 point) After listening to a list of words such as hot, snow, warm, winter, ice, wet, frigid, chilly, heat,
weather, freeze, air, shiver, Arctic, and frost, people often remember having heard the related word cold
even though it was not presented (Gallo, 2006; Roediger & McDermott, 1995).
This happens due to _ _ _ _ _ _ _ _ _ _ _ _ _ _ _encoding realated world in long term memory _ _ _ _ _ _ _
________________________

True or False questions


Read each statement and indicate whether they are correct by underlining either True or False

(1 point) Each perspective of psychology (the humanistic, cognitive, sociocultural, evolutionary,


humanistic, behaviorist perspectives) stress the biological, psychological, or social-cultural factors more
than another, each of the perspectives are interesting and valuable by themselves but unfortunately for
the science of psychology, the different perspectives contradict one another, they’re impossible to use in
combination and you have to choose one of them that you agree with more.

True or false? false

(1 point) In a Carr and VanDeusen (2004) study, male college students at a large midwestern university
completed an anonymous survey on which they indicated whether they had ever engaged in sexually
coercive behavior as well as the frequency with which they viewed various forms of pornography. The
researchers found a statistically significant correlation, such that the more pornography the students
reported using, the greater the likelihood that they had committed sexual violence (Hald, Malamuth, &
Yuen, 2010).
From this study we can draw the conclusion that using pornography made the male students more likely
to commit sexual violence.
True or false? true
(1 point) Every time Ana posts half-naked photographs of herself, she gets lots of attention, likes and
comments on social media. Now uploading half-naked photos and receiving attention became strongly
associated for her, so the number of such posts on her page is steadily increasing. This is an example of
classical conditioning.
True or false? false

(1 point) The difference between flashbulb and other types of memories is that flashbulb memories are
accurate because processes of reconstruction do not apply to them.
True or false? False

Multiple-choice questions
Circle the correct answer.

(1 point) “Boys are more physically aggressive than girls because a male hormone testosterone is linked
with aggression.” This statement is likely made by a psychologist working in _ _ _ perspective of
psychology.

E. the biological perspective


F. the evolutionary perspective
G. the psychodynamic perspective
H. the behaviorist perspective
(1 point) “In this study, Japanese students refrained themselves from expressing as anger and disgust
because in Japanese culture it’s not that desirable to show negative emotions.” .” This statement is likely
made by a psychologist working in _ _ _ perspective of psychology.

A. the humanistic perspective


B. the cognitive perspective
C. the sociocultural perspective
D. the evolutionary perspective
(1 point) “Our beliefs about ourselves matter. Do we see ourselves as in control of things? Do we view
ourselves as relatively superior or inferior? Our answers influence our emotions and actions.” This
statement is likely made by a psychologist working in _ _ _ perspective of psychology.

A. the humanistic perspective


B. the cognitive perspective
C. the sociocultural perspective
D. the behaviorist perspective

(1 point) To test the effect of a new drug on depression, a researcher randomly assigns people to control
and experimental groups. Those in the experimental group take a pink pill containing the new
medication; those in the control group take a pink pill that contains no medication. Which statement is
true?
E. The medication is the dependent variable.
F. Depression is the independent variable.
G. Participants in the control group take a placebo.
H. Participants in the experimental group take a placebo.
(1 point) A researcher wants to test the hypothesis that people are less likely to help a person having a
seizure in the street if there are some other people witnessing this situation than if they’re alone. In the
experiment he plans, the researcher will vary the number of people witnessing the situation where an
actor (an assistant of the experimenter) acts as if he’s having a seizure. The number of people will be his

A. placebo control.
B. double-blind control.
C. independent variable.
D. dependent variable.

(1 point) A double-blind procedure is often used to prevent researchers’ biases from influencing the
outcome of an experiment. In this procedure,
E. only the participants know whether they are in the control group or the experimental group.
F. experimental and control group members will be carefully matched for age, sex, income, and
education level.
G. neither the participants nor the researchers know who is in the experimental group or control
group.
H. someone separate from the researcher will ask people to volunteer for the experimental group or
the control group.

(1 point) Ana started working for a pet food store. In the beginning she was annoyed by the smell in the
store but after some period of time she got used to it. What is the technical term for “getting used to bad
smell” to the extent that it doesn’t bother Ana, and that she doesn’t react to it anymore?

E. Sensitization.
F. Stimulus discrimination.
G. Habituation.
H. Response acquisition.
(1 point) When playing with a slot machine players have no way of knowing how many times they have
to play before they win. All they know is that eventually, a play will win. This is why slot machines are so
effective, and players are often reluctant to quit. There is always the possibility that the next coin they
put in will be the winning one.
What reinforcement contingency do they psychologists use to design slot machines?
A. Variable-Ratio Schedule
B. Fixed-Ratio Schedule
C. Fixed-Interval Schedule
D. Intermittent Reinforcement Schedule

(1 point) When a toddler throws a tantrum in the store, parents usually refuse to give in. But once in a
while, when they’re tired or in a hurry, they decide to buy the candy, thinking they will do it just that one
time. This is an example that in daily life, parents sometimes unknowingly _ _ undesired behavior.
E. reinforce
F. extinct
G. punish
H. discriminate
(1 point) Students usually remember what was they read in the beginning and the end of a book chapter
and they don’t remember the middle sections quite well. This is due to

A. reconstructive nature of memory.


B. the serial position effect.
C. semantic encoding principle.
D. encoding specificity effect.

(1 point)Tom waits remembers he was born on 7th December, 1949. This is his
E. semantic memory
F. procedural memory
G. episodic memory
H. flashbulb memory

(1 point) Suppose you’re on your way home from watching a scary movie about a hitchhiker who was a
murderer when you see someone talking loudly with a friend. Because you saw the movie, you assume
that you are witnessing an argument that will probably end in a fight. This is an example of
E. priming.
F. meta-memory.
G. context effects.
H. spontaneous recovery.

Version I :
1. Which one of the following was a critical figure in the evolution of modern psychology?
a) John B.Watson
b) WilhelmWundt
c) WilliamJames
d) SigmundFreud
2. What is functionalism?
a) The perspective on mind and behavior that focuses on the examination of their
functions in an organism’s interactions with the environment.
b) The view that all human mental experience can be understood as a combination of
simple elements or events.
c) The perspective according to which complex phenomena should be explained by the
simplest underlying principles.
d) Theory that all behavior is a result of past learning.
3. During puberty, researchers have documented particularly important changes in
which of the following?
a) Limbic system and Frontal lobes
b) Frontal lobes and Hippocampus
c) Hippocampus and Limbic system
d) None of above
4. A behavioral psychologist would probably say that:
a) Cognition is most important when analyzing behavior.
b) We act based on rewards and punishments received.
c) The unconscious stimulates our behavior.
d) Introspection is a useful way to study mental processes.
5. A sample of 40 university students are given either vitamin B, or a placebo. Before
starting the experiment, and then after taking the pills for two weeks, the students fill out a
survey about their energy levels. The dependent variable is
a) The student’s energy level.
b) The two weeks of the experiment.
c) The vitamin B
d) Placebo pill
6. Which one of the following is a multidisciplinary field that attempts to understand
the brain processes that underlie higher cognitive functions in humans?
a) Cognitive neuroscience
b) Behavioral neuroscience
c) Biological perspective
d) Evolutionary perspective
7. are rules for participation in conversations; social conventions
for communicating, sequencing sentences, and responding appropriately to
others.
a) Phonology
b) Phonetics
c) Pragmatics
d) None of above
8. Which of the following is a research design that uses each participant as his or her own
control? For example, the behavior of an experimental participant before receiving
treatment might be compared to his or her behavior after receiving treatment.
a) Within-subjects design
b) Between-subjects design
c) Placebo control
d) A random sapling
9. How can we determine if a test has good validity?
a) It measures what it is supposed to measure
b) It produces the same result no matter which version of the test is used
c) It produces the same result when it is given at different times to the same group of people
d) All of the questions on it can be answered accurately by the subject
10. What is dependent variable?
a) In an experimental setting, a variable that the researcher measures to assess the
impact of a variation in the other variable
b) In an experimental setting, a variable that the researcher manipulates with the
expectation of having an impact on values of the other variable
c) The factor manipulated by there searcher
d) A variable that represents a quantity that is being manipulated in an experiment
11. What is habituation ?
a) A frequently repeated stimulus.
b) An increase in behavioral response when a stimulus is presented repeatedly.
c) An increase in brain activity when stimulus is presented
d) A decrease in a behavioral response when a stimulus is presented repeatedly.
12. What is classical conditioning?
a) A method of learning that occurs through rewards and punishments for behavior.
b) A learning procedure in which a biologically potent stimulus (e.g. food) is paired with a
previously neutral stimulus (e.g. a bell).
c) The weakening of a conditioned association in the absence of a reinforcer or
unconditioned stimulus .
d) A form of learning in which animals acquire a response that will allow them to escape
from an aversive stimulus .
13. What is positive reinforcement ?
a) A part of an experiment where participant is presented with visual stimuli
b) A behavior is followed by the removal of a negative stimulus, making it more likely
that the behavior will occur again in the future.
c) A behavior is followed by the presentation of a reinforcing stimulus, increasing the
probability of that the behavior occurring again in the future.
d) Forcing a participant to engage a certain behavior
14. Which one of the following is an example of negative reinforcement?
a) A teacher allows students to not take final test if they have perfect attendance. I.e.
takes away negative experience to increase a certain behavior.
b) A teacher compliments students when they answer correctly.
c) At a gym, customers receive a discount if they keep up with their workout routine.
d) A student receives a praise after a musical performance
15. What is a primary reinforcer?
a) Any stimulus, that has no innate biological value.
b) A negative stimulus, that is taken away in order to reinforce a certain behavior
c) Non-biological reinforcer, such as appraisal
d) Biologically determined reinforcer, such as food and water.
16. According to Piaget, which of the two basic processes work in tandem to
achieve cognitive growth?
a) Assimilation and accommodation
b) Object permanence and Scheme
c) Scheme and accommodation
d) None of above
17. As a result of Pavlov’s classical conditioning, what happened when dog heard a ring of
a bell?
a) Dog became aggressive
b) Dog started salivating
c) Dog had no reaction
d) Dog ran away
18. What is behavioral data?
a) A method of self-observation in which participants report their thoughts andfeelings
b) A set of assumptions
c) Scientific predictions
d) Reports of observations about the behavior of organisms and the condition under
which the behavior occurs
19. Which one of the following would be considered the best definition of psychology?
a) The study of individual’s mental processes, such as thinking, planning, and reasoning
b) The scientific study of human behavior
c) The scientific study of the behavior of individuals and their mental processes
d) The study of people’s behavior patterns
20. What is internalization according to Vygotsky?
a) The process through which children absorb knowledge from the social context.
b) An unconscious defense mechanism by which an individual "projects" his or her own
internal characteristics onto the outside world
c) The ability to explain and predict other people’s behavior
d) Being upset easily
21. Which principle suggests that that children must keep track of the relationship
between the order in which words appear and the meanings they express?
a) Operating principle
b) Operant theory
c) Child-directed stimuli
d) Speech principle
22. What prepares the body for emotional responses through the action of both
its sympathetic and parasympathetic divisions?
a) Somatic nervous system
b) Autonomic nervous system(ANS)
c) Medulla
d) Spinal cord
23. Which theory states that an emotional stimulus produces two co-occurring
reactions— arousal and experience of emotion—that do not cause each other.
a) James–Lange theory of emotion
b) Attachment theory
c) Appraisal theory
d) Cannon–Bard theory of emotion
24. What has neuroanatomy research particularly been focused on, as a part of the
limbic system that acts as a gateway for emotion and as a filter for memory?
a) Amygdala
b) Limbic system
c) Hypothalamus
d) Pons
25. Which one of the following is a sequence of internal activities triggered when an
organism is faced with a threat; prepares the body for combat and struggle or for
running away to safety; recent evidence suggests that the response is characteristic
only of males.
a) Withdrawal response
b) Approach behavior
c) Fight-or-flight response
d) None of above
26. Which one of the following best describes psychosomatic disorder?
a) Mental disorder mainly characterized by unclear or shifting self-image
b) Mental disorder attributed to acute stress
c) Physical disorder aggravated by or primarily attributable to prolonged emotional
stress or other psychological causes.
d) None of above
27. What are the big five personality traits?
a) Extraversion, agreeableness, humility, proclivity to anger, conscientiousness
b) Extraversion, agreeableness, conscientiousness, neuroticism, openness to experience
c) Introversion, agreeableness, humility, openness to experience, honesty
d) Introversion, proclivity to anger, humility, agree ableness, honesty

28. Individuals who score high on neuroticism are most likely to be:
a) Anxious, unstable, and temperamenta
b) Careless, frivolous, and irresponsible
c) Quiet , reserved, and shy
d) Talkative, energetic, and assertive
29. What is consistency paradox?
a) The observation that certain psychological theories are not consistent across different
cultures
b) The observation that human emotions have explainable pattern
c) The observation that humans consistently behave in a similar pattern in familiar
situations.
d) The observation that personality ratings across time and among different observers are
consistent while behavior ratings across situations are not consistent
30. Which theory of personality shares the assumption that personality is shaped by
and behavior is motivated by inner forces?
a) Psychodynamic personality theory
b) Social-cognitive personality theory
c) Trait personality theory
d) None of above
31. Which of the following best describes cognitive development?
a) The development of processes of knowing, including imagining, perceiving, reasoning,
and problem solving.
b) The stage where children enter puberty
c) The ability of a young child at the preoperational stage to take the perspective of another
person.
d) None of above
32. Which of the following is the field of psychology that specializes in mental testing in any
of its facets, including personality assessment, intelligence evaluation, and aptitude
measurement.
a) Cognitive psychology
b) Psychometrics
c) Organizational psychology
d) None of above
33. What is the factor of general intelligence underlying all intelligent
performance, According to Spearman?
a) r
b) s
c) N
d) g
34. which form of intelligence is the ability to understand other people and social
interactions?
a) Interpersonal
b) Intrapersonal
c) Spatial
d) Logical-mathematical
35. Which type of psychologist is least likely to focus on genetic aspects of human
psychology?
a) Industrial–organizational psychologists
b) Developmental psychologists
c) Personality psychologists
d) Biological psychologists
36. Georgian University is conducting a study where one group of students are instructed to
get only 8 hours of sleep and another group of students are advised to get 5 hours of sleep.
What type of experimental design is the university using to test how sleeping affects
students’ well-being?
a) Within-subjects design
b) Between-subjects design
c) Repeated measures design
d) None of above
37. Emily is a psychologist who is interested in the effects of noise level on concentration.
She believes that the noisier a room is, the less people will be able to concentrate. To test
her hypothesis, Emily gathers a bunch of volunteers and gives them a book to read in a
noisy room. Afterwards, she tests their memory of the reading the book. Then, she puts all
of the volunteers into a room that's quiet and has them read another book. Finally, she
tests them on the book they read in the quiet room. What type of experimental design is
Emily using?
a) Between-subjects design
b) Within-subjects design
c) Observational study
d) None of above
38. Research supports all of the following conclusions about the perception of faces
EXCEPT which conclusion?
a) infants (6 month and younger) prefer top-heavy visual displays
b) infants (6 months and younger) have better memory for human than monkeyfaces
c) children have better memory for faces from racial groups they see growing up than
faces from other racial groups
d) faces are perceived and remembered configurally rather than part-by-part
39. reliability refers to the:
a) Validity of data
b) Values of data
c) Levels of data
d) Consistency of data
40. Infants prefer visual displays, and at 6monthsrecognize
monkeyfaces human faces.
a) bottom-heavy; worse than
b) top-heavy; as well as
c) top-heavy; worse than
d) bottom-heavy; as well as
41. Which one of the following is the latest version of DSM?
a) V
b) IV
c) VI
d) III
42. What are side effects of serotonin reuptake inhibitors(SSRIs)?
a) Nausea
b) Dizziness
c) Lethargy
d) all of above
43. What class of medication is mostly prescribed for people who suffer from severe
anxiety?
a) SNRIs
b) SSRIs
c) Benzodiazepines
d) None of above
44. What physical symptoms are stress related to?
a) Increased heartrate
b) Tightened muscles
c) Increased blood pressure
d) All of above
45. Which hormone is known as “stress hormone”?
a) Cortisol
b) Serotonin
c) Dopamine
d) None of above
46. How would Freud describeid?
a) The storehouse of the fundamental drives, governmed by pleasure principle
b) The storehouse of individual values
c) “Ideal self” of personality
d) None of above
47. What is the function of ego?
a) To mediate between the unrealistic id and the external real world
b) To incorporates the values and morals of society which are learned from one's parents
and others
c) To engage in primary process thinking, which is primitive, illogical, irrational ,and
fantasy oriented.
d) None of above
48. What is the healthiest defense mechanism?
a) displacement
b) Isolation
c) projection
d) sublimation
49. People who use isolation as their defense mechanism usually:
a) Cut off emotional charge from hurtful situations or separate incompatible attitudes into
logic-tight compartments (holding conflicting attitudes that are never thought of
simultaneously or in relation to each other)
b) Placie blame for one’s difficulties on others or attribute one’s own “forbidden” desires to
others
c) Prevente dangerous desires from being expressed by endorsing opposing attitudes and
types of behavior and using them as “barriers”
d) None of above
50. People who protect self rom unpleasant reality by refusing to perceive it have which
of the following defense mechanisms?
a) Denial of reality
b) Fantasy
c) Sublimation
d) Reaction formation
………………………………………………………………………………………………….
Version II :
1. When a psychologist is using the method of naturalistic observation to find out what
type of public places attract humans the most, psychologist would:

a) carefully design controlled situations in which to observe behavior.


b) rely on observations of subjects' responses to questionnaires.
c) observe behavior as it happens outside the laboratory or clinic.
d) make records of the behavior of clients treated in therapy.

2. The study we use to determine the degree of relationship between (two) to variables is
called:

a) Naturalistic observation.
b) the correlational method.
c) a controlled1 experiment.
d) the survey method.

3. Explanations for aggressive behavior from this perspective include genetic


predisposition, high testosterone level and frontal lobe damage.
a) Cognitive
b) Social
c) Biological
d) Cross-cultural
4. Which perspective attempts to study the structure of conscious experience through
the method of introspection.

a) Functionalism
b) Behaviorism
c) Gestalt
d) Structuralism
5. psychologists note that only the fittest organisms reach maturity and reproduce
,and transmit genes to future generations

a) Social cognitive
b) Behavioral
c) Evolutionary
d) Psychodynamic
6. “The Strange Situation” test, where child is left alone in the room, is used to assess
what aspect of psychological development?
a) Attachment
b) Meta-cognitive awareness
c) Moral reasoning
d) Psychosocial stages
7. Referring to previous study described in question 10, what method was used to
measure effects of vitamin B on student’s energy levels?
a) Between-subjects
b) Within subjects
c) Observational study
d) All of above
8. Who is known as the father of classical conditioning?

a) Ivan Pavlov
b) B.F.Skinner
c) Sigmund Freud
d) Jean-Paul Sartre
9. According to Freud, which inner force contains the libido?
a) Superego
b) Ego
c) Id
d) None of above
10. Which of these might be prescribed to a patient who has been diagnosed with
major depressive disorder?
a) Norepinephrine
b) DSM
c) Acetylcholine
d) SSRIs
11. What is the name of the book that holds the diagnosis criteria and overview of
all documented psychological disorders?
a) Diagnostic and Statistical Manual of Mental Disorders
b) APA Manual of Psychological Disorders
c) Overview of Criteria of Mental Disorders
d) None of Above
12. What experimental technique in which biased expectations of experimenters are
eliminated by keeping both participants and experimental assistants unaware of
which participants have received which treatment.
a) Double-blind control
b) Random technique
c) Placebo control
d) None of above
13. Which one of the following is a definition of representative sample?
a) A subset of a population that closely matches the overall characteristics of the
population with respect to the distribution of males and females, racial and ethnic
groups, and soon
b) The entire set of individuals to which generalizations will be made based on an
experimental sample.
c) A subset of a population selected as participants in an experiment.
d) All of above
14. According to Piaget, what is the name of the process whereby new cognitive elements
are fitted in with old elements or modified to fit more easily; this process works in tandem
with accommodation
a) Scheme
b) Assimilation
c) Perception
d) None of above
15. What ages does formal operation stage cover?
a) From 1 to3
b) From 4 to7
c) From age 11on.
d) None of above
16. What is theory of mind?
a) The ability to explain and predict other people’s behavior based on an understanding of
their mental states.
b) The ability to clearly perceive other’s speech
c) The ability to express oneself
d) None of above
17. Who was one of the first psychologists to emphasize the pervasive role of
immediate, unlearned affective (emotional)reactions?
a) Paul Ekman
b) David Neal
c) William James
d) Silvan Tomkins
18. Which one of the following is a peripheral-feedback theory of emotion stating that
an eliciting stimulus triggers a behavioral response that sends different sensory and
motor feedback to the brain and creates the feeling of a specific emotion.
a) James–Lange theory of emotion
b) Theory of emotional response
c) Cannon–Bard theory of emotion
d) None of above
19. What is subjective well-being?
a) The processes through which people change the intensity and duration of the emotions
they experience
b) Individuals’ overall evaluation of life satisfaction and happiness.
c) Both A and B
d) None of above
20. What is stressor?
a) An internal or external event or stimulus that induces stress.
b) An internal or external event that decreases stress
c) An emotional state
d) Nona of above
21. Which one of the following is a response to stressors that is hypothesized to be typical
for females; stressors prompt females to protect their offspring and join social groups to
reduce vulnerability?
a) Fight-or-Flight
b) Tend-and-befriend response
c) Crying
d) None of above
22. How many stages does general adaptation syndrome (GAS)involve?
a) 1
b) 2
c) 3
d) 4
23. Before you participate in an experiment, the researcher should provide you with
information about procedures, potential risks, and expected benefits. This process is
called
a) A risk/gain assessment.
b) Informed debriefing.
c) Informed consent.
d) Operational definitions
24. Freud believed that every young boy has an innate impulse to view his father as
asexual rival for his mother’s attentions. What was the name of this complex?
a) Zeus
b) Oedipus
c) Inadequacy complex
d) None of above

25. When does Freudian slip occur?


a) When an unconscious desire is betrayed by your speech or behavior
b) When your unconscious desires are repressed by superego
c) When you’re unable to progress normally to the next stage of development
d) None of above
26. The perspective draws on the ways in which human mental abilities
serve adaptive purposes.
a) cognitive
b) humanistic
c) evolutionary
d) sociocultural
27. Which one of the following defense mechanisms involve retreating to earlier
developmental levels involving more childish responses and usually a lower level of
aspiration
a) Regression
b) Reaction formation
c) Sublimation
d) None of above
28. From the following answers, what would be the best definition ego defense mechanism?
a) Mental strategy (conscious or unconscious) used by the ego to defend itself against
conflicts experienced in the normal course of life.
b) Physical strategy used by the ego to defend itself against life’s stressors
c) Mental strategy used by ego to withdraw from situations when feeling threatened
d) None of above
29. From psychodynamic perspective, what is an intense emotional response triggered when
a repressed conflict is about to emerge into consciousness?
a) Joy
b) Anxiety
c) Regression
d) None of above
30. What is retrospection?
a) The process of thinking about past events
b) The process of thinking about future events
c) The process of uncovering unconscious thoughts
d) None of above
31. What is androcentric?
a) Female-centered
b) Male-centered
c) Gender neutral
d) None of above
………………………………………………………………………………………………………ve
rsion III :
1. Which one of the following psychologists focused on the way in which the mind
understands many experiences as gestalts—organized wholes—rather than as the sums of
simple parts?

a) Max Wertheimer
b) Freud
c) B.F. Skinner
d) Edward Titchener

2. A school of psychology that maintains that psychological phenomena can be understood


only when viewed as organized, structured wholes, not when broken down into primitive
perceptual elements.

a) CBT
b) Gestalt psychology
c) Structuralism
d) None of above

3. Which one of the following is psychodynamic perspective?

a) A psychological model that emphasizes an individual’s phenomenal world and inherent capacity
for making rational choices and developing to maximum potential.
b) A psychological model in which behavior is explained in terms of past experiences and
motivational forces; actions are viewed as stemming from inherited instincts, biological drives,
and attempts to resolve conflicts between personal needs and social requirements.
c) A multidisciplinary field that attempts to understand the brain processes that underlie behavior.
d) None of above
4. When explaining nature of aggression and violence psychologists who follow humanistic
perspective would look for:
a) Personal values and social conditions that foster self-limiting, aggressive perspectives instead of
growth-enhancing, shared experiences.
b) Consider what conditions would have made aggression an adaptive behavior for early humans.
c) Study the impact of violence in films and videos, including pornographic violence, on attitudes
toward gun control, rape, and war.
d) None of above

5. Who pioneered the use of electrical stimulation to probe structures deep in the brain.
a) Paul Broca
b) Walter Hess
c) Giacomo Rizzolatti
d) None of above
6. Which brain structure serves as a relay station between the brain and the endocrine
system?
a) the hippocampus
b) the hypothalamus
c) the pons
d) the amygdala
7. Genetic disorder, known as phenylketonuria is associated to have:

a) A potential negative impact on IQ


b) A potential positive impact on visual perception
c) Positive impact on development of bipolar disorder
d) None of above
8. What is crystalized intelligence?

a) the factor of general intelligence underlying all intelligent performance.


b) The facet of intelligence involving the knowledge a person has already acquired and the ability
to access that knowledge; measures by vocabulary, arithmetic, and general information tests.
c) The aspect of intelligence that involves the ability to see complex relationships and solve
problems.
d) None of above

9. Which one of the following is associated with being at risk for confirming a negative
stereotype of one’s group.

a) Stereotype threat
b) Negative stereotype theory
c) General anxiety disorder
d) None of above
10. Which one of the following is the research effort designed to describe what is
characteristic of a specific age or developmental stage.

a) Normative investigation
b) Experimental observation
c) Correlational design
d) None of above
11. What are the first two weeks after formation of the zygote are known as?

a) Embryonic stage
b) The fetal stage
c) Germinal stage
d) None of above
12. Newborns may find it relatively easy to learn to recognize their mothers’ faces because

a) they have deeper emotional connection with their mother


b) they are associated with voices with which they are already familiar
c) they are associated with a scent with which they are already familiar
d) None of above

13. ______ is recognition that objects exist independently of an individual’s action or


awareness; an important cognitive acquisition of infancy;

a) Object permanence
b) Assimilation
c) Scheme
d) None of above
14. From what age does the concrete operations stage begin?

a) 4
b) 5
c) 10
d) 7

15. What is rich factual knowledge?

a) Knowledge about the contexts of life and their temporal (developmental) relationships
b) General and specific knowledge about the conditions of life and its variations
c) Knowledge about the relative indeterminacy and unpredictability of life and ways to manage it
d) None of above
16. What’s is concept of a grammatical error, usually appearing during early language
development, in which rules of the language are applied too widely, resulting in incorrect
linguistic forms.
a) Overregularization

b) Pragmatics theory

c) Language-making capacity theory

d) None of above

17. According to Erickson, what “crisis” children from birth to 1 year experience?

a) Autonomy vs. self-doubt


b) Trust vs. mistrust
c) Secure & insecure attachment
d) None of above
18. According to Erickson, what “crisis” do people in their 30s and 40s usually experience? a)
Intimacy vs. Isolation
b) Ego integrity vs. Despair
c) Generativity vs. Stagnation
d) None of above

19. What did John Bowlby (1973), an influential theorist suggested?


a) Infants and adults are biologically predisposed to form attachments.
b) Children usually form anxious attachment
c) Children’s attachment style is easily changed
d) None of above
…………………………………………………………………………………………………………

Version I :

1. Hypothesis is testable explanation of the relationship between two or more variables.

2. Debriefing is the process of reviewing articles that are submitted to scientific journals.
3. Fixation is a state in which a person remains attached to objects or activities more
appropriate for an earlier stage of psychosexual development.
4. Conscious is the domain of the psyche that stores repressed urges and primitive impulses.
5. Libido The psychic energy that drives individuals toward sensual pleasures of all types,
especially sexual ones.
6. Superego is the aspect of personality involved in self-preservation activities and in
directing instinctual drives and urges into appropriate channels.
7. Idis the primitive, unconscious part of the personality that represents the internalization of
society’s values, standards, and morals
8. According to Freud, anxiety is an intense emotional response caused by the preconscious
recognition that a repressed conflict is about to emerge into consciousness.
9. Collective unconscious is the part of an individual’s unconscious that is inherited,
evolutionarily developed, and common to all members of the species.
10. Self-concept is a concept in personality psychology referring to a person’s constant striving to
realize his or her potential and to develop inherent talents and capabilities
11. Repression is the basic defense mechanism by which painful or guilt producing thoughts,
feelings, or memories are excluded from conscious awareness.
12. Scheme is Piaget’s term for a cognitive structure that develops as infants and young
children learn to interpret the world and adapt to their environment.
13. Those who suffer from depression experience persistent feelings of sadness and hopelessness
and lose interest in activities they once enjoyed, though they physical symptoms are never
presented.
14. Individual psychotherapy is the single most frequently used activity of clinicians and still
occupies the largest percentage of a clinician’s time.
15. Anxiet
y and depression are proven to be negatively correlated with each other.
16. In order for someone to be diagnosed with schizophrenia, they must experience visual
hallucinations.
17. Schizophrenia has only two
stages of development.

18. App
earance of cognitive psychology was a challenge to the limits of behaviorism.
19. Object permanence refers to children’s understanding that objects exist and behave
independently of their actions or awareness.
20. a longitudinal design, the same individuals are repeatedly observed and tested over time,
often for many years.
……………………………………………………………………………………………
Version II :

1. With mild, unpleasant stimulation, the sympathetic division is more active; with mild,
pleasant stimulation, the parasympathetic division is more active.
2. There are two distinct systems in the brain that handle approach-related and withdrawal-
related emotional responses.
3. Placing blame for one’s difficulties on others or attributing one’s own “forbidden” desires to
others is reaction formation
4. Experimental group is a group in an experiment that is not exposed to a treatment or does not
experience a manipulation of the independent variable.
5. Within-subjects design is a procedure that ensures that every member of a population has an
equal likelihood of participating in an experiment.
6. Selective optimization with compensation is a strategy for successful aging in which one
makes the most gains while minimizing the impact of losses that accompany normal
aging. According to Freud, Eros, as a broadly defined sexual drive, does not suddenly appear at
puberty but operates from birth. One of the major obstacles of psychosexual development, at least
for boys, occurs in the phallic stage. An archetype is a primitive symbolic representation of a
particular experience or object.
7. Unconditional positive regard is striving to realize one’s inherent potential.
8. Cardinal traits are traits around which a person organizes his or her life.

9. Psychologists define personality as the complex set of psychological qualities that


influence an individual’s characteristic patterns of behavior across different situations and
overtime.
10. B. F. Skinner suggested that children acquire language through ordinary processes of
learning.
11. Developmental psychologists Study the changes that occur in the physical, cognitive, and
social functioning of individuals across the life span; study the influence of genetics and
environments on those changes.
12. A hypothesis is a tentative and testable statement about the relationship between
causes and consequences.
13. A variable is any factor that
varies in amount or kind.
………………………………………………………………………………………………………
Version III :

42. Vygotsky’s concept of internalization helps explain the effect culture has on cognitive
development.

52. fluid intelligence shows that it is positively correlated with age


53. Naturalistic observation is an intensive observation of a particular individual or small
group of individuals.
55. Predictions in psychology are statements about the likelihood that a certain behavior will
occur or that a given relationship will be found.
56. The cortex is involved in emotional experiences through its internal neural networks and its
connections with other parts of the body.
57. Self-concept is a concept in personality psychology referring to a person’s constant striving to
realize his or her potential and to develop inherent talents and capabilities.
61. In a longitudinal design, the same individuals are repeatedly observed and tested over time,
often for many years.

62. According to behaviorist theory, behavior is driven, or motivated, by powerful inner forces

63. Maslow, and their colleagues defined a perspective that strives to deal with the whole person,
practicing a holistic approach to human psychology.
64. Positive psychology is a movement within psychology that applies research to provide people
with the knowledge and skills that allow them to experience fulfilling lives.

65. Stressor is the pattern of specific and nonspecific responses an organism makes to stimulus
events that disturb its equilibrium and tax or exceed its ability to cope.

66. According to Rotter, expectancy is the extent to which people believe that their behaviors in
particular situations will bring about rewards.
69. Encodings is the way you categorize information about yourself, other people, events, and
situations.
70. reciprocal determinism a concept of Albert Bandura’s social-learning theory that refers to the
notion that a complex reciprocal interaction exists among the individual, his or her behavior, and
environmental stimuli and that each of these components affects the others.
…………………………………………………………………………………………………….
Version I :
51. 4 goals of psychology
are to describe, explain, predictand control behavior
52. Serotonin reuptake inhibitors (SSRIs) are mostly prescribed to patients who are

depressed
53. Benzodiazepines
enhance the effect of the neurotransmitter GABA

54. The way you categorize information about yourself, other people, events, and situations is
encoding
55. A belief that one
can perform adequately in a particular situation is self-efficacy
56. Generalized evaluation of the self is a
person’s self-esteem
57. Terror management theory proposes that self-esteem helps people cope with the
inevitability of death
58. case study is intensive observation of a particular individual or small group of individuals
59. Informed consent is the process through which individuals are informed about
experimental procedures, risks, and benefits before they provide formal consent to become
research participants.
……………………………………………………………………………………………………
Version II :
1. In Maslow’s hierarchy of needs biological physiological needs are considered the most
primary
2. Correlation does not imply causation
3. significant difference A difference between experimental groups or conditions that would
have occurred by chance less than an accepted criterion; in psychology, the criterion most
often used is a probability of less than 5 times out of 100, or p <.05.
4. DSM stands for diagnostic and statistical manual
5. Behavior is the actions by which an organism adjusts to its environment.
6. Psychometrics is the field of psychology that specializes in mental testing in any of its
facets, including personality assessment, intelligence evaluation, and aptitude measurement.
7. Feelings of helplessness and hopelessness, loss of interest in daily activities, appetite or
weight changes could be the symptoms of depression
8. Depression disorder has the highest suicide rate.
………………………………………………………………………………………………………
Version III :
71. To discover the basic elements, Titchener relied on the technique of introspection
72. According to the psychodynamic perspective, behavior is driven, or motivated, by powerful
inner force.
73. Behavioral measures are ways to study overt actions and observable and recordable reactions.
74 self report measures are verbal answers, either written or spoken, to questions the researcher
poses.
75. hallucinogen is a drug that alters cognitions and perceptions and causes hallucinations.
76. morality is a system of beliefs and values that ensures that individuals will keep their
obligations to others in society and will behave in ways that do not interfere with the rights and
interests of others.
77. gender is a psychological phenomenon that refers to learned sex-related behaviors and attitudes
of males and females.
78.gender stereotype is a belief about attributes and behaviors regarded as appropriate for males and
females in a particular culture.
79. Erikson described intimacy as the capacity to make a full commitment to another person.
80.autonomic nervous system (ANS) prepares the body for emotional responses through the action
of both its sympathetic and parasympathetic divisions
…………………………………………………………………….
1._ _ _ _ _ _ _ _ _ _ _ _ _ _ _fundamental attribution error (FAE) _ _ _ _ _ _ _ _ _ _ _ _ _ _ _ _ _ _ _
represents the dual tendency for people to overestimate dispositional factors (blame or credit people)
and to underestimate situational factors (blame or credit the environment) when searching for the
cause of some behavior or outcome.
2. When people doubt their ability to perform a task, they may engage in _ _ _ _ _ _ self-handicapping_
_ _ _ _ _ _ _ _ _ _ _ _ _ _ _ _ _ _ _ _ _ _ _ _ _ _ _ — They deliberately sabotage their performance. The
purpose of this strategy is to have a ready-made excuse for failure that does not imply lack of ability.
3. George L. Engel’s _ _ _ _ _ _ _ _ _ biopsychosocial model of health and illness _ _ _ states that the
causes, manifestations, and outcomes of wellness and diseases are determined by the interactions
between biological, psychological, and social factors.
4. The same situations might cause stress in some people but not in others. According to Lazarus, to
experience some situations as stressful, first step is to evaluate the situations as stressful. _ _ _ _ _ _ _ _
_ _ _ _cognitive appraisal _ _ _ _ _ _ _ _ _ _ _ _ _ _ _ _ _ _ _ _ is the cognitive interpretation and
evaluation of a stressor.
5. According to Schachter, the experience of emotion is the joint effect of the two factors: _ _ _ _ _ _ _ _
_ _ _ _ _ _ _ _physiological arousal _ _ _ _ _ _ _ _ _ _ _ _ _ _ _ _ _ and cognitive appraisal.
6. _ _ _ _ _ _ _ _gender serotypes _ _ _ _ _ _ _ _ _ _ _ _ _ _ _ _ _ _ _ _ _ _ _ _ are beliefs about attributes
and behaviors regarded as appropriate for males and females in a particular culture.
7._ _ _ _ _ _crystallized intelligence _ _ _ _ _ _ _ _ _ _ _ _ _ _ _ _ _ _ _ _ _ _ _ _ _ _ _ _ involves the
knowledge a person has already acquired and the ability to access that knowledge; it is measured by
tests of vocabulary, arithmetic, and general information.
8. Four main goals of psychologists are describing, explaining, _ _ _ _ _ predicting_ _ _ _ _ _ _ _ _ _ _ _
_ _ and controlling behavior
9.When different people observe the same events, because of their personal motives or expectations,
they don’t always “see” the same thing. What can be a remedy / solution for this challenge to
objectivity? _ _ _ _ _ _ standardization and operional definitions_
Parents use various reinforcement schedules to teach children new behavior, strengthen the desired
behavior or try to stop undesired behavior. A continuous schedule is often the best in teaching a new
behavior. Once the response has been learned, _ _ _ _ _ _ _ _positive reinforcement _ _ _ _ _ _ _ _ _ _ _
_ _ _ _ _ _ _ _ can be used to strengthen the new behavior.
After listening to a list of words such as hot, snow, warm, winter, ice, wet, frigid, chilly, heat, weather,
freeze, air, shiver, Arctic, and frost, people often remember having heard the related word cold even
though it was not presented (Gallo, 2006; Roediger & McDermott, 1995).
This happens due to _ _ _ _ _ _ _ _ _ _ _ _ _encoded to long term memory _ _ _ _ _ _ _ _ _ _ _ _ _ _ _ _
_________________

…………………………………………………………………………..
1. According to Piaget, assimilation restructures or modifies the child’s existing schemes so that new
information is accounted for more completely.
2. Gardner’s Multiple Intelligences theory suggests that whether people people have mathematical and
linguistic intelligence or not is determined by the heritability estimate
3. Retrieval cues are the stimuli available as you search for a particular memory.
4. When a behavior is followed by the removal of an aversive stimulus, the event is called negative
reinforcement.
5. Reflex is a learned response elicited by specific stimuli that have biological relevance for an
organism.
6. Groups make decisions that are riskier and less conservative than would be made by the members
acting alone.
7. A correlation coefficient of -0.7 indicates a weaker relationship between two variables than a
correlation coefficient of 0.4.
8.Each perspective of psychology (the humanistic, cognitive, sociocultural, evolutionary, humanistic,
behaviorist perspectives) stress the biological, psychological, or social-cultural factors more than
another, each of the perspectives are interesting and valuable by themselves but unfortunately for the
science of psychology, the different perspectives contradict one another, they’re impossible to use in
combination and you have to choose one of them that you agree with more.
9.In a Carr and VanDeusen (2004) study, male college students at a large midwestern university
completed an anonymous survey on which they indicated whether they had ever engaged in sexually
coercive behavior as well as the frequency with which they viewed various forms of pornography. The
researchers found a statistically significant correlation, such that the more pornography the students
reported using, the greater the likelihood that they had committed sexual violence (Hald, Malamuth,
& Yuen, 2010). From this study we can draw the conclusion that using pornography made the male
students more likely to commit sexual violence.
10. Every time Ana posts half-naked photographs of herself, she gets lots of attention, likes and
comments on social media. Now uploading half-naked photos and receiving attention became strongly
associated for her, so the number of such posts on her page is steadily increasing. This is an example of
classical conditioning.
11. The difference between flashbulb and other types of memories is that flashbulb memories are
accurate because processes of reconstruction do not apply to them.
…………………………………………………………………………………………
People tend to take credit for their successes while denying or explaining away responsibility for their
failures. In many situations, people tend to make dispositional attributions for success and situational
attributions for failure. “I got the prize because of my ability”; “I lost the competition because it was
rigged.” These are examples of
I. self-fulfilling prophecy.
J. confirmation bias.
K. self-serving bias.
L. covariation bias.

(1 point) A teacher believes boys are inherently better at math, so he doesn’t spend equally enough
time and energy to helping female and male students. As a result, his male students get higher scores
on final math exam than girls. This difference between the final performance levels between girls and
boys in his class occurred because of the teacher’s
I. social role.
J. informational influence.
K. self-fulfilling prophecy.
L. self-serving bias.

(1 point) When a group makes a decision there’s a tendency to filter out undesirable input so that a
consensus may be reached, especially if it is in line with the group leader’s viewpoint. This tendency is
referred to as
I. group conformity.
J. group polarization.
K. groupthink.
L. group influence.

(1 point) When examining a patient’s problem every doctor in a team came to the same conclusion
except one of them but even though he had a different opinion he decided not to voice his different
view so that he wouldn’t feel fool and deviant from others. This is an example of
E. conformity.
F. social cognition.
G. obedience.
H. social role.

(1 point) According to Rogers, self-actualization is


I. the use of psychological theory to explain an individual’s course through life.
J. a person’s mental model of his or her typical behaviors and unique qualities.
K. a constant striving to realize one’s inherent potential.
L. complete love and acceptance of an individual by another person.

(1 point) The values for the Independent Construal of the Self in individualistic cultures (such as the
USA) are:
I. connectedness with others, group harmony and social relations.
J. personal freedom, self-fulfillment, independence, self-reliance.
K. loyalty to one’s group, duty to one’s family, adherence to social traditions.
L. enhancement of one’s groups’ status and well-being.

(1 point) Some people believe that events in their life derive primarily from their own actions. For
example, when receiving exam results, they tend to praise or blame themselves and their abilities, not
the teacher or the exam test. These people are said to have
I. internal locus of control.
J. acquired introversion.
K. external locus of control.
L. acquired extroversion.

(1 point) Self-concept refers to


I. the idea that people observe themselves to figure out the reasons they act as they do.
J. a person’s mental model of his or her typical behaviors and unique qualities.
K. the set of beliefs that one can perform adequately in a particular situation.
L. a generalized evaluative attitude toward the self that influences both moods and behavior.

(1 point) Which of the following is not a source of self-efficacy judgments?


I. Vicarious experience (your observations of the performance of others).
J. Persuasion (others may convince you person that you can do something, or you may convince
yourself).
K. Monitoring of your emotional arousal as you think about or approach a task (e.g. anxiety suggests
low expectations of efficacy; excitement suggests expectations of success).
L. Self-handicapping (you anticipate failure and you deliberately sabotage your performance so that
you can have a ready-made excuse for failure).

(1 point) Which of the following is not correct?


I. Wellness is a general condition of soundness and vigor of body and mind; not simply the absence of
illness or injury.
J. Health promotion is the development and implementation of general strategies and specific tactics to
eliminate or reduce the risk that people will become ill.
K. Health is a general condition of soundness and vigor of body and mind; not simply the absence of
illness or injury.
L. Wellness is optimal health, incorporating the ability to function fully and actively over the physical,
intellectual, emotional, spiritual, social, and environmental domains of health..

(1 point) According to the Transtheoretical Model (Stages of Change), when changing their unhealthy
behaviors to healthy ones, individuals move through six stages of change:

E. precontemplation, contemplation, preparation, action, maintenance, and termination.


F. action, termination, precontemplation, contemplation, preparation and maintenance.
G. preparation, precontemplation, contemplation, action, maintenance, and termination.
H. contemplation, precontemplation, action, maintenance, preparation, and termination.

(1 point) Failing to adhere to treatment regimens that doctors give to their patients is one of the most
serious problems in health care. Which of the following statements about patient adherence is correct?
I. Generally, patients who perceive greater threat from a disease also show greater likelihood to adhere
to treatment regimen given by a doctor.
J. Patients who face serious diseases that leave them in poor physical health always show higher levels
of adherence than patients who are less debilitated by the same diseases.
K. Physicians should stop trying to understand their patients’ attitudes and modify their behaviors to
match those attitudes.
L. When patients can rely on social support that allows them to accomplish their regimens correctly
they are less likely to adhere to treatments.

(1 point) Out of the seven strategies listed below, which ones does Problem-Directed Stress Coping
involve?
8. Fight (destroy, remove, or weaken the threat)
9. Somatically focused activities (use of antianxiety medication, relaxation, biofeedback)
10. Flight (distance oneself from the threat)
11. Cognitively focused activities (planned distractions, fantasies, thoughts about oneself)
12. Seek options to fight-or-flight (negotiating, bargaining, compromising)
13. Therapy to adjust conscious or unconscious processes that lead to additional anxiety
14. Prevent future stress (act to increase one’s resistance or decrease strength of anticipated stress)

E. 1, 2, 3, 6
F. 2, 4, 3, 7
G. 1, 3, 5, 7
H. 2, 3, 5, 6

(1 point) Traditionally psychologists assumed that subjective mental experience of an emotion


preceded the bodily expression or action. However, according to the James-Lange theory of emotion,
the arousal of the physical response precedes the appearance of the emotion (perceiving a stimulus
causes autonomic arousal and other bodily actions that lead to the experience of a specific emotion).
Which of the following would not serve as an appropriate example of James’ and Lange’s
understanding of how we experience emotions?

I. I see a wild animal, I feel fear, and I run away.


J. I see a wild animal, I run, and then I experience the emotion of fear.
K. I cry because I feel sorry.
L. I’m afraid because I tremble.

(1 point) In his experiments Robert Zajonc demonstrated mere exposure effect which describes how
sometimes people have preferences without knowing why. The mere exposure effect means that
I. repeated exposure to a stimulus (such as a song, a person, a painting) increases liking of that
stimulus.
J. repeated exposure to a stimulus (such as a song, a person, a painting) makes a perceiver feel bored of
that stimulus.
K. repeated exposure to a stimulus (such a song, a person, a painting) increases perceivability of the
details that went unnoticed before.
L. repeated exposure to a stimulus (such a song, a person, a painting) decreases willing to pay attention
to more details.

(1 point) According to the Kohlberg’s theory of moral development, on the last stage of moral
reasoning a person’s intention to behave morally is
I. To avoid pain or not to get caught
J. To gain acceptance and avoid disapproval
K. To be true to universal principles and feel oneself part of a cosmic direction that transcends social
norms
L. To promote the society’s welfare

(1 point) Children at the preoperational stage (roughly from 2 to 7 years of age) are unable to take the
perspective of another person. For example, a child talking on the phone might pinpoint to a toy that
she/he is talking about with the other person on the phone, she can’t take into account that the person
on the phone can’t see what she/he is pinpointing with his/her fingers to.
In Piaget’s theory, this inability is referred to as
I. centration.
J. conservation.
K. egocentrism.
L. assimilation.

(1 point) As children’s cognitive abilities develop, they come to understand that other people have
cognitive experiences of the world—and that those cognitive experiences may not be exactly the same.
Children become able to explain and predict other people’s behavior based on an understanding of
their mental states. In other words, children develop
I. expectancy theory.
J. theory of mind.
K. equity theory.
L. theory of individual psychology.

(1 point) According to Gardner’s Multiple Intelligences theory, some people have great ability to plan
and understand sequences of movements and they’re more likely to become great dancer or athletes.
This is to say that these people have high
I. bodily-kinesthetic intelligence.
J. existential intelligence.
K. spatial intelligence.
L. naturalist intelligence.

(1 point) Which of the following abilities does emotional intelligence involve?


9. The ability to perceive, appraise, and express emotions accurately and appropriately
10. The ability to use emotions to facilitate thinking, to understand and analyze emotions
11. The ability to use emotional knowledge effectively
12. The ability to regulate one’s emotions to promote both emotional and intellectual growth

I. 1, 2, 3
J. 1, 3, 4
K. 1, 2, 3, 4
L. 2, 3, 4

(1 point) A procedure conducted at the end of an experiment in which the researcher provides the
participant with as much information about the study as possible and makes sure that no participant
leaves feeling confused, upset, or embarrassed is called
E. informed consent.
F. cover story.
G. debriefing.
H. intentional deception.

(1 point) Which one of the following statements about memory is correct?


I. Semantic memories preserve the specific events that you have personally experienced.
J. Recognition is a method of retrieval in which an individual is required to reproduce the information
to which he/she was previously exposed.
K. When individuals are motivated enough to remember information accurately their memories can be
absolutely accurate.
L. No matter how emotionally important a memory or information can be for an individual, memories
are can always be subject to misinformation effect.

(1 point) “Boys are more physically aggressive than girls because a male hormone testosterone is linked
with aggression.” This statement is likely made by a psychologist working in _ _ _ perspective of
psychology.
I. the biological perspective
J. the evolutionary perspective
K. the psychodynamic perspective
L. the behaviorist perspective
(1 point) “In this study, Japanese students refrained themselves from expressing as anger and disgust
because in Japanese culture it’s not that desirable to show negative emotions.” .” This statement is
likely made by a psychologist working in _ _ _ perspective of psychology.
E. the humanistic perspective
F. the cognitive perspective
G. the sociocultural perspective
H. the evolutionary perspective
(1 point) “Our beliefs about ourselves matter. Do we see ourselves as in control of things? Do we view
ourselves as relatively superior or inferior? Our answers influence our emotions and actions.” This
statement is likely made by a psychologist working in _ _ _ perspective of psychology.
E. the humanistic perspective
F. the cognitive perspective
G. the sociocultural perspective
H. the behaviorist perspective

(1 point) To test the effect of a new drug on depression, a researcher randomly assigns people to
control and experimental groups. Those in the experimental group take a pink pill containing the new
medication; those in the control group take a pink pill that contains no medication. Which statement is
true?
I. The medication is the dependent variable.
J. Depression is the independent variable.
K. Participants in the control group take a placebo.
L. Participants in the experimental group take a placebo.

(1 point) A researcher wants to test the hypothesis that people are less likely to help a person having a
seizure in the street if there are some other people witnessing this situation than if they’re alone. In the
experiment he plans, the researcher will vary the number of people witnessing the situation where an
actor (an assistant of the experimenter) acts as if he’s having a seizure. The number of people will be
his
E. placebo control.
F. double-blind control.
G. independent variable.
H. dependent variable.

(1 point) A double-blind procedure is often used to prevent researchers’ biases from influencing the
outcome of an experiment. In this procedure,
I. only the participants know whether they are in the control group or the experimental group.
J. experimental and control group members will be carefully matched for age, sex, income, and
education level.
K. neither the participants nor the researchers know who is in the experimental group or control group.
L. someone separate from the researcher will ask people to volunteer for the experimental group or the
control group.

(1 point) Ana started working for a pet food store. In the beginning she was annoyed by the smell in the
store but after some period of time she got used to it. What is the technical term for “getting used to
bad smell” to the extent that it doesn’t bother Ana, and that she doesn’t react to it anymore?
I. Sensitization.
J. Stimulus discrimination.
K. Habituation.
L. Response acquisition.
(1 point) When playing with a slot machine players have no way of knowing how many times they have
to play before they win. All they know is that eventually, a play will win. This is why slot machines are
so effective, and players are often reluctant to quit. There is always the possibility that the next coin
they put in will be the winning one.
What reinforcement contingency do they psychologists use to design slot machines?
E. Variable-Ratio Schedule
F. Fixed-Ratio Schedule
G. Fixed-Interval Schedule
H. Intermittent Reinforcement Schedule
(1 point) When a toddler throws a tantrum in the store, parents usually refuse to give in. But once in a
while, when they’re tired or in a hurry, they decide to buy the candy, thinking they will do it just that
one time. This is an example that in daily life, parents sometimes unknowingly _ _ undesired
behavior.
I. reinforce
J. extinct
K. punish
L. discriminate

(1 point) Students usually remember what was they read in the beginning and the end of a book
chapter and they don’t remember the middle sections quite well. This is due to
E. reconstructive nature of memory.
F. the serial position effect.
G. semantic encoding principle.
H. encoding specificity effect.

(1 point)Tom waits remembers he was born on 7th December, 1949. This is his
I. semantic memory
J. procedural memory
K. episodic memory
L. flashbulb memory

(1 point) Suppose you’re on your way home from watching a scary movie about a hitchhiker who was a
murderer when you see someone talking loudly with a friend. Because you saw the movie, you assume
that you are witnessing an argument that will probably end in a fight. This is an example of
I. priming.
J. meta-memory.
K. context effects.
L. spontaneous recovery.

1- Serotonin is one of the four major neurotransmitters that regulate mood the statement is likely
made by a psychologist working in -------------perspective of psychology

the biological perspective


the evolutionary perspective
the psycho dynamic perspective
the behaviorist perspective

2-the traditional biomedical model of health links among the nervous system, the immune system,
behavioral styles, cognitive processing, and environmental domains of health
true false
3-a person has no intention of changing their behavior in the near future. According to the
prochaskas model, he-she is on the stage of
precontemplation
contemplation
preparation
action
maintenance

4- Children tend to imitate what a model does and says, whether the modeled behavior is prosocial
(positive, constructive, and helpful or antisocial
true false

5-A hostile attitude toward members of a group, based solely on their membership in that group is
referred to as ------prejudice---

6- Our stress response system is well suited to acute stress but less effective is the face of chronic
stress
true false

7-the smoker is thinking about quitting but has not yet undertaken any behavioral changes. He-she
is on the stage of
precontemplation
contemplation
preparation
maintenance

8- Bed wetting alarms have been used against nocturnal enuresis in adults since the beginning of the
20th century. The enuresis alarm is triggered when a sensor in the sheets or night clothes became
wet with urine. This sets of a signal which wakes the person to void in the toilet. Ensure is alarms
teach the users to wake up when they need to urinate. in terms of classical conditioning, alarm
sound is:
unconditioned stimulus (triggers response result of the unconditioned response) “Food smell”
unconditioned response (response to the stimulus) “salivating/feel of hunger”
conditioned stimulus (neutral stimulus that doesn’t give response until conditioning has occurred”
conditioned response
other choices are
Classical conditioning
Operant conditioning
habituation
sensitization
A and B
A and C
B and C

9-correlation coefficients of .50 and -.50 indicate relationship that are of different strengths
true false

10-playing the piano uses ----long-term ----- memory

11- the-------- method is most likely to use a double -blind design


A-self-report
B-behavioral
C-experimental
A and B
B and C
A ,B and C

12- couples in happy marriages tend to make --------attributions for each others positive actions
(that's just the way he is )and----------attributions for negative actions (hes just so busy providing for
all of us that he does not have the time)
self- serving external
self – serving internal
internal- external
external- internal

13-what pattern of remembering emerged in Hermann Ebbinghaus research?


Loss occurred at a steady rate
a small initial loss was followed by no further loss
there was no initial loss, but then there was a gradual decline
a sharp initial loss was followed by a gradual decline

14- confirmation bias can lead people to be believe things that are not true because evidence can
generally by found in support of even the most questionable propositions
true false

15- Selyes general adaptation syndrome consists of a reaction followed by


fight or flight
resistance then exhaustion
challenge then recovery
stressful the events

16- the actual reason you enjoy eating, exercise, or ex is that during these activities brain
endorphins which are hormones that reduce pain and boost pleasure. This statement is likely made
by a psychologist working in ---------perspective of psychologist
the biological perspective
the evolutionary perspective
the psycho dynamic perspective
the behaviorist perspective

17- to cope with stress, we tend to use -------------- strategies when we feel in control of our world,
and to use ------------strategies when we believe we cannot change a situation
emotion -focused, problem -focused
problem- focused, emotion-focused
positive-emotion , negative-emotion
positive- emotion ,negative emotion
negative emotion ,positive emotion

18-using of antianxiety medication is problem -directed stress -coping strategy


true false
18-nine in 10 students rate a condom as effective if it has a supposed (95)percent success rate in
stopping the HIV virus than causes AIDS only 4 in 10 think it successful given a (5) percent failure
rate (Linville et al , 1992)this is ----framing-------effect

19- Are there alternative ways of interpreting the evidence? this is a question that a critical
information consumer should ask after reading a conclusion
true false (not sure)

20- Joining the end of a ticket line is an example of -----conformity------whereas forming two lines
when a theater employee requests it is an example of compliance

21-the best psychological perspective so far in the sociocultural perspective since its view on human
nature is not limited to ethnocentric research conducted on western societies only and takes cultural
diversity into account
true false

23-according to the observational learning principle, children do what they see, if a models actions
and words are in consistent, children may imitate the hypocrisy they observe
true false

24-if you wanted stop bullying in your school, what would most likely be an effective strategy to
follow? Any victims
Punish the bully publicly
teach the victim to fight back
reward the bully for not bullying (not sure)
change the entire school system to have zero tolerance for bullying

25- if a questionnaire questionnaire aims at measuring people levels of empathy and in fact
measures peoples capacity to place oneself in anthers position, we can say that this questionnaire----
Validity-----

26-(i am good, you are lucky)-the phrase shortly summarizes the idea behind self-serving
attributional bias
true false

27- A research article (functional neuroimaging study revealed the impact of early parent – infant
interactions on children brain describes a study. In this study of 125 full-term and premature,
newborns at Nationwide Children Hospital in Columbus. Ohio, found early, Gentle displays of
affection from parents have lasting effects on how baby brains react to gentle touch. The authors
found that the more the parents hugged their toddlers, the more developed brains their had. The
children that were hugged more also showed more positive brain responses than children whose
parents hugged them less this study does not suggest the conclusion that the development rate on
the infants was caused by the frequency of parents hug.
true false

28- If you were a victim in an emergency, what lessons from social psychology whould you apply
to get the help you need
ask for it
make your request specific
engage particular individual observers
do all of the above
29- If a man begins dating a man whom he feels strongly connected to, he may feel that this person
is (the one) since the expects the relationship to last, he treats his partner with love and respect, as a
consequence the relationship ends up fulfilling. This exemplifies self-fulfilling prophecy
true false

30- what is stereotype threat?


Some members of a majority group feel threatened by minorities
some members of a stereotyped group feel anxious when they are made aware of a stereotype about
them
some members of a the majority group threaten a minority by stereotype them
some members of a minority group threaten against the stereotype they find unfair

31-people tend to make self-serving attributions which means they have the tendency to believe that
people behavior corresponds to (matches) their dispositions
true false

32- Which of the following statements is true?


The reasons behind the relationships they reveal cannot be established by correlational studies alone
Experiments can reveal cause and effect relationships between variables unless the researches use
randomization
Psychologist try to obtain representative sample in order to achieve random sampling
naturalistic observation entails watching behavior occurring in its natural setting and altering the
Behavior being observed

32- Prejudice is-----------,while discrimination is a (n)---------


behavior-attitude
attitude-behavior
instinct-choice
stimulus-response

33- When there is no knowing when the waiting will be over, peoples persistent behavior of re-
checking there e-mails is rewarded with --------------schedules
variable ratio
fixed ratio
variable interval
fixed interval

34- In which stage of the general adaption syndrome are the pituitary and adrenals stimulated?
Exhaustion
reaction
alarm
resistance

35-driving to school one wintry day, George narrowly misses a car that slides through a red light.
“slow down! What a terrible driver.” he thinks to himself. Moments later, George himself slips
through an intersection and yelps, “wow! These roads are awful. The city snow plows need to get
out here.” What social psychology principle has George just demonstrated?
ANSWER -------- Fundamental Attribution Error
36- couplesin happy marriages tend to make---------attributions for each others positive actions
(thats just the way he is )and ------------atributions for ngative actions(he is just so busyproviding for
all of us that he doesnot have the time)
self-serving -external
self serving-internal
internal-external
external-internal

37-feeling sad hearing upon a song associated with a long-lost relationship illustrates
acquisition
sensitization
spontaneous recovery
experimental extinction

38-in horror movies,sexually arousing images of women are sometimes paired with violence against
women.in terms of classical conditioning principles,after the pairing ,sexual arousal stemming from
viewing an attractive semi-nude woman is
an unconditioned stimulus
an unconditioned response
a conditioned stimulus
a conditioned response

39-which of the following is true about the ethical conduct of psychological research?
it is good scientific procedure to tell participants about the research hypotheses before they
participate.
if research participants are misied about a study they must be fully debriefed the end of the study.
research participantsara should not approve any public sharing of the records of their behavior
it is never permissible to use deception .

40- when experimenting with the influence of music on consumer choice ,75 %of subjects
given a pairing of the colored pen with the attractivemusic chose the pen over a non-
exposedpen when given a choice .and only 30 %of the participants chose the pen paired
with unattractive music if they had received a pairing of this pen with the aversive music
What form of learning manifested itself in this experiment?
Experiential learning
experimental learning
classical conditioning
observational learning

41-when Gordon Allport said that (defeated intellectually, prejudice lingers emotionally)what did he
mean?
You cannot argue intellectually with a prejudiced person
a prejudiced person cannot intellectually defend his or her attitude
a person implicit prejudices may decline while explicit prejudices remain
a persons explicit prejudices may decline while implicit prejudices remain
42- if a study shows positive correlation between holding authoritarian beliefs and infectious
diseases ,it means that
people with more authoritarian beliefs tend to have infectious diseases more
infectious diseases are more common in pople that dont hold authoritarian beliefs
people that oppose authoritarian beliefs tend to have more infectious diseases
infectious diseases are less common in people that hold authoritarian beliefs
43- What was the independent variable in the Stanford Prison Experiment?
prisoner or guard roles
IQ level differences of those in the two roles
cultural backgrounds of the volunteers
all of the above
44-Out of the seven strategies listed below, which on. does Problem-Directed Stress Coping
involve? 1. Fight (destroy, remove. or weaken the threat) 2. Somatically focused activities (use of
antianxiety medication, relaxation, biofeedback) 3. Flight (distance oneself from the threat) 4.
Cognitively focused activities (planned distractions, fantasies, thoughts about oneself) 5. Seek
options to fight-or-flight (negotiating, bargaining. compromising) 6. Therapy to adjust conscious or
unconscious processes that lead to additional anxiety 7. Prevent future stress (act to increase one's
resistance or decrease strength of anticipated stress)
1, 2, 3, 6
2, 3,7
1357
2355
45- A researcher wants to test the hypothesis that people are less likely to help a person having a
seizure in the street if there are some other people witnessing this situation than if they're alone. In
the experiment he plans, the researcher will vary the number of people witnessing the situation
where an actor (an assistant of the experimenter) acts as if he's having a seizure. The number of
people will t his
placebo control
double-blind control
independent vanable
dependent variable
46-Our beliefs about ourselves matter. Do we see ourselves in control? Do we view ourselves as
relatively superior or inferior? Our answers influence our emotions and actions' This statement is
likely made by psychologist working in ------------- perspective of psychology.
the humanistic perspective
the cognitive perspective
the sociocultural perspective
the behaviorist perspective
47-Which heuristic can best explain why so many people believe in the pseudoscience of astrology?
------------------ Representativeness Heuristic ------------------
48- George knows and likes most of his Armenian classmates but privately believes that his
Georgian culture is superior to all others. His belief is evidence of his
anti-Armenian prejudice.
stereotyping a minority.
ethnocentrism.
out-group homogeneity.
49- ---------- standardization/ operational procedures ------------method involves predetermined
steps aimed at minimizing biases and errors to ask questions, gather and analyze data and answer
the questions accordingly.
51-In yoga classes, students leam through watching their teacher and classmates. It is an
demonstrates that observational learning_ _ _ plays many roles in sports and activities.
52- In emergency situations, the larger the number of people present, the higher the probability of
helping.
true false
53-Advertisers often exploit classical conditioning principles. They try to create associations in
people's mind between the products (for example, sports cars) and passion. In this case, the
elements in then advertisement (e.g. sexually provocative individuals or situations) serve as ------
and feelings of sexual arousal is -------------. The product itself (in this case, sports cars) is ---------,
so that the feelings of arousal will become associated with it .
unconditioned stimulus, unconditioned response, conditioned stimulus
unconditioned response, conditioned response, conditioned stimulus
unconditioned stimulus, neutral stimulus, conditioned response
conditioned stimulus, unconditioned response, unconditioned shmulus
54- A stereotype is
the cognitive component of a prejudice.
a negative impression of a group of people.
always inaccurate.
cognitive summary that can be positive or negative
55- The _ _ _ method is most likely to use a double-blind design.
A . self-report
B. behavioral
C. experimental
A and B
B and C
A, B and C
56- A procedure conducted at the end of an experiment in which the researcher provides the
participants with as much information about the study as possible and makes sure that no participant
leaves feeling confused, upset, or embarrassed is called:
informed consent.
cover story.
debriefing.
intentional deception.
58-Each psychological research method could be used to explore a specific research question is a
powerful tool in some ways and a weak tool in others.
true false
59- What pattern of remembering emerged in Hermann Ebbinghatis's research?
Loss occurred at a steady rate.
A small initial loss was followed by no further loss.
There was no initial loss, but then there was a gradual decline
a sharp initial loss was followed by a gradual decline

60- In classical conditioning, the consequences of behavior. such as rewards and punishments,
influence the probability that the behavior will occur again
true false (if its operant conditioning then its true but it says classical)
61- When we try to categorize something by judging how similar it is to our conception of the
typical member of the category. we use representativeness heuristic.
True false
62- If you believe that people are poor and obese only because they're lazy, you are committing the
self- depreciation_ __ error.
63-The number of short - term illnesses and stress - related psychological disorders was higher than
usual in the months following an earthquake. Such findings suggest that
daily hassles have adverse health consequences.
experiencing a very stressful event increases a person's vulnerability to illness.
the amount of stress a person feels is directly related to the number of stressors experienced.
small, bad events don't cause stress. but large ones can be toxic.
64- When consumers respond more positively to ground beef described as '75 percent lean' than to
the same product labeled -25 percent fat.' they have been influenced by
believe perseverance.
belief fixation
confirmation bias.
Framing effects
65----Domestication------------- is an action of human beings and nonhuman animals that serves as
the means to adapt to their environments..
66- To improve your studying and memory, it is advised to use elaborative rehearsal and creating a
variety of memory cues for each concept.
true
false
67- In-depth interview is an example of behavioral measure.
True false
68-Fantasizing about winning money is a problem-focused way of coping with financial stress.
True false
69- Using of antianxiety medication is a problem-directed stress-coping strategy.
true False
70-The _ _ _ method is most likely to use a double-blind design.
A self-report
B. behavioral
C. experimental
A and B
B and C
A B and C
71-The predictions implied by a theory are called
operational definition
correlations.
hypotheses.
Replications.
72-What was the independent-variable in the Stanford Prison Experiment?
Prisoner or guard role
IQ level differences of those two roles
cultural backgrounds of the volunteers
all of the above
73-The Stanford prison experiment illustrates the power of to influence people's behavior
personality
childhood experiences
heredity
In the situation
74- "Advertisers often exploit classical conditioning principles. They try to create associations in
people's mind between their products (for example, sports cars) and passion. In this case, the
elements in their advertisement (e.g. sexually"provocative individuals or situations) serve as _ __
and feelings of sexual arousal is _ _ _ The product itself (in this case," sports cars) is _ __ , so that
the feelings of arousal will become associated with it.
a) Unconditioned stimulus; unconditioned response; conditioned stimulus
75- Seeing someone disobey a questionable order makes people ------------- likely to obey the order
themselves."
willing
prone
less likely
more likely
76- In August 2014, “the ice bucket challenge” exploded on social media, capitalizing on normative
conformity to raise unprecedented amounts of money in the battle against ALS. This exemplifies
the implication of normative social influence.
True False
77- A gymnastics coach encourages her students to improve their routines by watching recordings
of the Olympic gold medal gymnastics performances? This decisions might be informed by the
research on
Observational learning
Operant learning
Pavlovian learning
Experiential learning
79- Which of the following statements is correct:
Stereotype is an unjustifiable negative attitude toward a group and its member, while unjustifiable
negative feeling toward them is prejudice
Discrimination is an unjustifiable negative attitude toward a group and its members, while
unjustifiable behavior towards them is discrimination.
Prejudice is an unjustifiable negative attitude toward a group and its members, while unjustifiable
negative behavior towards them is discrimination
Both stereotype and prejudice stand for unjustifiable negative attitude toward a group and its
members while unjustifiable negative behavior towards them is discrimination
80- "The smoker is not yet thinking about quitting. He she s on the stage of:
Precontemplation
Contemplation
Preparation
Action
Maintenance

82- Which of the following had the least influence on participants' willingness to keep giving
shocks in the Milgram studies?
Loss of personal responsibility
Self-justification
Informational social influence
Participant’s aggression
83- What is a suppressed prejudice?
A person holds a prejudice without being aware of it.
A person has a tendency to become prejudiced under the right circumstances.
A person knows he or she Is prejudiced but chooses not to express It In public.
A person reveals a prejudice subtly, by implying a bias rather than saying so outright.
84-Which of the following statements about daily hassles is true?
Some of tho most common hassles involve threats to survival
As a daily hassles diminish, people's sense well-being increases
More frequent and intense hassles are associated with better health.
The effect of hassles do not accumulate: Many hassles are no worse than a few
85- When examining a patient’s problem every doctor in the team came to the same conclusion
except one of them but even though he had a different opinion he decided not to voice his different
view so that he wouldn’t feel fool and deviant from others. This is an example of:
Conformity
Social cognition
Obedience
Social role
87-"Sex Sells” is a common saying in advertising. With the knowledge about this form of learning,
we can fairly well explain how sexual images in advertisements can condition your response to a
product. What is this form of learning. “Classical conditioning” Unconditioned stimuli.
88-”In this study, Japanese students refrain themselves from expressing as anger and disgust
because in Japanese culture it is not that desirable to show negative emotions.” This statement is
likely made by a psychologist working in ------------ perspective of psychology.
Humanistic
cognitive
sociocultural
evolutionary
89- People’s perceptions, including those of researchers, are subjective and can be influenced by
their motives or expectations. Looking at the same thing, different people might “see” different
thing. What procedure is the solution to minimize this observer bias? Standardization and
operational procedure
90- Researchers (Schemer et al. 2008) found that people were more interested in products that had
been embedded in music videos of artists that they liked and less likely to be interested when the
products were in videos featuring artists they did like. Which form of learning does finding
demonstrate? Classical conditioning
91- Living near a busy train line can be unpleasant. However, over time, most people who live near
major train lines become to the sound of trains going past.
Reinforced
Punished
sensitized
habituation
acquainted
92- Which response shows the stages of the general adaptation syndrome in the correct order?
alarm reaction, exhaustion, resistance
resistance, alarm reaction, exhaustion
exhaustion, resistance, alarm reaction
alarm reaction, resistance, exhaustion
93- "If men define situations as real, they are real in their consequences”. The knowledge of self-
fulfilling prophecy is relevant for understanding this quote from W.I Thomas.
true false
94-According to the Transtheoretical Model (Stages of Change), when changing their unhealthy
behaviors to healthy ones, individuals move through six stages of change:
precontemplation, contemplation, preparation, action, maintenance, and termination
action, termination, precontemplation, contemplation, preparation and maintenance.
preparation, precontemplation, contemplation, action maintenance and termination
contemplation, precontemplation, action, maintenance, preparation and termination.
95- A person is aware their behavior is problematic, but are not committed to do anything about it.
According to the Prochaska's model, he/she is on the stage of
Precontemplation
Contemplation
Preparation
Action
Maintenance
96- In Solomon Asch's experiments, about what percent of participants went along with the group's
obviously mistaken judgment at least once?
90 percent
70 percent
50 percent
30 percent
97- If a man's conditioned fear of spiders is triggered by the sight of other creatures that look like
spiders, he is demonstrating------stimulus generalization--------"
98- Students usually remember what was they read in the beginning and the end of a book chapter
and they don't remember the middle section quite well. This is due to
the reconstructive nature of memory.
the serial position effect.
the semantic encoding principle
the encoding specificity effect
99-All of the following are good examples of the self-fulfilling prophecy except one Which one?
A teacher believes that boys are better at math than girls. and boys in his class do better than girls in
math
Ana is worried that her son is not gifted in music, but he does better at his piano lessons than she
expected
Ana thinks her dog isn't very good at learning tricks. Her dog knows fewer tricks than most dogs_
Ana thinks that members of the Alpha-Beta team members are unfriendly and snobby Whenever he
meets members of this team, they are unfriendly toward him.
100- In order to achieve a fuller understanding of the psychological phenomenon in question it is
important to combine multiple psychological perspective.
True False
101- In contrast to most people, depressed individuals tend to attribute negative outcomes to lasting,
internal causes such as their own traits or lack of ability, but attribute positive outcomes to
temporary, external causes such as good luck or special favors from others. This suggests that ----
self-serving bias---------- may play a role in developing/protecting from depression."
102- Accidental deaths are more likely to be reported by the media, so people find it easier to bring
to mind examples of such deaths than deaths from strokes, so more people end up fearing dying by
an accident than by stroke. This exemplifies:
the representativeness heuristic
base rate information
103- When playing with a slot machine players have no way knowing how many times they have to
play before they win. All they know is that eventually, a play will win. This why machines are so
effective and players are often reluctant to quit. There is always the possibility that the next coin
They put in will be the winning one. What reinforcement contingency do They psychologists use to
design slot machines?
Variable-ratio Schedule
fixed-ratio Schedule
fixed interval schedule
variable interval Schedule
104- Which one of the following can’t be a rule for a consumer of psychological information?
Be suspicious of complex answers to simple psychological questions
Avoid the inference that correlation is causation
105- Physicians’ diagnosis are influenced by how easily they can bring different disease to mind.
This means that they seem to use the -----long term memory--------- when making diagnosis.
106- A teacher believes boy's are inherently better math so he doesn't spend equally enough time
and energy to helping female and male students. As a result, his male students get higher scores on
math exam than girls. This difference between the final performance levels between girls and boys
in his class occurred because of the teacher’s:
social role
informational influence
self-fulfilling prophecy
self-serving bias
107- The value of elaborative rehearsal over maintenance rehearsal has been cited as evidence for
the Levels of Processing model of memory
True False
108-The most successful managers were video recorded at their workplace to analyze their habits
This represents an example of:
naturalistic observation (A research technique in which unobtrusive observations are made of
behaviors that occur in natural environments.)
correlational research
experimental study
case-study
109- If potential benefits gained from the study outweigh possible risks associated with the
experimental procedures, researchers are no allowed to conduct the experiment
true false
110- Knowing that two events are negatively correlated provides:
proof that as one increases, the other decreases.
a basis for prediction.
an explanation of why the events are related.
proof that as one increases, the other also increases.
an indication that an underlying third factor is at work.
111- Researchers are advised to follow the three Rs. Which one doesn’t belong to the list
1)Reduce the number of animals used in research
2)regulate the amount of time animals are exposed to pain and distress.
112- George and Ana meet on a blind date. They get along well until they get into the black
cabriolet to go to a movie. Ana is quiet and reserved for the rest of the evening. It turns out that her
brother had recently been in a serious accident in that same car and seeing it brought up those
unwanted emotions. George assumes that Ana has a cold and reserved personality, thereby
demonstrating:
Actor-observer effect
Fundamental attribution error
Perceptual salience
Downward social comparison
113- Making predictions about future behavior is dependent on having accurate explanations of the
behavior of interest.
True False
114- Terrorists acts exemplify impulsive aggression.
True False
115- Studying language by listening to people in public places is example of ---------- research.
Correlational
behaviorist The psychological perspective primarily concerned with observable behavior that can be
objectively recorded and with the relationships of observable behavior to environmental stimuli.
Observational The process of learning new responses by watching the behavior of another.
experimental
116- According to research in social psychology, why do many people believe that their horoscopes
are accurate descriptions of who they are and what is likely to happen to them?
Horoscopes are written in a vague way so that most people view them as representative of their
personalities and past behaviors.
Horoscopes trigger automatic decision making
People find it difficult to bring to mind examples that are similar to the horoscope
Horoscopes automatically prime people’s life goals.
117- Under variable interval schedule, reinforcement will be delivered after a specified average
number of responses have been made:
true False
118- The high school class reunion you are likely to experience a flood of memories that would
unlikely to come to mind under other circumstances. What memory process explains this?
Implicit memory
anterograde amnesia
encoding specificity
the TOT phenomenon
retrograde amnesia
119- health constituted the freedom from disease, pain, or defect.
Introduction to Psychology
Total 25 points

Fill in the gaps questions


Fill in the gaps.
(1 point) Four main goals of psychologists are describing, explaining, _ _ _ _ _ _ predicting _ _ _
_ _ _ _ _ _ _ _ _ _ and controlling behavior.

(1 point) When different people observe the same events, because of their personal motives or
expectations, they don’t always “see” the same thing. What can be a remedy / solution for this
challenge to objectivity? _ _ _ _ _ _ _ _ _ _ _ _ _ Operational procedures_ _ _ _ _ _ _ _ _ _ _ _ _ _
_________________

(1 point) Parents use various reinforcement schedules to teach children new behavior,
strengthen the desired behavior or try to stop undesired behavior. A continuous schedule is
often the best in teaching a new behavior. Once the response has been learned, _ _ _ _ _ _
positive reinforcement _ _ _ _ _ _ _ _ _ _ _ _ _ _ _ _ _ _ _ _ _ can be used to strengthen the new
behavior.

(1 point) After listening to a list of words such as hot, snow, warm, winter, ice, wet, frigid, chilly,
heat, weather, freeze, air, shiver, Arctic, and frost, people often remember having heard the
related word cold even though it was not presented (Gallo, 2006; Roediger & McDermott,
1995).
This happens due to _ _ _ _ _ _ _ _ _ _ _ _ _ _ _ _ _ _ _ _ _ _ _ _ _ _ _ _ _ _ _ _ _ _ _ _ _ _ _ _ _
__

True or False questions


Read each statement and indicate whether they are correct by underlining either True or False
(1 point) Each perspective of psychology (the humanistic, cognitive, sociocultural,
evolutionary, humanistic, behaviorist perspectives) stress the biological, psychological, or
social-cultural factors more than another, each of the perspectives are interesting and valuable
by themselves but unfortunately for the science of psychology, the different perspectives
contradict one another, they’re impossible to use in combination and you have to choose one
of them that you agree with more.
True or false? (not sure)

(1 point) In a Carr and VanDeusen (2004) study, male college students at a large midwestern
university completed an anonymous survey on which they indicated whether they had ever
engaged in sexually coercive behavior as well as the frequency with which they viewed various
forms of pornography. The researchers found a statistically significant correlation, such that
the more pornography the students reported using, the greater the likelihood that they had
committed sexual violence (Hald, Malamuth, & Yuen, 2010).
From this study we can draw the conclusion that using pornography made the male students
more likely to commit sexual violence.
True or false?
(1 point) Every time Ana posts half-naked photographs of herself, she gets lots of attention,
likes and comments on social media. Now uploading half-naked photos and receiving
attention became strongly associated for her, so the number of such posts on her page is
steadily increasing. This is an example of classical conditioning.
True or false?

(1 point) The difference between flashbulb and other types of memories is that flashbulb
memories are accurate because processes of reconstruction do not apply to them.
True or false?

Multiple-choice questions
Circle the correct answer.
(1 point) “Boys are more physically aggressive than girls because a male hormone testosterone
is linked with aggression.” This statement is likely made by a psychologist working in _ _ _
perspective of psychology.
A. the biological perspective
B. the evolutionary perspective
C. the psychodynamic perspective
D. the behaviorist perspective
(1 point) “In this study, Japanese students refrained themselves from expressing as anger and
disgust because in Japanese culture it’s not that desirable to show negative emotions.” .” This
statement is likely made by a psychologist working in _ _ _ perspective of psychology.
A. the humanistic perspective
B. the cognitive perspective
C. the sociocultural perspective
D. the evolutionary perspective
(1 point) “Our beliefs about ourselves matter. Do we see ourselves as in control of things? Do
we view ourselves as relatively superior or inferior? Our answers influence our emotions and
actions.” This statement is likely made by a psychologist working in _ _ _ perspective of
psychology.
A. the humanistic perspective
B. the cognitive perspective
C. the sociocultural perspective
D. the behaviorist perspective

(1 point) To test the effect of a new drug on depression, a researcher randomly assigns people
to control and experimental groups. Those in the experimental group take a pink pill
containing the new medication; those in the control group take a pink pill that contains no
medication. Which statement is true?
The medication is the dependent variable.
Depression is the independent variable.
Participants in the control group take a placebo.
Participants in the experimental group take a placebo.

(1 point) A researcher wants to test the hypothesis that people are less likely to help a person
having a seizure in the street if there are some other people witnessing this situation than if
they’re alone. In the experiment he plans, the researcher will vary the number of people
witnessing the situation where an actor (an assistant of the experimenter) acts as if he’s having
a seizure. The number of people will be his
placebo control.
double-blind control.
independent variable.
dependent variable.

(1 point) A double-blind procedure is often used to prevent researchers’ biases from


influencing the outcome of an experiment. In this procedure,
only the participants know whether they are in the control group or the experimental group.
experimental and control group members will be carefully matched for age, sex, income, and
education level.
neither the participants nor the researchers know who is in the experimental group or control
group.
someone separate from the researcher will ask people to volunteer for the experimental group or
the control group.

(1 point) Ana started working for a pet food store. In the beginning she was annoyed by the
smell in the store but after some period of time she got used to it. What is the technical term
for “getting used to bad smell” to the extent that it doesn’t bother Ana, and that she doesn’t
react to it anymore?
A. Sensitization.
B. Stimulus discrimination.
C. Habituation.
D. Response acquisition.
(1 point) When playing with a slot machine players have no way of knowing how many times
they have to play before they win. All they know is that eventually, a play will win. This is why
slot machines are so effective, and players are often reluctant to quit. There is always the
possibility that the next coin they put in will be the winning one.
What reinforcement contingency do they psychologists use to design slot machines?
A. Variable-Ratio Schedule
B. Fixed-Ratio Schedule
C. Fixed-Interval Schedule
D. Intermittent Reinforcement Schedule

(1 point) When a toddler throws a tantrum in the store, parents usually refuse to give in. But
once in a while, when they’re tired or in a hurry, they decide to buy the candy, thinking they
will do it just that one time. This is an example that in daily life, parents sometimes
unknowingly _ _ undesired behavior.
A. reinforce
B. extinct
C. punish
D. discriminate

(1 point) Students usually remember what was they read in the beginning and the end of a
book chapter and they don’t remember the middle sections quite well. This is due to
A. reconstructive nature of memory.
B. the serial position effect.
C. semantic encoding principle.
D. encoding specificity effect.

(1 point)Tom waits remembers he was born on 7th December, 1949. This is his
A. semantic memory
B. procedural memory
C. episodic memory
D. flashbulb memory

(1 point) Suppose you’re on your way home from watching a scary movie about a hitchhiker
who was a murderer when you see someone talking loudly with a friend. Because you saw the
movie, you assume that you are witnessing an argument that will probably end in a fight. This
is an example of
A. priming.
B. meta-memory.
C. context effects.(not sure)
D. spontaneous recovery.
Introduction to Psychology Final Exam Test
This test involves questions based on all chapters included in the syllabus (7 fill in the gaps
questions, 7 true/false questions, 21 multiple-choice questions).
Total 35 points

Fill in the gaps questions


Fill in the gaps.
(1 point)
_ fundamental attribution error_ _ _ _ _ _ _ _ _ _ _ _ _ _ _ _ _ _ _ _ _ _ _ _ _ _ _ _ _ _ _ _ _
represents the dual tendency for people to overestimate dispositional factors (blame or
credit people) and to underestimate situational factors (blame or credit the environment)
when searching for the cause of some behavior or outcome.

(1 point)
When people doubt their ability to perform a task, they may engage in _self-handicapping _
_ _ _ _ _ _ _ _ _ _ _ _ _ _ _ _ _ _ _ _ _ _ _ _ _ _ _ _ _ _ _ _ — They deliberately sabotage
their performance. The purpose of this strategy is to have a ready-made excuse for failure
that does not imply lack of ability.

(1 point)
George L. Engel’s _ _ _ _ biopsychosocial model of health and illness._ _ _ _ _ _ _ _ _ _ _ _ _
_ _ _ _ _ _ _ _ _ _ _ _ _ _ _ _ _ states that the causes, manifestations, and outcomes of
wellness and diseases are determined by the interactions between biological, psychological,
and social factors.

(1 point)
The same situations might cause stress in some people but not in others. According to
Lazarus, to experience some situations as stressful, first step is to evaluate the situations as
stressful. _ _ _ _ _ _ _ _ _ _ _ _ _ _ _ _ _ _ _ cognitive appraisal_ _ _ _ _ _ _ _ _ _ _ _ _ is the
cognitive interpretation and evaluation of a stressor.

(1 point)
According to Schachter, the experience of emotion is the joint effect of the two factors: _ _
_ _ _ _ _ _ _ _ _ _ psychological arousal_ _ _ _ _ _ _ _ _ _ _ _ _ _ _ _ _ _ _ _ _ and cognitive
appraisal.

(1 point)
_ _ _ _ _ _ _ _ _ _ gender streotypes /gender role_ _ _ _ _ _ _ _ _ _ _ _ _ _ _ _ _ _ _ _ _ _ are
beliefs about attributes and behaviors regarded as appropriate for males and females in a
particular culture.
(1 point)
_ _ _ _ _ _ _ _ _ _ _ _ _ _ _ _ crystallized intelligence_ _ _ _ _ _ _ _ _ _ _ _ _ _ _ _ _ _ involves
the knowledge a person has already acquired and the ability to access that knowledge; it is
measured by tests of vocabulary, arithmetic, and general information.

True or False questions


Read each statement and indicate whether they are correct by underlining either True or False.
(1 point)
According to Piaget, assimilation restructures or modifies the child’s existing schemes so
that new information is accounted for more completely.
True or False?

(1 point)
Gardner’s Multiple Intelligences theory suggests that whether people people have
mathematical and linguistic intelligence or not is determined by the heritability estimate.
True or False?

(1 point)
Retrieval cues are the stimuli available as you search for a particular memory.
True or False?

(1 point)
When a behavior is followed by the removal of an aversive stimulus, the event is called
negative reinforcement.
True or False?

(1 point)
Reflex is a learned response elicited by specific stimuli that have biological relevance for an
organism.
True or False?

(1 point)
Groups make decisions that are riskier and less conservative than would be made by the
members acting alone.
True or False?

(1 point)
A correlation coefficient of -0.7 indicates a weaker relationship between two variables than
a correlation coefficient of 0.4.
True or False?
Multiple-choice questions
Circle the correct answer.
(1 point)
People tend to take credit for their successes while denying or explaining away
responsibility for their failures. In many situations, people tend to make dispositional
attributions for success and situational attributions for failure. “I got the prize because of
my ability”; “I lost the competition because it was rigged.” These are examples of
self-fulfilling prophecy.
confirmation bias.
self-serving bias.
covariation bias.

(1 point)
A teacher believes boys are inherently better at math, so he doesn’t spend equally enough
time and energy to helping female and male students. As a result, his male students get
higher scores on final math exam than girls. This difference between the final performance
levels between girls and boys in his class occurred because of the teacher’s
social role.
informational influence.
self-fulfilling prophecy.
self-serving bias.

(1 point)
When a group makes a decision there’s a tendency to filter out undesirable input so that a
consensus may be reached, especially if it is in line with the group leader’s viewpoint. This
tendency is referred to as
E. group conformity.
F. group polarization.
G. groupthink.
H. group influence.

(1 point)
When examining a patient’s problem every doctor in a team came to the same conclusion
except one of them but even though he had a different opinion he decided not to voice his
different view so that he wouldn’t feel fool and deviant from others. This is an example of
E. conformity.
F. social cognition.
G. obedience.
H. social role.

(1 point)
According to Rogers, self-actualization is
E. the use of psychological theory to explain an individual’s course through life.
F. a person’s mental model of his or her typical behaviors and unique qualities.
G. a constant striving to realize one’s inherent potential.
H. complete love and acceptance of an individual by another person.
(1 point)
The values for the Independent Construal of the Self in individualistic cultures (such as the
USA) are:
E. connectedness with others, group harmony and social relations.
F. personal freedom, self-fulfillment, independence, self-reliance.
G. loyalty to one’s group, duty to one’s family, adherence to social traditions.
H. enhancement of one’s groups’ status and well-being.

(1 point)
Some people believe that events in their life derive primarily from their own actions. For
example, when receiving exam results, they tend to praise or blame themselves and their
abilities, not the teacher or the exam test. These people are said to have
E. internal locus of control.
F. acquired introversion.
G. external locus of control.
H. acquired extroversion.

(1 point)
Self-concept refers to
E. the idea that people observe themselves to figure out the reasons they act as they do.
F. a person’s mental model of his or her typical behaviors and unique qualities.
G. the set of beliefs that one can perform adequately in a particular situation.
H. a generalized evaluative attitude toward the self that influences both moods and behavior.

(1 point)
Which of the following is not a source of self-efficacy judgments?
E. Vicarious experience (your observations of the performance of others).
F. Persuasion (others may convince you person that you can do something, or you may
convince yourself).
G. Monitoring of your emotional arousal as you think about or approach a task (e.g. anxiety
suggests low expectations of efficacy; excitement suggests expectations of success).
H. Self-handicapping (you anticipate failure and you deliberately sabotage your
performance so that you can have a ready-made excuse for failure).

(1 point)
Which of the following is not correct?
E. Wellness is a general condition of soundness and vigor of body and mind; not simply the
absence of illness or injury.
F. Health promotion is the development and implementation of general strategies and
specific tactics to eliminate or reduce the risk that people will become ill.
G. Health is a general condition of soundness and vigor of body and mind; not simply the
absence of illness or injury.
H. Wellness is optimal health, incorporating the ability to function fully and actively over
the physical, intellectual, emotional, spiritual, social, and environmental domains of
health..

(1 point)
According to the Transtheoretical Model (Stages of Change), when changing their
unhealthy behaviors to healthy ones, individuals move through six stages of change:

E. precontemplation, contemplation, preparation, action, maintenance, and termination.


F. action, termination, precontemplation, contemplation, preparation and maintenance.
G. preparation, precontemplation, contemplation, action, maintenance, and termination.
H. contemplation, precontemplation, action, maintenance, preparation, and termination.

(1 point)
Failing to adhere to treatment regimens that doctors give to their patients is one of the most
serious problems in health care. Which of the following statements about patient adherence
is correct?
Generally, patients who perceive greater threat from a disease also show greater likelihood to
adhere to treatment regimen given by a doctor.
Patients who face serious diseases that leave them in poor physical health always show higher
levels of adherence than patients who are less debilitated by the same diseases.
Physicians should stop trying to understand their patients’ attitudes and modify their behaviors to
match those attitudes.
When patients can rely on social support that allows them to accomplish their regimens correctly
they are less likely to adhere to treatments.

(1 point)
Out of the seven strategies listed below, which ones does Problem-Directed Stress Coping
involve?
1. Fight (destroy, remove, or weaken the threat)
2. Somatically focused activities (use of antianxiety medication, relaxation,
biofeedback)
3. Flight (distance oneself from the threat)
4. Cognitively focused activities (planned distractions, fantasies, thoughts about
oneself)
5. Seek options to fight-or-flight (negotiating, bargaining, compromising)
6. Therapy to adjust conscious or unconscious processes that lead to additional anxiety
7. Prevent future stress (act to increase one’s resistance or decrease strength of
anticipated stress)

A. 1, 2, 3, 6
B. 2, 4, 3, 7
C. 1, 3, 5, 7
D. 2, 3, 5, 6

(1 point)
Traditionally psychologists assumed that subjective mental experience of an emotion
preceded the bodily expression or action. However, according to the James-Lange theory of
emotion, the arousal of the physical response precedes the appearance of the emotion
(perceiving a stimulus causes autonomic arousal and other bodily actions that lead to the
experience of a specific emotion). Which of the following would not serve as an appropriate
example of James’ and Lange’s understanding of how we experience emotions?

A. I see a wild animal, I feel fear, and I run away.


B. I see a wild animal, I run, and then I experience the emotion of fear.
C. I cry because I feel sorry.
D. I’m afraid because I tremble.

(1 point)
In his experiments Robert Zajonc demonstrated mere exposure effect which describes how
sometimes people have preferences without knowing why. The mere exposure effect means
that
A. repeated exposure to a stimulus (such as a song, a person, a painting) increases liking of
that stimulus.
B. repeated exposure to a stimulus (such as a song, a person, a painting) makes a perceiver
feel bored of that stimulus.
C. repeated exposure to a stimulus (such a song, a person, a painting) increases
perceivability of the details that went unnoticed before.
D. repeated exposure to a stimulus (such a song, a person, a painting) decreases willing to
pay attention to more details.

(1 point)
According to the Kohlberg’s theory of moral development, on the last stage of moral
reasoning a person’s intention to behave morally is
A. To avoid pain or not to get caught
B. To gain acceptance and avoid disapproval
C. To be true to universal principles and feel oneself part of a cosmic direction that
transcends social norms
D. To promote the society’s welfare

(1 point)
Children at the preoperational stage (roughly from 2 to 7 years of age) are unable to take
the perspective of another person. For example, a child talking on the phone might
pinpoint to a toy that she/he is talking about with the other person on the phone, she can’t
take into account that the person on the phone can’t see what she/he is pinpointing with
his/her fingers to.
In Piaget’s theory, this inability is referred to as
A. centration.
B. conservation.
C. egocentrism.
D. assimilation.

(1 point)
As children’s cognitive abilities develop, they come to understand that other people have
cognitive experiences of the world—and that those cognitive experiences may not be exactly
the same. Children become able to explain and predict other people’s behavior based on an
understanding of their mental states. In other words, children develop
A. expectancy theory.
B. theory of mind.
C. equity theory.
D. theory of individual psychology.

(1 point)
According to Gardner’s Multiple Intelligences theory, some people have great ability to
plan and understand sequences of movements and they’re more likely to become great
dancer or athletes. This is to say that these people have high
A. bodily-kinesthetic intelligence.
B. existential intelligence.
C. spatial intelligence.
D. naturalist intelligence.

(1 point) Which of the following abilities does emotional intelligence involve?


1. The ability to perceive, appraise, and express emotions accurately and appropriately
2. The ability to use emotions to facilitate thinking, to understand and analyze
emotions
3. The ability to use emotional knowledge effectively
4. The ability to regulate one’s emotions to promote both emotional and intellectual
growth

A. 1, 2, 3
B. 1, 3, 4
C. 1, 2, 3, 4
D. 2, 3, 4

(1 point)
A procedure conducted at the end of an experiment in which the researcher provides the
participant with as much information about the study as possible and makes sure that no
participant leaves feeling confused, upset, or embarrassed is called
A. informed consent.
B. cover story.
C. debriefing.
D. intentional deception.

(1 point)
Which one of the following statements about memory is correct?
A. Semantic memories preserve the specific events that you have personally experienced.
B. Recognition is a method of retrieval in which an individual is required to reproduce the
information to which he/she was previously exposed.
C. When individuals are motivated enough to remember information accurately their
memories can be absolutely accurate.
D. No matter how emotionally important a memory or information can be for an individual,
memories are can always be subject to misinformation effect.
Psychology Final

Quiz 1:
In psychology behavior is the means by which organisms adjust to their environment.
This method uses objectively collected information as the factual basis for drawing conclusions. It
consists of a set of orderly steps used to analyze and solve problems. This method is called
scientific method
Main goals of psychology are
Describing, explaining, predicting, controlling behavior and mental processes
Provide an example of controlling a behavior.
Match the perspectives of psychology with their sample questions

Match the perspectives in psychology with their focus


With respect to the goals of psychology, why is it appropriate to characterize psychologists as
“rather optimistic”?
Why is it often good to consider the same research question from several of psychology’s seven
perspectives?

Quiz 2:

Which of the following sequences of stages reflects the process of conducting and reporting
psychological research?

Consider open questions → act on open questions → design the study → form a hypothesis →
Initial observation or question → analyze the data and draw conclusions → report the findings
Initial observation or question → form a hypothesis → design the study → analyze the data and
draw conclusions → report the findings → consider open questions → act on open questions
Form a hypothesis → consider open questions → act on open questions → Initial observation or
question → design the study → analyze the data and draw conclusions → report the findings
Design the study → form a hypothesis → consider open questions → act on open questions Initial
observation or question → analyze the data and draw conclusions → report the findings

In some research designs, which are referred to as between-subject designs


designs, different groups of participants are randomly assigned, by chance procedures, to an
experimental condition (exposed to one or more experimental treatments) or to a control condition
(not exposed to an experimental treatment).

Random assignment, Randomization is one of the major steps researchers take to eliminate
confounding variables that relate to individual differences among potential research participants.
This procedure makes it quite likely that the two groups will be similar in important ways at the start
of an experiment because each participant has the same probability of being in a treatment
condition as in a control condition. We shouldn’t have to worry, for example, that in an experiment
of the influence of watching violent movies on the level of aggression everyone in the experimental
group loves violent television and everyone in the control group hates it.

In a Carr and VanDeusen (2004) study, male college students at a large Midwestern university
completed an anonymous survey on which they indicated whether they had ever engaged in
sexually coercive behavior as well as the frequency with which they viewed various forms of
pornography. The researchers found a statistically significant correlation, such that the more
pornography the students reported using, the greater the likelihood that they had committed sexual
violence (Hald, Malamuth, & Yuen, 2010). Can we draw the conclusion that using pornography
made the male students more likely to commit sexual violence? Explain your answer.

Any factor that affects the dependent variable along with or instead of the independent variable is
called Confounding
When different people observe the same events, because of their personal motives or
expectations, they don’t always “see” the same thing. What can be a remedy for this challenge to
objectivity? / What is the solution to minimize observer biases? Operationalization, Operationalize
variables

How would you conduct an experiment to test the hypothesis that it’s easier to remember words
written in the Sylfaen font than in Times New Roman font?

What will be the experimental design here? (Referring to the experiment described in the 7th
question)

Depends on ur example, could be within subjects or etc


What will be the independent variable? (Referring to the experiment described in the 7th
question)
Font type

What will be the dependent variable? (Referring to the experiment described in the 7th question)
The numbers of words recalled

What will be the control variables? (Referring to the experiment described in the 7th question)
1- Color and size of text 2- Age and of sample. 3-Length and complexity of each quote 4-Time they
are given to read and analyze each quote.

Can we make a causal conclusion here? (That one font or the other leads to better remembering.
Explain your answer shortly)
Suppose a researcher found a strong negative correlation between college students’ GPA (grade
point average) and the amount of alcohol they drink. Which of the following is the best conclusion
from this study?
I. Students with a high GPA study more and thus have less time to drink.
J. Drinking a lot interferes with studying.
K. If you know how much alcohol a student drinks, you can predict his or her GPA fairly well.
L. The higher a student’s GPA, the more he or she drinks.
M. People who are intelligent get higher grades and drink less.

Descriptive (Observational) and correlational studies describe behavior, detect relationships, and
predict behavior. But to explain behaviors, psychologists use
I. naturalistic observation.
J. experiments.
K. surveys.
L. case studies.

A researcher wants to determine whether noise level affects the blood pressure of elderly people.
In one group she varies the level of noise in the environment and records participants’ blood
pressure. In this experiment, the level of noise is the
I. control condition.
J. placebo.
K. dependent variable.
L. independent variable.

The predictions implied by a theory are called


operational definitions.
correlations.
hypotheses.
replications.

You wish to take an accurate poll in a certain country by questioning people who truly represent
the country’s adult population. Therefore, you need to ensure that you question
I. at least 50 percent males and 50 percent females.
J. a small but intelligent sample of the population.
K. a very large sample of the population.
L. a random sample of the population.

A study finds that the more childbirth training classes women attend, the less pain medication they
require during childbirth. This finding can be stated as
I. positive correlation.
J. negative correlation.
K. cause -effect relationship.
L. illusory correlation.

Knowing that two events are correlated provides


I. a basis for prediction.
J. an explanation of why the events are related.
K. proof that as one increases, the other also increases.
L. an indication that an underlying third factor is at work.

A newspaper article describes how a “cure for cancer has been found.” A critical thinker probably
will
I. dismiss the article as untrue.
J. accept the information as a wonderful breakthrough.
K. question the article, evaluate the evidence, and assess the conclusions.
L. question the article but quickly accept it as true if the author has an excellent reputation.

To test the effect of a new drug on depression, we randomly assign people to control and
experimental groups. Those in the experimental group take a pink pill containing the new
medication; those in the control group take a pink pill that contains no medication. Which statement
is true?
The medication is the dependent variable.
Depression is the independent variable.
Participants in the control group take a placebo.
Participants in the experimental group take a placebo.

A double-blind procedure is often used to prevent researchers’ biases from influencing the
outcome of an experiment. In this procedure
I. only the participants know whether they are in the control group or the experimental group.
J. experimental and control group members will be carefully matched for age, sex, income, and
education level.
K. neither the participants nor the researchers know who is in the experimental group or
control group.
L. someone separate from the researcher will ask people to volunteer for the experimental
group or the control group.
Quiz 3:
Provide at least 2 reasons for justifying using animals for research.

My answer is written on small notebook

2.Provide at least 2 reasons against using animals for research.

My answer is written on small notebook

3.Define “Informed Consent” is a process in which the person who is undergoing a certain
procedure in informed of the risks, side effects, and benefits of

4.If a psychological measure gives approximately the same evaluation of a personality trait on
different occasions, it is said to have reliability
5.If a psychological measure intends to measure happiness and if using this measure (e.g.
questionnaire) allows us to predict how happy someone is likely to be in particular situations, this
questionnaire has Validity
6.To understand the effects of a brain damage, researchers examined every aspect of brain
functioning of a person after a serious car accident. I.e. they conducted a case study

7.Provide two examples of self-report measure 1) questionnaire 2) an interview 3)survey

8.Provide an example when behavioral measure is used instead We use behavioural measures
when we only want to observe the action of another person. for example, a mother watching her
kids play

9.In the context of psychological measurement "reliable" means… consistent or stable. We use
this degree when we get the same result everytime we are doing the test.

Quiz 4:
E. Recall one example of advertisement you've seen / heard where they used classical
conditioning principles.
F. Among other reasons for engaging in unprotected sex, some men report the feeling of
discomfort while wearing a condom as the cause why they don't use protection (e.g.
Yarber et al., 2005). How can condom use (or refusing to use a condom) be related to
habituation?

G. In Stanley Kubrick’s movie Clockword Orange the mainc character is an adolescent prone
to extreme violence (rape, murder, assault etc.). When he gets in prison , to “cure” him,
each day they give him a drug which causes nausea (but he’s not aware of the fact, he
just thinks those are vitamins). They force him watch extremely violent movies including
rapes and murders. While watching the movies he starts feeling nauseous because of the
drug. After a few weeks he forms an association between violence and feeling the ill.
When tested in different situations causing being violent before, he doesn't manifest
violence but feeling sick (nauseous) instead. In this situation, what is the unconditioned
stimulus? Drugs
H. Name the unconditioned response. (Referring to the situation described in the question
#4)
Feeling nausea as a result of drugs
I. Name the conditioned stimulus. (Referring to the situation described in the question #4)
Watching violent movies
J. Name the conditioned response. (Referring to the situation described in the question #4)
Feeling nausea as a result of watching violence

Quiz 5:

E. Every time Ana posts half-naked photographs of herself, she gets lots of attention, likes and
comments on social media. Now uploading half-naked photos and receiving attention became
strongly associated for her, so the number of such posts on her page is steadily increasing. This
is an example of classical conditioning.

E. True
F. False
F. When a behavior is followed by the removal of an aversive stimulus, the event is called negative
reinforcement.
E. True
F. False

G. Parents use various reinforcement schedules to teach children new behavior, strengthen the
desired behavior or try to stop undesired behavior. A continuous schedule is often the best in
teaching a new behavior. Once the response has been learned_partial reinforcement _can be
used to strengthen the new behavior.

H. When playing with a slot machine players have no way of knowing how many times they have to
play before they win. All they know is that eventually, a play will win. This is why slot machines
are so effective, and players are often reluctant to quit. There is always the possibility that the
next coin they put in will be the winning one. What reinforcement contingency do they
psychologists use to design slot machines?
E. Variable-Ratio Schedule
F. Fixed-Ratio Schedule
G. Fixed-Interval Schedule
H. Intermittent Reinforcement Schedule

I. When a toddler throws a tantrum in the store, parents usually refuse to give in. But once in a
while, when they’re tired or in a hurry, they decide to buy the candy, thinking they will do it just
that one time. This is an example that in daily life, parents sometimes unknowingly _ _
undesired behavior.
E. negatively reinforce
F. extinct
G. positively punish
H. discriminate
I. negatively punish
J. positively reinforce
J. Ana started working for a pet food store. In the beginning she was annoyed by the smell in the
store but after some period of time she got used to it. What is the technical term for “getting
used to bad smell” to the extent that it doesn’t bother Ana, and that she doesn’t react to it
anymore?
5. Sensitization
6. Stimulus discrimination
7. Habituation
8. Response acquisition

Quiz 6:

E. The extent to which a particular item stands out from or is distinct from other items in time is
referred to as Temporal distinctiveness

F. Improved memory for items at the end of a list is referred to as Recency Effect

G. 4.The principle that subsequent retrieval of information is enhanced if cues received at the time
of recall are consistent with those present at the time of encoding is referred to as Encoding
specificity

H. Generic, categorical memory, such as the meaning of words and concepts is referred to as
Semantic Memory

I. Long-term memory for an autobiographical event and the context in which it occurred is
referred to as Episodic memory

J. A method of retrieval in which an individual is required to reproduce the information previously


presented is called recall

K. Internally or externally generated stimulus available to help with the retrieval of a memory. This
is a definition of retrieval cue

L. 10.People’s vivid and richly detailed memory in response to personal or public events that have
great emotional significance are called flashbulb memories
M. Memory processes associated with the preservation of information for retrieval at any later
time. This is a definition of Long term memory (LTM)

N. The process of putting information together based on general types of stored knowledge in the
absence of a specific memory representation is called Reconstructive memory

O. A method of retrieval in which an individual is required to identify stimuli as having been


experienced before is referred to as) recognition
Quiz 7:

1. Your parents ask you if you can think of 12 reasons why your university is better than its main rival
universities. You find it hard to come up with so many reasons and so end up thinking, “Hmm, maybe
the universities aren’t all that different.” Which mental strategies did you probably use to reach this
conclusion? available heuristic

2. George is definitely not the most attractive guy at the university, but he is extremely confident about
who he is and how he looks. He is convinced that most women find him to be very attractive, and he in
fact usually gets dates with girls who are much more attractive than he is. What is the best explanation
of George’s success? self-fulfilling prophecy

3. “Simple rules for making complex decisions or drawing inferences in a rapid manner and seemingly
effortless manner.” “A thinking strategy that enables quick, efficient judgments.” These are definitions
of heuristic
4. .Ana thinks religious people are more moral than atheists. After searching the internet, she only read
the articles that had titles which indicated that morality of of religious people is superior to the morality
of people who don’t believe in god. This is an example of confirmation bias

5. .Nine in 10 students rate a condom as effective if it has a supposed “95 percent success rate” in
stopping the HIV virus that causes AIDS; only 4 in 10 think it successful given a “5 percent failure rate”
(Linville et al., 1992). This is due to framing_ effect.

6.. According to research in social psychology, why do many people believe that their horoscopes are
accurate descriptions of who they are and what is likely to happen to them?
E. Horoscopes are written in a vague way so that most people view them as representative of their
personalities and past behaviors.
F. Horoscopes trigger automatic decision making.
G. People find it difficult to bring to mind examples that are similar to the horoscope.
H. Horoscopes automatically prime people’s life goals.

K. Suppose you wanted to make your friend Ana to feel like a more assertive person. According to
research on ______, you should ask her to think of _____ times in the past when she acted in
an unassertive manner.

E. Representativeness heuristic; 12
F. Availability heuristic; 3
G. Representativeness heuristic; 3
H. Availability heuristic; 12

L. All of the following are good examples of the self-fulfilling prophecy except one. Which one?
8. A teacher believes that boys are better at math than girls, and boys in his class do better than
girls in math.
9. Ana thinks that members of the Alpha-Beta team members are unfriendly and snobby.
Whenever he meets members of this team, they are unfriendly toward him.
10. Ana thinks her dog isn’t very good at learning tricks. Her dog knows fewer tricks than most dogs.
11. Ana is worried that her son is not gifted in music, but he does better at his piano lessons than
she expected.

Quiz 8:

“I didn’t leave as much tip to the waiter as others from the group did and now I am perceived as stingy
by them. I just didn’t have more amount of money. It’s terrible when your actions are judged when
those actions don’t reflect your character or personality. But they won’t even bother find out more
about me” – The persons complaints come from getting her behavior judged based on _ _
_ .Fudemental attribution error (FAE)

3.In contrast to most people, depressed individuals tend to attribute negative outcomes to lasting,
internal causes such as their own traits or lack of ability, but attribute positive outcomes to temporary,
external causes such as good luck or special favors from others. This suggests that _ _ _ may play a role
in developing/protecting from depression..
self-serving bias

4.Even when people knew that the author’s choice of an essay topic was externally caused (i.e., in the
no-choice condition), they assumed that what he wrote reflected how he really felt about Castro. That
is, they made a mistake based on self-serving attributional bias.
True
False
5.Self-serving attributional bias is the tendency to believe that people’s behavior corresponds to
(matches) their dispositions.
True
False

6.People make self-serving attributions which means they have the tendency to believe that a behavior
is due to a person’s disposition, even when there are situational forces present that are sufficient to
explain the behavior.
True
False

7.“If men define situations as real, they are real in their consequences.” The knowledge of self-fulfilling
prophecy is relevant for understanding this quote from W.I. Thomas.
True
False

8.“Whether you think you can or you think you can’t, you’re right.” The knowledge of self-fulfilling
prophecy is relevant for understanding this quote from Henry Ford.
True
False

9.If a man begins dating a man whom he feels strongly connected to, he may feel that this person is
“the one.” Since he expects the relationship to last, he treats his partner with love and respect; as a
consequence the relationship ends up fulfilling. This exemplifies self-fulfilling prophecy
True
False

10.Couples in unhappy marriages tend to make _ _ _ attributions for each other’s positive actions (“He
just wanted to look good in front of our guests”), and _ _ _ attributions for negative actions (“He just
doesn’t care”).
self-serving | external
self-serving | internal
internal | external
externa l | internal

11.Couples in happy marriages tend to make _ _ _ attributions for each other’s positive actions (“That’s
just the way he is”) and _ _ _ attributions for negative actions (“He’s just so busy providing for all of us
that he doesn’t have the time”).
self-serving | external
self-serving | internal
internal | external
external | internal

12.Victory finds a hundred fathers but defeat is an orphan.” (COUNT GALEAZZO CIANO, THE CIANO
DIARIES, 1938) Match the phrase of a famous author with the corresponding social psychological
notion.
Self-serving bias
The need of causal attribution
Social comparison theory
Self-perception theory

13."In the beginning was not the word, not the deed, not the silly serpent. In the beginning was why?
Why did she pluck the apple? Was she bored? Was she inquisitive? Was she paid? Did Adam put her up
to it? If not, who did?” (JOHN LE CARRÉ, THE RUSSIA HOUSE, 1989) Match the phrase of a famous
author with the corresponding social psychological notion.
Self-serving bias
The need of causal attribution
Social comparison theory
Self-perception theory

14.If we encounter a person who appears to be high on drugs, and we make the fundamental
attribution error, we will probably attribute the person’s behavior to
Moral weakness or an addictive personality.
Peer pressure.
The easy availability of drugs on city streets.
Society’s acceptance of drug use.

15.In the research carried out by Robert Rosenthal and Lenore Jacobson, what caused the performance
of some students to improve dramatically?
Teachers were led to expect such improvement and so changed the way they treated these students.
These students performed exceptionally well on a special test designed to predict improved
performance.
Teachers gave these students higher grades because they knew the researchers were expecting the
improvement.
The students felt honored to be included in the experiment and, therefore, were motivated to improve.

16.People tend to take credit for their successes while denying or explaining away responsibility for
their failures. In many situations, people tend to make dispositional attributions for success and
situational attributions for failure. “I got the prize because of my ability”; “I lost the competition
because it was rigged.” These are examples of
self-fulfilling prophecy.
confirmation bias.
self-serving bias.
covariation bias.

17.A teacher believes boys are inherently better at math, so he doesn’t spend equally enough time and
energy to helping female and male students. As a result, his male students get higher scores on final
math exam than girls. This difference between the final performance levels between girls and boys in
his class occurred because of the teacher’s
social role.
informational influence.
self-fulfilling prophecy.
self-serving bias.

Quiz 9:

Which of the following statements is correct?

Stereotype is an unjustifiable negative attitude toward a group and its members, while unjustifiable
negative feeling towards them is prejudice.
Discrimination is an unjustifiable negative attitude toward a group and its members, while unjustifiable
negative behavior towards them is prejudice.
Prejudice is an unjustifiable negative attitude toward a group and its members, while unjustifiable
negative behavior towards them is discrimination.
Both stereotype and prejudice stand for unjustifiable negative attitudes toward a group and its
members while unjustifiable negative behavior towards them is discrimination.
2.1. prejudice is a(n) _ _ _ _ , while discrimination is a(n) _ _ _ _ ,

behavior/attitude
attitude/behavior
instinct/choice
stimulus/response
3.Most wars and te;rrorist acts are _ _ _ aggression.

Hostile
Instrumental
Indirect
Passive
4.What is stereotype threat?
Feeling threatened by prejudices we wish we didn’t have
Feeling threatened by stereotypes we hold about other people
Feeling threatened by people who confirm our stereotypes
Feeling threatened by stereotypes that others hold of our group

5.Our biology (our genes, neural systems, and biochemistry— including testosterone and alcohol levels)
completely determines our tendencies to be aggressive.
True
False

6.Our biology (our genes, neural systems, and biochemistry— including testosterone and alcohol levels)
influence our tendencies to be aggressive.
(1/1 Points)
True
False
7.Social influences, such as exposure to violent media, and cultural influences, such as whether we’ve
grown up in a “culture of honor” or had a father-absent home, can affect our aggressive responses.

True
False

8.In emergency situations, the larger the number of bystanders, the higher the probability of helping.
True
False

Quiz 10:

2. Selye’s general adaptation syndrome (GAS) consists of an alarm reaction followed by

fight or flight.
resistance then exhaustion.
challenge then recovery.
stressful life events.
3.The number of short - term illnesses and stress - related psychological disorders was higher than usual
in the months following an earthquake. Such findings suggest that

daily hassles have adverse health consequences.


experiencing a very stressful event increases a person’s vulnerability to illness.
the amount of stress a person feels is directly related to the number of stressors experienced.
small, bad events don’t cause stress, but large ones can be toxic.
4.To cope with stress, we tend to use _ _ _ strategies when we feel in control of our world, and to use _
_ _ strategies when we believe we cannot change a situation.
emotion-focused; problem-focused
problem-focused; emotion-focused
positive-emotion; negative-emotion
negative-emotion; positive-emotion

5.In which area of health psychology has the most research been done?
the definition of health
stress
biofeedback
changes in lifestyle

6.Imagine a family is moving to a new and larger home in a safer neighborhood with better schools. Will
this situation be a source of stress for the family?
No, because the change is a positive one.
No, because moving is not really stressful.
Yes, because any change requires adjustment.
Yes, because it provokes guilt that the family does not really deserve this good fortune.

7.In which stage of the general adaptation syndrome are the pituitary and adrenals stimulated?
exhaustion
alarm
reaction
resistance

8.Fantasizing about winning money is a problem-focused way of coping with financial stress.
True
False

9.Ana was recently in a car accident. In coping with the situation, she has focused on getting estimates
for her car repair, seeking medical treatment, and working with her insurance agent to obtain
compensation for the expenses of the car repair and her medical needs. Ana is employing problem-
focused coping.
True
False

77. A research article Functional Neuroimaging Study Revealed the impact of Early Parent-Infant
interactions on Children's Brain describes a study. In this study of 125 full -term and premature
newborns at Nationwide Children's Hospital in Columbus, Ohio, found early gentle displays of
affection from parents have lasting effects on how baby brains react to gentle touch the Authors found
that the more the parents hugged their toddlers , the more developed trains they had the children that
were hugged more also showed more positive brain responses than children whose parents hugged
them less study suggest that
A. early exposure to hugs stimulates positive brain responses and train development
B. it’s highly recommended for parents to hug their children as Frequently as possible
C. Hugging can have lasting effect on their children
A and B
B and C
All of the above (A,B and C)

Open questions

1. In psychology behavior is the means by which organisms adjust to their environment.


2. This method uses objectively collected information as the factual basis for drawing
conclusions. It consists of a set of orderly steps used to analyze and solve problems. This
method is called scientific method
3. Main goals of psychology are
Describing, explaining, predicting, controlling behavior and mental processes

4.If a psychological measure gives approximately the same evaluation of a personality trait on different
occasions, it is said to have reliability

5.If a psychological measure intends to measure happiness and if using this measure (e.g. questionnaire)
allows us to predict how happy someone is likely to be in particular situations, this questionnaire has Validity
6.To understand the effects of a brain damage, researchers examined every aspect of brain functioning of a
person after a serious car accident. I.e. they conducted a case study

I didn’t leave as much tip to the waiter as others from the group did and now I am perceived as stingy by them.
I just didn’t have more amount of money. It’s terrible when your actions are judged when those actions don’t
reflect your character or personality. But they won’t even bother find out more about me” – The persons
complaints come from getting her behavior judged based on _ _ _ .Fudemental attribution error (FAE)

3.In contrast to most people, depressed individuals tend to attribute negative outcomes to lasting, internal
causes such as their own traits or lack of ability, but attribute positive outcomes to temporary, external causes
such as good luck or special favors from others. This suggests that _ _ _ may play a role in
developing/protecting from depression..

self-serving bias

1. Parents use various reinforcement schedules to teach children new behavior, strengthen the
desired behavior or try to stop undesired behavior. A continuous schedule is often the best in
teaching a new behavior. Once the response has been learned_partial reinforcement _can be
used to strengthen the new behavior.
2. sex sells is a common saying in advertising with the knowledge about this form of learning, we can
fairly well explain how sexual images in advertisements can condition your response to a product
What is this form of learning. Classical Conditioning
3. You're watching a stand up comedy show with other people. The comedian says a joke you
don't understand but you still keep applauding with others. This may be due to _
informative_ _ _ _ _ _ _ _ _social influence
4. playing the piano uses ……procedural… memory
5. 65----Domestication/behavior specifically adaptation------------- is an action of human beings
and nonhuman animals that serves as the means to adapt to their environments.

6. - In contrast to most people, depressed individuals tend to attribute negative outcomes to


lasting, internal causes such as their own traits or lack of ability, but attribute positive
outcomes to temporary, external causes such as good luck or special favors from others. This
suggests that ----self-serving bias---------- may play a role in developing/protecting from
depression."
7.
. If a man's conditioned fear of spiders is triggered by the sight of other creatures that look like spiders, he is
demonstrating…stimulus generalization……….

You’re on your friends’ birthday party. You don’t think raising a toast is essential to show respect
towards someone but everyone else lifted their glasses and expressed their good wishes or desires
towards your friend, so even though you don’t feel like doing it, you raise a toast too. This is an
example of _ _ _ _normative _ _ _ _ _ _ _ _ _ social influence.

In yoga classes, students leam through watching their teacher and classmates. It is an demonstrates that
observational learning_ _ _ plays many roles in sports and activities.

Operationalization... method involves predetermined steps aimed at minimizing biases and errors to ask
questions . Gather and analyze data and answer the question accordingly
In contrast to most people, depressed individuals tend to attribute negative outcomes to lasting, internal causes such
as their own traits or lack of ability but attribute positive outcomes to temporary , external causes such as good luck
or special favors from others. This suggests that ….. Self-serving bias.... may play a role in developing protecting from
depression

Physicians' diagnoses are influenced by how easily they can bring different diseases to mind. This means that
they seem to use the ……. representativeness heuristic …… when making diagnosis

46. People's perceptions, including those of researchers, are subjective and can be influenced by their motives or
expectations. Looking at the same thing, different people might “see” different thing. What procedure is the
solution to minimize this observer bias?

Operationalize variables

if you believe that people are poor and obese only because they're lazy , you are committing the…Fundamental
Attribution…… error
9 in 10 college students rate condom as effective if it has supposed "95% success rate" in stopping HIV virus
that causes AIDS -only 4 in 10 think it successful given "5% failure rate" this is ……framing …… effect
88. Questionnaire, questionnaire aims at measuring people's levels of empathy and in fact it measure peoples
capacity to place oneself in another’s position we can say that this questionnaire..... is a Questionnaire for
Cognitive and Affective Empathy (QCAE).............\ high Validity

95. _ _ _ _ _ _ _ _ _ _ _ _ _ _ _ fundamental attribution error _ _ _ _ _ _ _ _ _ _ _ _ _ _ _ _ _ _ _ represents the


dual tendency for people to overestimate dispositional factors (blame or credit people) and to
underestimate situational factors (blame or credit the environment) when searching for the cause of some
behavior or outcome.
. After listening to a list of words such as hot, snow, warm, winter, ice, wet, frigid, chilly, heat, weather,
freeze, air, shiver, Arctic, and frost, people often remember having heard the related word cold even though
it was not presented (Gallo, 2006; Roediger & McDermott, 1995).

This happens due to _ _ _ _ _ _ _ _ _ _ _ _ _ _ _ _ encoded to long term memory _ _ _ _ _ _ _ _ _ _ _ _ _ _ _ _ _


______

. a hostile attitude toward members of a group, based solely on their membership in that group is referred to
as –Prejudice

- Joining the end of a ticket line is an example of -----conformity------whereas forming two lines when a theater
employee requests it is an example of compliance

Informed Consent---------is an agreement to participate in an experiment, granted in full awareness of the


nature of a research study, which has been explained in advance.

-Which heuristic can best explain why so many people believe in the pseudoscience of astrology?

------------------ Representativeness Heuristic ------------------

1. Improved memory for items at the end of a list is referred to as Recency Effect

1. When different people observe the same events, because of their personal motives or
expectations, they don’t always “see” the same thing. What can be a remedy / solution for this
challenge to objectivity? _ _ _ _ _ _ _ _ _ _ _ _ _ Operationalization_ _ _ _ _ _ _ _ _ _ _ _ _ _ _ _ _
______________

Most participants in the Holocaust were not sadists or psychopaths who enjoyed mass murder but
rather ordinary citizens subjected to complex and powerful social pressures. E.g. Adolf Eichmann, the
Nazi official responsible for the transportation of Jews to the death camps, was not the bloodthirsty
monster that many people made him out to be but rather a common bureaucrat who did what he
was told without questioning his orders.

Which form of social influence is exemplified by the case of Adolf Eichmann? Obedience
Charities might solicit money by ending their request with the words “even a penny would help.”
Such “even a penny” appeals substantially significantly the percentage of people who donate.

This may be due to _ _ _persuasion _ _ _ _ _ _ _ technique at work.

"I was walking down the street when I was approached by an eleven- or twelveyear-old boy. He
introduced himself and said that he was selling tickets to the annual Boy Scouts circus to be held on
the upcoming Saturday night. He asked if I wished to buy any at five dollars apiece. Since one of the
last places I wanted to spend Saturday evening was with the Boy Scouts, I declined. “Well,”he said, “if
you don’t want to buy any tickets, how about buying some of our big chocolate bars? They’re only a
dollar each.” I bought a couple and, right away, realized that something noteworthy had happened. I
knew that to be the case because: (a) I do not like chocolate bars; (b) I do like dollars; (c) I was
tanding there with two of his chocolate bars; and (d) he was walking away with two of my dollars.

Cialdini's (1984) story illustrates the power of _ _ door in the face _ _ _ _ _ _ _ _ _ _ _ technique

2. 4.The principle that subsequent retrieval of information is enhanced if cues received at the time
of recall are consistent with those present at the time of encoding is referred to as Encoding
specificity

3. Generic, categorical memory, such as the meaning of words and concepts is referred to as
Semantic Memory

4. Long-term memory for an autobiographical event and the context in which it occurred is
referred to as Episodic memory

5. A method of retrieval in which an individual is required to reproduce the information previously


presented is called recall

6. Internally or externally generated stimulus available to help with the retrieval of a memory. This
is a definition of retrieval cue

7. 10.People’s vivid and richly detailed memory in response to personal or public events that have
great emotional significance are called flashbulb memories
8. Memory processes associated with the preservation of information for retrieval at any later
time. This is a definition of Long term memory (LTM)

9. The process of putting information together based on general types of stored knowledge in the
absence of a specific memory representation is called Reconstructive memory

10. A method of retrieval in which an individual is required to identify stimuli as having been
experienced before is referred to as) recognition
Quiz 7:

1. Your parents ask you if you can think of 12 reasons why your university is better than its main rival
universities. You find it hard to come up with so many reasons and so end up thinking, “Hmm, maybe the
universities aren’t all that different.” Which mental strategies did you probably use to reach this conclusion?
available heuristic

2. George is definitely not the most attractive guy at the university, but he is extremely confident about who he
is and how he looks. He is convinced that most women find him to be very attractive, and he in fact usually
gets dates with girls who are much more attractive than he is. What is the best explanation of George’s
success? self-fulfilling prophecy

3. “Simple rules for making complex decisions or drawing inferences in a rapid manner and seemingly
effortless manner.” “A thinking strategy that enables quick, efficient judgments.” These are definitions of
heuristic

4. .Ana thinks religious people are more moral than atheists. After searching the internet, she only read the
articles that had titles which indicated that morality of of religious people is superior to the morality of people
who don’t believe in god. This is an example of confirmation bias

5. .Nine in 10 students rate a condom as effective if it has a supposed “95 percent success rate” in stopping the
HIV virus that causes AIDS; only 4 in 10 think it successful given a “5 percent failure rate” (Linville et al.,
1992). This is due to framing_ effect.

1. the same situations might cause stress in some people but not in others. According to Lazarus,
to experience some situations as stressful, first step is to evaluate the situations as stressful.
Cognitive appraisal is the cognitive interpretation and evaluation of a stressor.
2. If you believed that immigrants’ successes are due to government help but that their failures
are due to laziness, you would be committing the Fundamental Attribution Error
1. Which heuristic can best explain why so many people believe in the pseudoscience of astrology?
Representativeness

Heuristic

51-Researchers (Schemer et al; 2008) found that people were more interested in products that had
been embedded in music videos of artists that they liked and less likely to be interested when the
products were in videos featuring artists that they did not like. Which form of learning does finding
demonstrate?

ANSWER---- Classical Conditioning

53. Driving to school one wintry day, George narrowly misses a car that slides through a red light. "Slow
down! What a terrible driver." he thinks to himself, Moments later, George himself slips through an
intersection and yelps, Wow These roads are awful. The city snow plows need to get out here. What social
psychology principle has George just demonstrated?

ANSWER -------- Fundamental Attribution Error

96. When people doubt their ability to perform a task, they may engage in _ _ _ _ _ _ _ _
handicapping_ _ _ _ _ _ _ _ _ _ _ _ _ _ _ _ _ _ _ _ _ _ _ _ _ _ _ — They deliberately sabotage their
performance. The purpose of this strategy is to have a ready-made excuse for failure that does not
imply lack of ability.

97. George L. Engel’s _ _ _ _ _ _ _ _ _ _ _ biopsychosocial model of health and illness _ _ _ _ _ _ _ _ _ _ _


_ _ _ _ _ _ _ _ _ _ _ _ states that the causes, manifestations, and outcomes of wellness and diseases
are determined by the interactions between biological, psychological, and social factors.

98. The same situations might cause stress in some people but not in others. According to Lazarus, to
experience some situations as stressful, first step is to evaluate the situations as stressful. _ _ _ _ _ _ _
_ _ _ _ _ _ _ cognitive appraisal_ _ _ _ _ _ _ _ _ _ _ _ _ is the cognitive interpretation and evaluation of
a stressor.

99. According to Schachter, the experience of emotion is the joint effect of the two factors: _ _ _ _ _ _
_ _ _ _ _ _ _ _ _ _ _ psychological arousal _ _ _ _ _ _ _ _ _ _ _ _ _ _ _ _ and cognitive appraisal.
100. _ _ _ _ _ _ _ _ _ _ _ _ _ _ _ _ _ gender stereotypes _ _ _ _ _ _ _ _ _ _ _ _ _ _ _ are beliefs about
attributes and behaviors regarded as appropriate for males and females in a particular culture.

101. _ _ _ _ _ _ _ _ _ _ _ _ _ _ _ _ _ _ crystalized intelligence _ _ _ _ _ _ _ _ _ _ _ _ _ _ _ _ involves the


knowledge a person has already acquired and the ability to access that knowledge; it is measured by
tests of vocabulary, arithmetic, and general information.

. Four main goals of psychologists are describing, explaining, _ _ _ _ _ _ _ _ predicting _ _ _ _ _ _ _ _ _ _


_ and controlling behavior.

128. When different people observe the same events, because of their personal motives or
expectations, they don’t always “see” the same thing. What can be a remedy / solution for this
challenge to objectivity? _ _ _ _ _ _ _ _ _ _ _ _ _ _ _ __ standardization and operional definitions _ _ _ _
________________________

129. Parents use various reinforcement schedules to teach children new behavior, strengthen the
desired behavior or try to stop undesired behavior. A continuous schedule is often the best in teaching
a new behavior. Once the response has been learned, _ _ _ _ _ _ _ _ positive reinforcement _ _ _ _ _ _
_ _ _ _ _ _ _ _ _ _ _ _ _ can be used to strengthen the new behavior.

130

-"Sex Sells” is a common saying in advertising. With the knowledge about this form of learning, we can
fairly well explain how sexual images in advertisements can condition your response to a product. What
is this form of learning. “Classical conditioning” Unconditioned stimuli.

- Researchers (Schemer et al. 2008) found that people were more interested in products that had been
embedded in music videos of artists that they liked and less likely to be interested when the products
were in videos featuring artists they did like. Which form of learning does finding demonstrate? Classical
conditioning
_ fundamental attribution error_ _ _ _ _ _ _ _ _ _ _ _ _ _ _ _ _ _ _ _ _ _ _ _ _ _ _ _ _ _ _ _ _ represents
the dual tendency for people to overestimate dispositional factors (blame or credit people) and to
underestimate situational factors (blame or credit the environment) when searching for the cause of
some behavior or outcome.

(1 point)
When people doubt their ability to perform a task, they may engage in _self-handicapping _ _ _ _ _ _ _
_ _ _ _ _ _ _ _ _ _ _ _ _ _ _ _ _ _ _ _ _ _ _ _ _ _ — They deliberately sabotage their performance. The
purpose of this strategy is to have a ready-made excuse for failure that does not imply lack of ability.

(1 point)

George L. Engel’s _ _ _ _ biopsychosocial model of health and illness._ _ _ _ _ _ _ _ _ _ _ _ _ _ _ _ _ _ _ _


_ _ _ _ _ _ _ _ _ _ states that the causes, manifestations, and outcomes of wellness and diseases are
determined by the interactions between biological, psychological, and social factors.

(1 point)

The same situations might cause stress in some people but not in others. According to Lazarus, to
experience some situations as stressful, first step is to evaluate the situations as stressful. _ _ _ _ _ _ _
_ _ _ _ _ _ _ _ _ _ _ _ cognitive appraisal_ _ _ _ _ _ _ _ _ _ _ _ _ is the cognitive interpretation and
evaluation of a stressor.

According to Schachter, the experience of emotion is the joint effect of the two factors: _ _ _ _ _ _ _ _
_ _ _ _ psychological arousal_ _ _ _ _ _ _ _ _ _ _ _ _ _ _ _ _ _ _ _ _ and cognitive appraisal.

_ _ _ _ _ _ _ _ _ _ gender streotypes /gender role_ _ _ _ _ _ _ _ _ _ _ _ _ _ _ _ _ _ _ _ _ _ are beliefs


about attributes and behaviors regarded as appropriate for males and females in a particular culture.

_ _ _ _ _ _ _ _ _ _ _ _ _ _ _ _ crystallized intelligence_ _ _ _ _ _ _ _ _ _ _ _ _ _ _ _ _ _ involves the


knowledge a person has already acquired and the ability to access that knowledge; it is measured by
tests of vocabulary, arithmetic, and general information.

2. Ana , the president of the Psychology Club , needs assistance preparing for an
upcoming fund - raising event She made a plea for help in an e - mail she sent in a mass
mailing to all 60 club members however nobody has responded help or not help What
might be the problem ? (What might be causing people's failure to help her ? ) diffusion
of responsibility
3. Four main goals of psychologists are describing, explaining, _ _ _ _ _ _ predicting _ _ _ _
_ _ _ _ _ _ _ _ _ and controlling behavior.
4.

5.
6.
7. (1 point) Parents use various reinforcement schedules to teach children new behavior,
strengthen the desired behavior or try to stop undesired behavior. A continuous
schedule is often the best in teaching a new behavior. Once the response has been
learned, _ _ _ _ _ _ partial reinforcement _ _ _ _ _ _ _ _ _ _ _ _ _ _ _ _ _ _ _ _ _ can be
used to strengthen the new behavior.
8.
9. (1 point) After listening to a list of words such as hot, snow, warm, winter, ice, wet, frigid,
chilly, heat, weather, freeze, air, shiver, Arctic, and frost, people often remember having
heard the related word cold even though it was not presented (Gallo, 2006; Roediger &
McDermott, 1995).
10. This happens due to _ _ _ _ _ _ _ _ _ _ _ _ _ _ _long term memory “not sure” _ _ _ _ _ _
_________________________

Name the conditioned response. (Referring to the situation described in the question #4)

Feeling nausea as a result of watching violence


Question N1:

Making predictions about future behavior is dependent on having accurate explanations of the
behavior of interest.

Answer: False

Point: 0.00

Question N2:

“In this study, Japanese students refrained themselves from expressing as anger and disgust because
in Japanese culture it’s not that desirable to show negative emotions.” This statement is likely made
by a psychologist working in _ _ _ perspective of psychology.

Answer:

the cognitive perspective

Point: 0.00

Question N3:

Our beliefs about ourselves matter. Do we see ourselves as in control of things? Do we view
ourselves as relatively superior or inferior? Our answers influence our emotions and actions.” This
statement is likely made by a psychologist working in _ _ _ perspective of psychology.

Answer:

the behaviorist perspective

Point: 0.00

Question N4:

About how many items can be held in short-term memory?

Answer:

seven

Point: 0.50

Question N5:

Correlation coefficients of .50 and −.50 indicate relationships that are of different strengths.
Answer: False

Point: 0.50

Question N6:

In case studies researchers gather large amounts of descriptive data relatively quickly and
inexpensively.

Answer: True

Point: 0.00

Question N7:

If a study shows positive correlation between holding authoritarian beliefs and infectious diseases, it
means that

Answer:

people with more authoritarian beliefs tend to have infectious diseases more

Point: 0.50

Question N8:

In the longest study (nearly 80 years) from Harvard researchers who have analyzed the rich data
coming from thousands of participants, including vast medical records and hundreds of in-person
interviews and questionnaires, found a strong correlation between people’s flourishing lives and
their relationships with family, friends, and community. Several studies found that people’s level of
satisfaction with their relationships at age 50 was a better predictor of physical health than their
cholesterol levels were. The data showed that those who kept warm relationships got to live longer
and happier and the loners often died earlier.

The conclusion from this study is that

Answer:

Our relationships and how happy we are in our relationships has a powerful influence on our health.

Point: 0.00

Question N9:
Ana took a personality test and the results indicated she is an extroverted person. After 2 weeks she
took the test again and now she turned out to be an introvert. This test probably has

Answer:

low reliability

Point: 0.50

Question N10:

Any institution intending to do a psychological research is required to have an institutional review


board that reviews the ethical aspects of the research before it is conducted.

Answer: True

Point: 0.50

Question N11:

Descriptive (observational) and correlational studies describe behavior, detect relationships, and
predict behavior. But to explain behaviors, psychologists use

Answer:

case studies.

Point: 0.00

Question N12:

The predictions implied by a theory are called

Answer:

hypotheses.

Point: 0.50

Question N13:

Knowing that two events are negatively correlated provides

Answer:
a basis for prediction.

Point: 0.50

Question N14:

Which of the following statements is true?

Answer:

Experiments can reveal cause-and-effect relationships between variables unless the researchers use
randomization.

Point: 0.00

Question N15:

Under a variable interval schedule, reinforcement will be delivered after a specified average number
of responses have been made.

Answer: False

Point: 0.50

Question N16:

Bedwetting alarms have been used against nocturnal enuresis in adults since the beginning of the
20th century. The enuresis alarm is triggered when a sensor in the sheets or night clothes becomes
wet with urine. This sets off a signal which wakes the person to void in the toilet.

Enuresis alarms “teach” the users to wake up when they need to urinate. What kind of learning is
used?

Answer:

D. Sensitization

Point: 0.00

Question N17:

Bedwetting alarms have been used against nocturnal enuresis in adults since the beginning of the
20th century. The enuresis alarm is triggered when a sensor in the sheets or night clothes becomes
wet with urine. This sets off a signal which wakes the person to void in the toilet.
Enuresis alarms “teach” the users to wake up when they need to urinate. In terms of classical
conditioning, urinating in sleep is

Answer:

Positively reinforced

Point: 0.00

Question N18:

According to the observational learning principle, children do what they see. If a model’s actions and
words are inconsistent, children may imitate the hypocrisy they observe.

Answer: True

Point: 0.50

Question N19:

In classical conditioning, the consequences of behavior, such as rewards and punishments, influence
the probability that the behavior will occur again

Answer: True

Point: 0.00

Question N20:

In Pavlov’s early work, bell is to food as

Answer:

unconditioned response is to conditioned stimulus.

Point: 0.00

Question N21:

In Watson’s work with Little Albert, why was Albert afraid of the Santa Claus mask?

Answer:

He generalized his learned fear of the rat.


Point: 0.50

Question N22:

Advertisers often exploit classical conditioning principles. They try to create associations in people’s
mind between their products (for example, sports cars) and passion.

In this case, the elements in their advertisement (e.g. sexually provocative individuals or situations)
serve as _ _ _ and feelings of sexual arousal is _ _ _ . The product itself (in this case, sports cars) is _
_ _ , so that the feelings of arousal will become associated with it.

Answer:

unconditioned stimulus | neutral stimulus | conditioned response

Point: 0.00

Question N23:

Your dog is barking so loudly that it’s making your ears ring. You clap your hands, the dog stops
barking, your ears stop ringing, and you think to yourself, “I’ll have to do that when he barks again.”
The end of the barking was for you a

Answer:

negative reinforcer.

Point: 0.50

Question N24:

If you were going to use Bandura’s findings in developing a program to prevent violence among
middle school children, you might

Answer:

have children watch videos of children who are responding constructively to aggressive acts on the
playground.

Point: 0.50

Question N25:
The value of elaborative rehearsal over maintenance rehearsal has been cited as evidence for the
Levels of Processing model of memory.

Answer: True

Point: 0.50

Question N26:

If it is easier to remember something in the place where you learned it, you have context-dependent
memory.

Answer: True

Point: 0.50

Question N27:

Which one of the following statements about memory is correct?

Answer:

Recognition is a method of retrieval in which an individual is required to reproduce the information


to which he/she was previously exposed.

Point: 0.00

Question N28:

Maria has always loved to wear winter clothing. Which of the following statements might be
correct?

Answer:

D. An implicit memory of feeling safe and secure in winter clothing with her parents may be
triggering this vague sense of comfort.

Point: 0.00

Question N29:

Many factors might possibly affect our judgment, (such as the credibility of the person giving the
information, the emotional state of the listener etc.) but not the way information is presented.

Answer: False
Point: 0.50

Question N30:

When consumers respond more positively to ground beef described as “75 percent lean” than to the
same product labeled “25 percent fat,” they have been influenced by

Answer:

framing effects

Point: 0.50

Question N31:

Which one of the following would be an example of the confirmation bias at work?

Answer:

Ana ignores negative information about her favorite political candidate.

Point: 0.50

Question N32:

Even when people knew that the author’s choice of an essay topic was externally caused, they
assumed that what he wrote reflected how he really felt about Castro. That is, they made a mistake
based on self-serving attributional bias

Answer: True

Point: 0.00

Question N33:

People tend to make self-serving attributions which means they have the tendency to believe that
people’s behavior corresponds to (matches) their dispositions.

Answer: False

Point: 0.50

Question N34:

“I’m good; you are lucky” – The phrase shortly summarizes the idea behind self-serving attributional
bias.
Answer: True

Point: 0.50

Question N35:

Which statement most clearly reflects the fundamental attribution error?

Answer:

People who are unemployed are too lazy to work.

Point: 0.50

Question N36:

“Victory finds a hundred fathers but defeat is an orphan.” (Count Galeazzo Ciano, The Ciano Diaries,
1938). Match the phrase of a famous author with the corresponding social psychological notion.

Answer:

Self-serving bias

Point: 0.50

Question N37:

Couples in happy marriages tend to make _ _ _ attributions for each other’s positive actions (“That’s
just the way he is”) and _ _ _ attributions for negative actions (“He’s just so busy providing for all of
us that he doesn’t have the time”).

Answer:

internal | external

Point: 0.50

Question N38:

Polar plunges started as sanctioned and carefully planned fund-raisers such as the one depicted
here. But when teenagers started daring each other to jump into icy-cold water on their own and
post videos of these exploits online, the desire to be accepted and liked by others led to dangerous
and even deadly behavior. This exemplifies the power of normative social influence.
Answer: True

Point: 0.50

Question N39:

In Milgram’s original study, about what proportion of the “teacher-subjects” gave the maximum
shock?

Answer:

about two-thirds

Point: 0.50

Question N40:

The Stanford prison experiment illustrates the power of to influence people’s behavior.

Answer:

the situation

Point: 0.50

Question N41:

What was the independent variable in the Stanford Prison Experiment?

Answer:

prisoner or guard roles

Point: 0.50

Question N42:

If you wanted to stop bullying in your school, what would most likely be an effective strategy to
follow?

Answer:

Change the entire school system to have zero tolerance for bullying
Point: 0.50

Question N43:

In Asch’s line studies, participants who were alone when asked to report the length of the lines gave
the correct answer 98% of the time. However, when they were with the confederates who
sometimes gave an obviously wrong answer, 76% of participants gave the wrong answer at least
once. This suggests that Asch’s studies are an illustration of

Answer:

public compliance without private acceptance.

Point: 0.50

Question N44:

Altruism is any helping behavior that a person carries out to increase other people’s warfare.

Answer: True

Point: 0.00

Question N45:

A stereotype is

Answer:

cognitive summary that can be positive or negative

Point: 0.00

Question N46:

What is stereotype threat?

Answer:

Some members of a stereotyped group feel anxious when they are made aware of a stereotype
about them.

Point: 0.50

Question N47:
George knows and likes most of his Armenian classmates but privately believes that his Georgian
culture is superior to all others. His belief is evidence of his

Answer:

ethnocentrism.

Point: 0.50

Question N48:

A prejudice is

Answer:

a hostile attitude toward members of a group, based solely on their membership in that group.

Point: 0.50

Question N49:

What is a suppressed prejudice?

Answer:

A person knows he or she is prejudiced but chooses not to express it in public.

Point: 0.50

Question N50:

Fantasizing about winning money is a problem-focused way of coping with financial stress.

Answer: True

Point: 0.00

Question N51:

Using of antianxiety medication is a problem-directed stress-coping strategy.

Answer: False
Point: 0.50

Question N52:

Selye’s general adaptation syndrome consists of an alarm reaction followed by

Answer:

resistance then exhaustion.

Point: 0.50

Question N53:

In which stage of the general adaptation syndrome are the pituitary and adrenals stimulated?

Answer:

exhaustion

Point: 0.00

Question N54:

Which stage of the general adaptation syndrome is associated with the outcome of disease?

Answer:

exhaustion

Point: 0.50

Question N55:

The traditional biomedical model of health links among the nervous system, the immune system,
behavioral styles, cognitive processing, and environmental domains of health.

Answer: True

Point: 0.00

Question N56:

Health constitutes the freedom from disease, pain, or defect.


Answer: False

Point: 0.50

Question N57:

Out of the seven strategies listed below, which ones does Problem-Directed Stress Coping involve?

1. Fight (destroy, remove, or weaken the threat)

2. Somatically focused activities (use of antianxiety medication, relaxation, biofeedback)

3. Flight (distance oneself from the threat)

4. Cognitively focused activities (planned distractions, fantasies, thoughts about oneself)

5. Seek options to fight-or-flight (negotiating, bargaining, compromising)

6. Therapy to adjust conscious or unconscious processes that lead to additional anxiety

7. Prevent future stress (act to increase one’s resistance or decrease strength of anticipated
stress)

Answer:

1, 3, 5, 7

Point: 0.50

Question N58:

A person is aware their behavior is problematic, but are not committed to do anything about it.
According to the Prochaska’s model, he/she is on the stage of

Answer:

Contemplation

Point: 0.50
Question N59:

A person has to work hard to prevent relapse. According to the Prochaska’s model, he/she is on the
stage of

Answer:

Maintenance

Point: 0.50
Question N1:

Making predictions about future behavior is dependent on having accurate explanations of the
behavior of interest.

Answer: False

Point: 0.00

Question N2:

“In this study, Japanese students refrained themselves from expressing as anger and disgust because
in Japanese culture it’s not that desirable to show negative emotions.” This statement is likely made
by a psychologist working in _ _ _ perspective of psychology.

Answer:

the cognitive perspective

Point: 0.00

Question N3:

Our beliefs about ourselves matter. Do we see ourselves as in control of things? Do we view
ourselves as relatively superior or inferior? Our answers influence our emotions and actions.” This
statement is likely made by a psychologist working in _ _ _ perspective of psychology.

Answer:

the behaviorist perspective

Point: 0.00

Question N4:

About how many items can be held in short-term memory?

Answer:

seven

Point: 0.50

Question N5:

Correlation coefficients of .50 and −.50 indicate relationships that are of different strengths.
Answer: False

Point: 0.50

Question N6:

In case studies researchers gather large amounts of descriptive data relatively quickly and
inexpensively.

Answer: True

Point: 0.00

Question N7:

If a study shows positive correlation between holding authoritarian beliefs and infectious diseases, it
means that

Answer:

people with more authoritarian beliefs tend to have infectious diseases more

Point: 0.50

Question N8:

In the longest study (nearly 80 years) from Harvard researchers who have analyzed the rich data
coming from thousands of participants, including vast medical records and hundreds of in-person
interviews and questionnaires, found a strong correlation between people’s flourishing lives and
their relationships with family, friends, and community. Several studies found that people’s level of
satisfaction with their relationships at age 50 was a better predictor of physical health than their
cholesterol levels were. The data showed that those who kept warm relationships got to live longer
and happier and the loners often died earlier.

The conclusion from this study is that

Answer:

Our relationships and how happy we are in our relationships has a powerful influence on our health.

Point: 0.00

Question N9:
Ana took a personality test and the results indicated she is an extroverted person. After 2 weeks she
took the test again and now she turned out to be an introvert. This test probably has

Answer:

low reliability

Point: 0.50

Question N10:

Any institution intending to do a psychological research is required to have an institutional review


board that reviews the ethical aspects of the research before it is conducted.

Answer: True

Point: 0.50

Question N11:

Descriptive (observational) and correlational studies describe behavior, detect relationships, and
predict behavior. But to explain behaviors, psychologists use

Answer:

case studies.

Point: 0.00

Question N12:

The predictions implied by a theory are called

Answer:

hypotheses.

Point: 0.50

Question N13:

Knowing that two events are negatively correlated provides

Answer:
a basis for prediction.

Point: 0.50

Question N14:

Which of the following statements is true?

Answer:

Experiments can reveal cause-and-effect relationships between variables unless the researchers use
randomization.

Point: 0.00

Question N15:

Under a variable interval schedule, reinforcement will be delivered after a specified average number
of responses have been made.

Answer: False

Point: 0.50

Question N16:

Bedwetting alarms have been used against nocturnal enuresis in adults since the beginning of the
20th century. The enuresis alarm is triggered when a sensor in the sheets or night clothes becomes
wet with urine. This sets off a signal which wakes the person to void in the toilet.

Enuresis alarms “teach” the users to wake up when they need to urinate. What kind of learning is
used?

Answer:

D. Sensitization

Point: 0.00

Question N17:

Bedwetting alarms have been used against nocturnal enuresis in adults since the beginning of the
20th century. The enuresis alarm is triggered when a sensor in the sheets or night clothes becomes
wet with urine. This sets off a signal which wakes the person to void in the toilet.
Enuresis alarms “teach” the users to wake up when they need to urinate. In terms of classical
conditioning, urinating in sleep is

Answer:

Positively reinforced

Point: 0.00

Question N18:

According to the observational learning principle, children do what they see. If a model’s actions and
words are inconsistent, children may imitate the hypocrisy they observe.

Answer: True

Point: 0.50

Question N19:

In classical conditioning, the consequences of behavior, such as rewards and punishments, influence
the probability that the behavior will occur again

Answer: True

Point: 0.00

Question N20:

In Pavlov’s early work, bell is to food as

Answer:

unconditioned response is to conditioned stimulus.

Point: 0.00

Question N21:

In Watson’s work with Little Albert, why was Albert afraid of the Santa Claus mask?

Answer:

He generalized his learned fear of the rat.


Point: 0.50

Question N22:

Advertisers often exploit classical conditioning principles. They try to create associations in people’s
mind between their products (for example, sports cars) and passion.

In this case, the elements in their advertisement (e.g. sexually provocative individuals or situations)
serve as _ _ _ and feelings of sexual arousal is _ _ _ . The product itself (in this case, sports cars) is _
_ _ , so that the feelings of arousal will become associated with it.

Answer:

unconditioned stimulus | neutral stimulus | conditioned response

Point: 0.00

Question N23:

Your dog is barking so loudly that it’s making your ears ring. You clap your hands, the dog stops
barking, your ears stop ringing, and you think to yourself, “I’ll have to do that when he barks again.”
The end of the barking was for you a

Answer:

negative reinforcer.

Point: 0.50

Question N24:

If you were going to use Bandura’s findings in developing a program to prevent violence among
middle school children, you might

Answer:

have children watch videos of children who are responding constructively to aggressive acts on the
playground.

Point: 0.50

Question N25:
The value of elaborative rehearsal over maintenance rehearsal has been cited as evidence for the
Levels of Processing model of memory.

Answer: True

Point: 0.50

Question N26:

If it is easier to remember something in the place where you learned it, you have context-dependent
memory.

Answer: True

Point: 0.50

Question N27:

Which one of the following statements about memory is correct?

Answer:

Recognition is a method of retrieval in which an individual is required to reproduce the information


to which he/she was previously exposed.

Point: 0.00

Question N28:

Maria has always loved to wear winter clothing. Which of the following statements might be
correct?

Answer:

D. An implicit memory of feeling safe and secure in winter clothing with her parents may be
triggering this vague sense of comfort.

Point: 0.00

Question N29:

Many factors might possibly affect our judgment, (such as the credibility of the person giving the
information, the emotional state of the listener etc.) but not the way information is presented.

Answer: False
Point: 0.50

Question N30:

When consumers respond more positively to ground beef described as “75 percent lean” than to the
same product labeled “25 percent fat,” they have been influenced by

Answer:

framing effects

Point: 0.50

Question N31:

Which one of the following would be an example of the confirmation bias at work?

Answer:

Ana ignores negative information about her favorite political candidate.

Point: 0.50

Question N32:

Even when people knew that the author’s choice of an essay topic was externally caused, they
assumed that what he wrote reflected how he really felt about Castro. That is, they made a mistake
based on self-serving attributional bias

Answer: True

Point: 0.00

Question N33:

People tend to make self-serving attributions which means they have the tendency to believe that
people’s behavior corresponds to (matches) their dispositions.

Answer: False

Point: 0.50

Question N34:

“I’m good; you are lucky” – The phrase shortly summarizes the idea behind self-serving attributional
bias.
Answer: True

Point: 0.50

Question N35:

Which statement most clearly reflects the fundamental attribution error?

Answer:

People who are unemployed are too lazy to work.

Point: 0.50

Question N36:

“Victory finds a hundred fathers but defeat is an orphan.” (Count Galeazzo Ciano, The Ciano Diaries,
1938). Match the phrase of a famous author with the corresponding social psychological notion.

Answer:

Self-serving bias

Point: 0.50

Question N37:

Couples in happy marriages tend to make _ _ _ attributions for each other’s positive actions (“That’s
just the way he is”) and _ _ _ attributions for negative actions (“He’s just so busy providing for all of
us that he doesn’t have the time”).

Answer:

internal | external

Point: 0.50

Question N38:

Polar plunges started as sanctioned and carefully planned fund-raisers such as the one depicted
here. But when teenagers started daring each other to jump into icy-cold water on their own and
post videos of these exploits online, the desire to be accepted and liked by others led to dangerous
and even deadly behavior. This exemplifies the power of normative social influence.
Answer: True

Point: 0.50

Question N39:

In Milgram’s original study, about what proportion of the “teacher-subjects” gave the maximum
shock?

Answer:

about two-thirds

Point: 0.50

Question N40:

The Stanford prison experiment illustrates the power of to influence people’s behavior.

Answer:

the situation

Point: 0.50

Question N41:

What was the independent variable in the Stanford Prison Experiment?

Answer:

prisoner or guard roles

Point: 0.50

Question N42:

If you wanted to stop bullying in your school, what would most likely be an effective strategy to
follow?

Answer:

Change the entire school system to have zero tolerance for bullying
Point: 0.50

Question N43:

In Asch’s line studies, participants who were alone when asked to report the length of the lines gave
the correct answer 98% of the time. However, when they were with the confederates who
sometimes gave an obviously wrong answer, 76% of participants gave the wrong answer at least
once. This suggests that Asch’s studies are an illustration of

Answer:

public compliance without private acceptance.

Point: 0.50

Question N44:

Altruism is any helping behavior that a person carries out to increase other people’s warfare.

Answer: True

Point: 0.00

Question N45:

A stereotype is

Answer:

cognitive summary that can be positive or negative

Point: 0.00

Question N46:

What is stereotype threat?

Answer:

Some members of a stereotyped group feel anxious when they are made aware of a stereotype
about them.

Point: 0.50

Question N47:
George knows and likes most of his Armenian classmates but privately believes that his Georgian
culture is superior to all others. His belief is evidence of his

Answer:

ethnocentrism.

Point: 0.50

Question N48:

A prejudice is

Answer:

a hostile attitude toward members of a group, based solely on their membership in that group.

Point: 0.50

Question N49:

What is a suppressed prejudice?

Answer:

A person knows he or she is prejudiced but chooses not to express it in public.

Point: 0.50

Question N50:

Fantasizing about winning money is a problem-focused way of coping with financial stress.

Answer: True

Point: 0.00

Question N51:

Using of antianxiety medication is a problem-directed stress-coping strategy.

Answer: False
Point: 0.50

Question N52:

Selye’s general adaptation syndrome consists of an alarm reaction followed by

Answer:

resistance then exhaustion.

Point: 0.50

Question N53:

In which stage of the general adaptation syndrome are the pituitary and adrenals stimulated?

Answer:

exhaustion

Point: 0.00

Question N54:

Which stage of the general adaptation syndrome is associated with the outcome of disease?

Answer:

exhaustion

Point: 0.50

Question N55:

The traditional biomedical model of health links among the nervous system, the immune system,
behavioral styles, cognitive processing, and environmental domains of health.

Answer: True

Point: 0.00

Question N56:

Health constitutes the freedom from disease, pain, or defect.


Answer: False

Point: 0.50

Question N57:

Out of the seven strategies listed below, which ones does Problem-Directed Stress Coping involve?

1. Fight (destroy, remove, or weaken the threat)

2. Somatically focused activities (use of antianxiety medication, relaxation, biofeedback)

3. Flight (distance oneself from the threat)

4. Cognitively focused activities (planned distractions, fantasies, thoughts about oneself)

5. Seek options to fight-or-flight (negotiating, bargaining, compromising)

6. Therapy to adjust conscious or unconscious processes that lead to additional anxiety

7. Prevent future stress (act to increase one’s resistance or decrease strength of anticipated
stress)

Answer:

1, 3, 5, 7

Point: 0.50

Question N58:

A person is aware their behavior is problematic, but are not committed to do anything about it.
According to the Prochaska’s model, he/she is on the stage of

Answer:

Contemplation

Point: 0.50
Question N59:

A person has to work hard to prevent relapse. According to the Prochaska’s model, he/she is on the
stage of

Answer:

Maintenance

Point: 0.50

You might also like